738
Orthopaedic Oncology OITE Review 2012 8.29.2012 Special thanks to Matteos, Boulton, Kujo, First Aid for the Boards Those who have come before us Ruth Delaney, MD

Oncology oite-review-2012

Embed Size (px)

Citation preview

Page 1: Oncology oite-review-2012

Orthopaedic OncologyOITE Review 2012

8.29.2012

Special thanks to Matteos, Boulton, Kujo, First Aid for the Boards

Those who have come before us

Ruth Delaney, MD

Page 2: Oncology oite-review-2012

Introduction

• History: age, sex, duration of sx, presence and quality of pain, history of trauma, weight loss, smoking history and history of prior malignancy– Red flags: pain that

extends beyond expected duration, night pain, T-spine pain

– Beware the history of trauma

Page 3: Oncology oite-review-2012

Radiographic Differential Diagnosis

Bone Forming Tumors Cartilage Forming Tumors The Rest

Osteoid Osteoma Osteochondroma InfectionOsteoblastoma Chondromyxoid fibroma MetastasesOsteosarcoma Chondroblastoma Round cell tumorsBlastic Metastases Enchondroma Fibrous DysplasiaPaget's Disease Chondrosarcoma Non-Ossifying Fibroma(Fibrous Dysplasia) Simple Bone Cyst

Aneurysmal Bone CystHistiocytosisGiant cell tumorMetabolic condition

PINK BLUE

Page 4: Oncology oite-review-2012

Bone-Producing Tumors

• Osteoid Osteoma• Osteoblastoma• Enostosis (bone island)• Osteosarcoma

Page 5: Oncology oite-review-2012

Bone Forming Tumors• Possibilities: fracture callus,

myositis ossificans, fibrous dysplasia, osteosarcoma, osteoblastoma, osteoid osteoma

• WOVEN bone & spindle cell stroma– Bone differentiates reactive

from neoplastic– Stromal cells differentiate

benign from malignant• Woven bone: reactive (OB

rimming) vs neoplastic (no OB rimming)

• Malignant stroma: hypercellularity, atypia, pleomorphism, high mitotic rate, etc.

Page 6: Oncology oite-review-2012

Osteoid Osteoma• Most common first two decades• Classic pattern of constant pain

relieved by ASA/NSAIDs• Proximal femur is the most

common location followed by the tibia, posterior elements of the spine, and the humerus

• Osteoid Osteoma is found in the diaphysis or the metaphysis of the proximal end of the bone more often than the distal end

• Small lytic nidus with target appearance

• Four diagnostic features include (1) a sharp round or oval lesion that is (2) less than 2 cm in diameter, (3) has a homogeneous dense center and (4) a 1-2 mm peripheral radiolucent zone

• CT scan as preferred method of evaluation

Page 7: Oncology oite-review-2012

Osteoid Osteoma

Page 8: Oncology oite-review-2012

Osteoid Osteoma

• Histology: Osteoid trabeculae in loose, vascular, stromal connective tissue

• Rx: – NSAIDs x 2 yrs (50%

successful)– CT-guided radiofrequency

ablation (95% successful)

Page 9: Oncology oite-review-2012

Osteoid Osteoma

• Affects young (<30 yo old), lower extremity, pain gets better with NSAIDS

• Imaging shows a central nidus with reactive bone, hot on bone scan, always <1-1.5cm.

• Treatment: RF ablation• Histology: woven bone, sharp border• *most common benign bone tumor of

carpal bones

Page 10: Oncology oite-review-2012

Osteoblastoma• ~80% of these tumors occur in

patients younger than 30 years• ~40% located in the spine--

usually involve the posterior elements, and 17% of spinal osteoblastomas in the sacrum

• Osteoblastoma of the long tubular bones (LE>>UE) is often diaphyseal

• On x-ray--appear as a radio-lucent defect with a central density due to ossification. The lesion is well circumscribed and may have a surrounding sclerosis

• Classic: calcified lesion in posterior elements of the spine

Page 11: Oncology oite-review-2012

Osteoblastoma

• Histology: Osteoblastic rimming of trabeculae

• Rx: excision with at least a Marginal Margin because recurrence rate 20% (40%+ w/curettage)

Page 12: Oncology oite-review-2012

Osteoid Osteoma vs. Osteoblastoma

• Common• Usually < 1 cm• Regular tissue

pattern• Pain

constant/nocturnal• Pain often relieved

with NSAIDs• Axial involvement

uncommon

• Uncommon• Usually > 2 cm• Irregular tissue

pattern• Pain sporadic• Pain not relieved

with NSAIDs• Axial

involvement

common

Page 13: Oncology oite-review-2012

Bone Island or Enostosis

• Focus of mature cortical bone within the cancellous bone

• Cold on bone scan

Page 14: Oncology oite-review-2012

Osteosarcoma

• Osteosarcoma is very rare in young children

• Incidence increases steadily with age, increasing more dramatically in adolescence, corresponding with the growth spurt

• Typically occurs in first 3 decades with second peak beyond sixth decade

Page 15: Oncology oite-review-2012

Osteosarcoma

• most commonly occurs in the extremities of long bones near metaphyseal growth plates

• most common sites are the femur (42%, 75% of which are distal femur), tibia (19%, 80% of which are proximal tibia), and humerus (10%, 90% of which are proximal humerus)

• other significant locations are the skull and jaw (8%) and pelvis (8%)

Page 16: Oncology oite-review-2012

Osteosarcoma

• most commonly occurs in the extremities of long bones near metaphyseal growth plate

• OCCURS AT MOST METABOLICALLY ACTIVE SITES

• The most common sites are the femur (41%, 75% of which are distal femur), tibia (19%, 80% of which are proximal tibia), and humerus (10%, 90% of which are proximal humerus).

• Other significant locations are the skull and jaw (8%) and pelvis (8%)

Most in the Distal Femur!!!Second Most in Proximal Tibia!!!

Page 17: Oncology oite-review-2012

Osteosarcoma• most common presenting

symptom is pain, particularly pain with activity

• lesions can be purely osteolytic (~30% of cases), purely osteoblastic (~45% of cases), or a mixture of both

• On xray, permeative metaphyseal lesions with soft tissue extension and new bone formation

• Periosteal reaction is common and frequently takes on a “sunburst” appearance

• MRI of the primary lesion is the best method to assess the extent of intramedullary disease as well as associated soft tissue masses and skip lesions

Page 18: Oncology oite-review-2012

Osteosarcoma

• Histology: spindle–shaped tumor cells that produce osteoid

• Rx:Neo chemorestage resection Maintenance chemo = 60-70% long term survival with localized OSA

• Surgery alone = 10-20% pt survival

• Increased risk OSA: missing Rb gene (Ch13), Paget’s, prior XRT

Page 19: Oncology oite-review-2012

Osteosarcoma Sub-types• Conventional (~90%)-

usually begins in medullary canal

• Parosteal (~5%)-low grade “lobulated” attachment to cortex/75% at posterior aspect of distal femur

• Telengiectatic (~4%)-purely lytic, “bag of blood”, confused with ABC but has spindle cell

• Periosteal (2%)-juxtacortical, diaphyseal, “sunburst,” creates “crater”

• Others: well-diff, IM, small cell, multicentric

Page 20: Oncology oite-review-2012

Osteosarcoma Sub-Types

• High- grade intramedullary• Telangiectatic • Parosteal• Periosteal

Page 21: Oncology oite-review-2012

High-grade intramedullary

• Most Common type of OS• 75% present as IIB, and the other 25%

present as mets (lung)• 50% Occur around the knee• *image entire bone to look for skip mets

(equivalent to mets).• Hot on Bone scan, Histology is pink osteoid.• Tx-Chemo, surgery, chemo• Percentage of tumor necrosis in response to

chemo is main prognostic factor.

Page 22: Oncology oite-review-2012

Telangiectatic Osteosarcoma

• Looks similar to ABC – Watch out• Lytic with no mineralization• * if located on ulnar side of DR= OS• * if located on radial side of DR = GCT

Page 23: Oncology oite-review-2012

Parosteal Osteosarcoma

• Low grade surface lesion- “stuck on bone”• 80% distal posterior femur, painless mass• Often confused with Fibrous dysplasia by

pathologist• Bland spindle cells around the bone• Treatment- wide surgical excision- NO

CHEMO.

Page 24: Oncology oite-review-2012

Periosteal Osteosarcoma

• Rare, diaphyseal location, second decade, sunburst pattern , chondroblastic

• Treatment: chemo, surgery, chemo

• Prognosis is worse than parosteal but not as bad as high-grade intramedullary.

Page 25: Oncology oite-review-2012

Summary – Osteoblastoma & Osteosarcoma

Page 26: Oncology oite-review-2012

Osteoblastoma

• Big brother to osteoid osteoma, pain NOT relieved by NSAIDS.

• 2-6cm in size.• Posterior elements of spine most common

location.• Histology: woven bone with distinct

borders.

Page 27: Oncology oite-review-2012

Osteosarcoma

• Most common primary sarcoma of bone• Affects young patients• Associated with retinoblastoma• Most commonly presents in the 2nd and 3rd

decade of life (also in older adults with Pagets)

• Best predictor of survival is the stage of the disease.

• Most common presentation is Stage IIB

Page 28: Oncology oite-review-2012

Other Bone-Forming Conditions

Page 29: Oncology oite-review-2012

Other Bone-Forming Conditions

• Blastic Metastases: permeative lesion with infrequent soft tissue extension– 30/60/90 rule

renal/breast/prostate• Paget’s: early lyticlate

blastic (coarse trabeculae and bony enlargement—mosaic pattern)– OITE Pearls:

• Assoc w/Paramyxovirus• increased cranial

diameter (hats don’t fit)• deafness (2/2 nerve

compression)

Page 30: Oncology oite-review-2012

Fibrous Dysplasia• Peak age of diagnosis is 5 - 20

years – Two thirds of patients with

polyostostic disease are symptomatic before the age of 10

– With monostotic disease, patients as old as 20 - 30 years are asymptomatic

• ~70-80% are monostotic – This form most frequently occurs in

the rib (28%), femur (23%), tibia or craniofacial bones (10-25%), humerus, and vertebrae, in decreasing order of frequency

• ~20-30% of fibrous dysplasias are polyostotic– This form more frequently involves

the skull and facial bones, pelvis, spine, and shoulder girdle

– The sites of involvement are the femur (91%), tibia (81%), pelvis (78%), ribs, skull and facial bones (50%), upper extremities, lumbar spine, clavicle, and cervical spine, in decreasing order of frequency

Page 31: Oncology oite-review-2012

Fibrous Dysplasia

• Xrays: lucent lesion in the diaphysis or metaphysis, with endosteal scalloping and with or without bone expansion and the absence of periosteal reaction– Usually, the matrix is

smooth and relatively homogeneous; classically, described as “ground-glass” appearance

– Classic “Shepherd’s crook” deformity

Page 32: Oncology oite-review-2012

Fibrous Dysplasia• Histology: irregular foci of woven bone

arising from a cellular fibrous blue background– irregular trabeculae have been

described as “Chinese letters” or “alphabet soup”

• Rx: Observation /benign/usually disappears– If >75% cortex or

painfulcurettage/grafting– rigid, intramedullary fixation with the

strongest possible device (a steel or titanium cephalomedullary nail) is the best method for treatment of proximal femoral lesions

• McCune-Albright Syndrome: polyostotic, café au lait spots, precocious puberty, hyperthyroid Mutation is alpha subunit of a G protein

Page 33: Oncology oite-review-2012

Myositis Ossificans

• Juxtaposed to bone, and not connected to cortex

• Can be confused with synovial cell sarcoma

• Can be confused with parosteal OS• MO ossifies from periphery inward

Page 34: Oncology oite-review-2012

Bone-Forming Lesions Histology Summary

• Reactive lesion: woven bone with osteoblastic rimming and a benign spindle cell stroma (frx callus, myositis ossificans)

• Benign or low-grade bone forming neoplasm: woven bone bone with no osteoblastic rimming and bland-appearing spindle cell sroma (osteoid osteoma, osteoblastoma, fibrous dysplasia)

• Malignant bone tumor: woven bone with no osteoblastic rimming associated with malignant spindle cell stroma (osteosarcoma)

Page 35: Oncology oite-review-2012

Bone-Forming Lesions OITE questions

Page 36: Oncology oite-review-2012

85. A 15 yo cross country runner reports shin pain that is present during running, at rest, and at night. The pain is relieved by ibuprofen. Radiographs, a bone scan, and a CT scan are shown in Figures 32a through 32c. What is the most likely diagnosis?

1. Cortical desmoid

2. Osteoblastoma

3. Stress fracture

4. Enchondroma

5. Osteoid osteoma

OITE QUESTION

Page 37: Oncology oite-review-2012

PINK = BONE

Page 38: Oncology oite-review-2012

85. A 15 yo cross country runner reports shin pain that is present during running, at rest, and at night. The pain is relieved by ibuprofen. Radiographs, a bone scan, and a CT scan are shown in Figures 32a through 32c. What is the most likely diagnosis?

1. Cortical desmoid

2. Osteoblastoma

3. Stress fracture

4. Enchondroma

5. Osteoid osteoma

OITE QUESTION

Page 39: Oncology oite-review-2012

OITE QUESTION

Osteosarcoma most commonly develops in which of the following locations?

1- Pelvis2- Distal humerus3- Proximal tibia4- Proximal femur5- Proximal humerus

Page 40: Oncology oite-review-2012

OITE QUESTION

> > >

OCCURS AT MOST METABOLICALLY ACTIVE SITES

Page 41: Oncology oite-review-2012

OITE QUESTION

Osteosarcoma most commonly develops in which of the following locations?

1- Pelvis2- Distal humerus3- Proximal tibia4- Proximal femur5- Proximal humerus

Preferred Response: 3Recommended Reading(s):Menendez LR (ed): Orthopaedic Knowledge Update: Musculoskeletal Tumors. Rosemont,IL, American Academy of Orthopaedic Surgeons, 2002, pp 175-186.Mankin HJ, Hornicek FJ, Rosenberg AE, et al: Survival data for 648 patients withosteosarcoma treated at one institution. Clin Orthop Relat Res 2004;429:286-291

Page 42: Oncology oite-review-2012

OITE QUESTION

A 9-year-old boy is seen for bilateral thigh pain. He has a history of precocious puberty. Examination reveals multiple café-au-lait spots. AP pelvis and frog lateral hip radiographs are shown in Figures 21a and 21b. His condition is linked to an abnormality in1- G protein function.2- osteoclastic function.3- vitamin D metabolism.4- sulfate transporter gene.5- type I collagen formation.

Page 43: Oncology oite-review-2012

OITE QUESTION

A 9-year-old boy is seen for bilateral thigh pain. He has a history of precocious puberty. Examination reveals multiple café-au-lait spots. AP pelvis and frog lateral hip radiographs are shown in Figures 21a and 21b. His condition is linked to an abnormality in1- G protein function.2- osteoclastic function.3- vitamin D metabolism.4- sulfate transporter gene.5- type I collagen formation. Recommended Reading(s):

DiCaprio MR, Enneking WF: Fibrous dysplasia: Pathophysiology, evaluation, andtreatment. J Bone Joint Surg Am 2005;87:1848-1864.Parekh SG, Donthineni-Rao R, Ricchetti E, et al: Fibrous dysplasia. J Am Acad OrthopSurg 2004;12:305-313.

Page 44: Oncology oite-review-2012
Page 45: Oncology oite-review-2012

OITE QUESTION

Which of the following diseases of bone (when nonmetastatic at diagnosis) carries the worst prognosis for 5-year survival)?

• 1- Lymphoma• 2- Osteosarcoma• 3- Ewing’s sarcoma• 4- Paget’s sarcoma• 5- Conventional chondrosarcoma

Page 46: Oncology oite-review-2012

OITE QUESTION

Which of the following diseases of bone (when nonmetastatic at diagnosis) carries the worst prognosis for 5-year survival)?

• 1- Lymphoma• 2- Osteosarcoma• 3- Ewing’s sarcoma• 4- Paget’s sarcoma• 5- Conventional chondrosarcoma

Page 47: Oncology oite-review-2012

Cartilage-forming Lesions

Page 48: Oncology oite-review-2012

Radiographic Differential Diagnosis

Bone Forming Tumors Cartilage Forming Tumors "Third List"

Osteoid Osteoma Osteochondroma InfectionOsteoblastoma Chondromyxoid fibroma MetastasesOsteosarcoma Chondroblastoma Round cell tumorsBlastic Metastases Enchondroma Fibrous DysplasiaPaget's Disease Chondrosarcoma Non-Ossifying Fibroma(Fibrous Dysplasia) Simple Bone Cyst

Aneurysmal Bone CystHistiocytosisGiant cell tumorMetabolic condition

Page 49: Oncology oite-review-2012

Cartilage-Producing Lesions

• Enchondroma• Periosteal Chondroma• Osteochondroma• Multiple Hereditary Osteochondroma• Chondromyxoid Fibroma• Chondrosarcoma• Clear Cell Chondrosarcoma• Differentiated Chondrosarcoma

Page 50: Oncology oite-review-2012

Cartilage Forming Tumors

• Normal Cartilage: sparsely cellular, one cell per lacuna, one pyknotic nucleus per cell, well formed matrix

• 3 tumor patterns:– Benign (enchondroma)

merging into low grade which merges into intermediate then high grade

– Chondroblastoma• Cobblestone

chondroblasts and intervening chicken wire calcification

– Chondromyxoid fibroma• Benign spindle cell lesion

with some areas of immature cartilage

Page 51: Oncology oite-review-2012

Enchondroma

• Benign, metaphyseal, 60% are in the hand• Most common primary lesion in the hand• Imaging- stippled rings and arches, “popcorn”

calcifications, “blue balls” of cartilage• Compared to “smoke up the chimney” for

bone infarct• Ollier’s disease- multiple lesions/ 30%

malignant transformation• Mafucci disease- multiple bone lesions and

hemangiomas- 100% malignant

Page 52: Oncology oite-review-2012

Enchondroma• peak incidence 20-40 yo

(Ollier’s 0-10 yo) • usually found in the short

tubular bones of the hands and feet – most common primary

tumor in the hand and is normally found in the diaphysis

• NOT common in axial skeleton (if so, think chondrosarcoma)

• Xrays: lucent area in medullary canal (may see endosteal scalloping) with variable mineralization (rings, stipples, arcs)”popcorn”

Page 53: Oncology oite-review-2012

Enchondroma• Multiple enchondromas may occur in 3

distinct disorders: – Ollier disease: nonhereditary disorder

characterized by multiple enchondromas with a predilection for unilateral distribution (30% develop low grade chondrosarcoma)

– Maffucci syndrome: nonhereditary, less common than Ollier disease (100% develop chondrosarcoma)• +multiple hemangiomas• +other malignancies – follow with

PET scan– Metachondromatosis: multiple

enchondromas and osteochondromas• inherited by autosomal dominant

transmission

Page 54: Oncology oite-review-2012

Enchondroma

Page 55: Oncology oite-review-2012

Enchondroma• Histology: mineralized

hyaline cartilage– May see bone encased

lobules separated by marrow

• Rx: solitary painless enchondromas may be observed– Painful or worrisome lesions

should be treated with biopsy followed by intralesional resection/curettage

– Large defects can be filled with bone graft

– All specimens must be analyzed carefully for malignancy.

Page 56: Oncology oite-review-2012

Periosteal Chondroma

• Rare• Occurs in 10-20 year olds• Presents under the periosteum• 50% at the proximal humerus

Page 57: Oncology oite-review-2012

Osteochondroma

• 35% of all benign lesions, very common• Does not grow in adulthood• Key feature is continuity with the medullary

canal and with the cortex• Sessile, broad based and points away

from the joint• Cartilage cap is less than 2cm, if bigger-

be concerned for chondrosarcoma transformation 1%.

Page 58: Oncology oite-review-2012

Osteochondroma

• Histology: endochondral ossification on the basal surface of hyaline cartilage so it resembles a normal growth plate with rows of chondrocytes– cartilage more

disorganized than normal, has binucleate chondrocytes in lacunae, and is covered with a thin layer of periosteum

• Rx: Asymptomatic = Observation

Page 59: Oncology oite-review-2012

MHE- Multiple Hereditary Exostosis

• A subset of osteochondromas• Most frequent lesion to have secondary

chondrosarcomas, especially if they grow after skeletal maturity.

• Autosomal Dominant• EXT1, EXT2, EXT3

Page 60: Oncology oite-review-2012

Osteochondroma• most common benign bone tumor• most commonly present in the

second decade of life• grow until skeletal maturity and then

stop once the growth plates fuse• most commonly found around the

knee and the proximal humerus• Can be pedunculated or sessile• Hallmarks: Grow away from the

physis and have medullary continuity

• Secondary sarcomatous degeneration should be suspected if grows after puberty or has cartilage cap >3 cm in adulthood

• OITE pearl: Osteochondromatosis (HMOCE - Hereditary Multiple Osteo- Cartilagenous Exostoses)– AD Ch8 (EXT1), Ch11 (EXT2)– risk of malignant transformation

to chondrosarcoma may be 25-30% compared to approximately 1% for a solitary osteochondromas

Page 61: Oncology oite-review-2012

Patients with multiple hereditary osteochondromas frequently have loss of forearm rotation and ulnar shortening. What procedure is most likely to improve forearm rotation in thesepatients?1- Tendon transfer2- Ulnar lengthening3- Radial head excision4- Distal ulnar resection5- Simple excision of the osteochondroma

OITE QUESTION

Page 62: Oncology oite-review-2012

OITE QUESTION

Page 63: Oncology oite-review-2012

Shin EK, Jones NF, Lawrence JF: Treatment of multiple hereditary osteochondromas of theforearm in children: A study of surgical procedures. J Bone Joint Surg Br 2006;88:255-260.Akita S, Murase T, Yonenobu K, et al: Long-term results of surgery for forearm deformitiesin patients with multiple cartilaginous exostoses. J Bone Joint Surg Am 2007;89:1993-1999.

Patients with multiple hereditary osteochondromas frequently have loss of forearm rotation and ulnar shortening. What procedure is most likely to improve forearm rotation in thesepatients?1- Tendon transfer2- Ulnar lengthening3- Radial head excision4- Distal ulnar resection5- Simple excision of the osteochondroma

OITE QUESTION

Page 64: Oncology oite-review-2012

Chondromyxoid Fibroma

• Occurs in 20-30 year olds in proximal tibia or foot, metaphysis.

• Eccentric, well-demarcated lesion, 25% recur

• Treatment: Curretage and bone graft• Histology: stellate appearing cells, “soap

bubbles” or snot

Page 65: Oncology oite-review-2012

Chondromyxoid fibroma

• Extremely rare• Presents 2nd-3rd decades• found most often in the

metaphysis around the knee in the proximal tibia, proximal fibula, or distal femur

• Xray: eccentrically placed Iytic lesion with well defined margins in the metaphysis of the lower extremity

• Histology: chondrocytes in myxoid matrix separated by fibrous bands

• Rx: Extended curettage/Graft ~10% recurrence– En bloc excision as

complete cure

Page 66: Oncology oite-review-2012

Chondroblastoma

• Epiphyseal lesion of the young about the knee, proximal humerus, and proximal femur

• Painful-often abut the joint, and can cross the physis

• 2-5% metastasis to the lungs• Calcaneus lesion = Codman’s tumor• Histology: Chicken-wire calcifications;

cobblestone appearance• Treatment: Intralesional curretage and bone

graft

Page 67: Oncology oite-review-2012

Chondroblastoma• ~90% occur in those aged 5-25

years• Most common epiphyseal lesion

in children behind infection• typically occurs in the epiphysis

of a long bone• most common site is the lower

extremity (72% of cases), and 50% of tumors occur around the knee – The femur is involved in

33% of cases; the humerus, in 20%; and the tibia, in 18%

• symptoms are nonspecific and include joint pain, tenderness, swelling– Joint effusion occurs in

approximately 30% of patients

– Get an xray before you tap a joint!

Page 68: Oncology oite-review-2012

Chondroblastoma

• Xray: radiolucent with well defined margins– fine calcifications, either

punctate or in rings, may be visible

• Histology: densely packed polygonal Blue chondroblasts with “Fried Egg” appearance and “chicken wire” calcification

• Rx: biopsy and curettage with +/- adjuvant liquid nitrogen or phenol, or a mechanical burr– ~20% recurrence rate

• 2% mets to lungs• Classic: painful lucent lesion in

the epiphysis of a childBLUE = Cartilage

Page 69: Oncology oite-review-2012
Page 70: Oncology oite-review-2012

Chondrosarcoma

• Treatment is surgery!• Occurs in patients >50 yrs old• Most common location is the pelvis, ribs, and

proximal femur• Axial and proximal skeletal lesions are more

aggressive• Histologic grade correlates with mets:• Grade 1- 5%• Grade 2-20%• Grade 3- 60%• Dedifferentiated- almost 0% survival

Page 71: Oncology oite-review-2012

Chondrosarcoma• 2nd most frequent primary

malignant tumor of bone, representing approximately 25% of all primary osseous neoplasms

• Usually occur in patients older than 40 years

• THINK axial skeleton: most commonly involve the pelvic bones, femur, humerus, ribs, scapula, sternum, or spine– In tubular bones, the

metaphysis is the most common site of origin

• most common symptom at presentation is pain, often present for months and typically dull in character– may be worse at night– average duration of symptoms

prior to presentation is 1-2 years

Page 72: Oncology oite-review-2012

Chondrosarcoma• Xrays: fusiform, lucent defect

with scalloping of the inner cortex and periosteal reaction – Extension into the soft tissue

may be present as well as punctate or stippled calcification of the cartilage matrix

• CT helpful in defining the integrity of the cortex and distribution of calcification

• MRI invaluable in surgical planning as it demonstrates the intraosseus and soft tissue involvement of the tumor– also helpful in evaluating

possible malignant degeneration of osteochondromas by allowing accurate measurements of the cartilage cap which should be less than 2 cm thick

Page 73: Oncology oite-review-2012

Dedifferentiated Chondrosarcoma

• Most Malignant cartilage lesion• 50% have a pathologic fracture due to

tumor’s aggressive and very destructive nature.

• Treatment- aggressive- chemo, surgery, chemo

• 13% 5 year survival rate

Page 74: Oncology oite-review-2012

Chondrosarcoma• Categorized by grade:

– Grade 1 (Low grade)-low cellularity, abundant ECM, small dark nuclei (5% mets)

– Grade 2 (Low grade)-increased cellularity [50% chondrosarc] (20% mets)

– Grade 3 (High grade/ most aggressive)-high cellularity, mitotic figures, etc. (70% mets)

• 75% are Low grade• Most classified as

conventional, but other subgroups are clear cell, myxoid, mesenchymal, and dedifferentiated

Page 75: Oncology oite-review-2012

Chondrosarcoma

• Centralin Medullary cavity• Surface

– Seen in 20-40 yo group most often in pelvis/prox femur

– “explosive” lesion of stippling that is NOT circumscribed

• Dedifferentiated– mix of low grade chondrosarcoma

and high grade spindle cell sarcoma where the spindle cells are no longer identifiable as having a cartilage origin

– biphasic quality is on x-ray with areas of endosteal scalloping/cortical thickening contrasted with areas of cortical destruction

– Most malignant—5 yr surv 10%

• Mesenchymalpredilection for the spine, ribs and jaw and it presents in the third decade rare variant with a bimorphic histologic picture of low grade cartilaginous cells and hypercellular small, uniform, and undifferentiated cells that resemble Ewing's sarcoma

• Clear cell

Page 76: Oncology oite-review-2012

Chondrosarcoma

• Clear cell– adult variant of

chondroblastoma– found in the epiphysis of the

femur and humerus• Classic—head of femur

– Histology: clear cells with vacuolated cytoplasm

• matrix has significantly calcified trabeculae and giant cells

– rare, low-grade tumor with an improved prognosis over other chondrosarcomas

• OITE pearls: Epiphyseal lesions = chondroblastoma (pedi), giant cell and clear cell (adult)

Page 77: Oncology oite-review-2012

Chondrosarcoma• Treatment =Wide surgical

excision • XRT and Chemo resistant• Biopsies must be planned

with future tumor excision in mind

• Patients with adequately resected low grade chondrosarcomas have an excellent survival rate

• The survival of patients with high grade tumors depends on the location, size and stage of the tumor

Page 78: Oncology oite-review-2012

Cartilage-Forming LesionsOITE Questions

Page 79: Oncology oite-review-2012

OITE QUESTION

A 12-year-old patient has an epiphyseal lesion. What is the most likely diagnosis?

1- Giant cell tumor2- Aneurysmal bone cyst3- Eosinophilic granuloma4- Dysplasia epiphysealis hemimelica5- Chondroblastoma

Page 80: Oncology oite-review-2012
Page 81: Oncology oite-review-2012

OITE QUESTION

A 12-year-old patient has an epiphyseal lesion. What is the most likely diagnosis?

1- Giant cell tumor2- Aneurysmal bone cyst3- Eosinophilic granuloma4- Dysplasia epiphysealis hemimelica5- Chondroblastoma

Page 82: Oncology oite-review-2012

OITE QUESTION

• Which of the following conditions shown in these pictures has the highest rate of malignant change?  1. Ollier’s disease 2. Enchondromatosis 3. Maffucci’s syndrome 4. Multiple exostoses 5. Solitary osteochondroma

Page 83: Oncology oite-review-2012
Page 84: Oncology oite-review-2012

OITE QUESTION

• Which of the following conditions shown in these pictures A-D has the highest rate of malignant change?  1. Ollier’s disease 2. Enchondromatosis 3. Maffucci’s syndrome 4. Multiple exostoses 5. Solitary osteochondroma

Page 85: Oncology oite-review-2012

23. An 80-year-old woman reports the sudden development of pain in the left distal thigh. She denies any history of trauma. Figures 9a through 9d show radiographs, a bone scan, and a biopsy specimen. What is the likely diagnosis?

1- Enchondroma

2- Osteosarcoma

3- Dedifferentiated chondrosarcoma

4- Metastatic breast carcinoma

5- Paget’s disease

OITE QUESTION

Page 86: Oncology oite-review-2012

OITE 2006 – Ortho Diseases

Page 87: Oncology oite-review-2012

23. An 80-year-old woman reports the sudden development of pain in the left distal thigh. She denies any history of trauma. Figures 9a through 9d show radiographs, a bone scan, and a biopsy specimen. What is the likely diagnosis?

1- Enchondroma

2- Osteosarcoma

3- Dedifferentiated chondrosarcoma

4- Metastatic breast carcinoma

5- Paget’s disease

OITE QUESTION

Page 88: Oncology oite-review-2012

Radiographic Differential Diagnosis

Bone Forming Tumors Cartilage Forming Tumors "Third List"

Osteoid Osteoma Osteochondroma InfectionOsteoblastoma Chondromyxoid fibroma MetastasesOsteosarcoma Chondroblastoma Round cell tumorsBlastic Metastases Enchondroma Fibrous DysplasiaPaget's Disease Chondrosarcoma Non-Ossifying Fibroma(Fibrous Dysplasia) Simple Bone Cyst

Aneurysmal Bone CystHistiocytosisGiant cell tumorMetabolic condition

Page 89: Oncology oite-review-2012

“Third List” Tumors

• Fibrous Dysplasia (COVERED IN BONE PRODUCING LESIONS)

• Non-ossifying fibroma• Osteofibrous dysplasia• Aneurysmal Bone cyst• Unicameral Bone cyst• Myeloma• Lymphoma• Ewing’s Sarcoma• Neuroblastoma• Histiocytosis• Giant Cell Tumor

Page 90: Oncology oite-review-2012

Fibrous Dysplasia• Developmental abnormality that can be

mono- or polyostotic• Café au lait spots- Coast of Maine (more

rugged)• Coast of California (smoother) —

neurofibromatosis• Associated with Gs-alpha Protein• Common location is proximal femur-

shepard’s crook.• X-ray has ground glass appearance

Page 91: Oncology oite-review-2012

Fibrous Dysplasia

• Peak age of diagnosis is 5 to 20 years

• sites of involvement are the femur (91%), tibia (81%), pelvis (78%), ribs, skull and facial bones (50%), upper extremities, lumbar spine, clavicle, and cervical spine, in decreasing order of frequency

• Xrays: lucent lesion in the diaphysis or metaphysis, with endosteal scalloping and with or without bone expansion and the absence of periosteal reaction– “ground glass” appearance

• Rx: Observation b/benign/usually disappears– If >75% cortex or

painfulcurettage/grafting

Page 92: Oncology oite-review-2012

LEARN FROM THE MISTAKES OF OTHERS IN FIBROUS DYSPLASIA

• Do NOT use autograft• Do NOT use cancellous

allograft• Do NOT try to remove all of the

disease• Do NOT allow the deformity to

be come severe• Do NOT use plates and screws

if possible• Do NOT expect a single

operation to solve the problemStanton, Robert P. J Bone & Mineral Res, 2006; 21:p105-109.

Page 93: Oncology oite-review-2012

Osteofibrous Dysplasia• A.k.a ossifying fibroma• most common site in adults

is the mandible• most common site in

children is the tibia, followed by other long bones

• occurs during the first decade of life and presents clinically as a painless, enlarging mass

• XRAY: multiple eccentric, well-circumscribed lucent lesions in anterior cortex of tibia

• Histology: irregular spicules of trabecular bone lined by osteoblasts in fibroblastic background– stains positive for

cytokeratin

Page 94: Oncology oite-review-2012

Osteofibrous Dysplasia• Treatment

– Older patients : • conservative management • Curettage

– Pts w/open growth plates• observe• if bowing of the tibia becomes an issue, bracing is

recommended

Page 95: Oncology oite-review-2012

Fibrous Dysplasia vs Osteofibrous Dysplasia

Fibrous Dysplasia Osteofibrous Dysplasia – less well circumscribed bone surrounded by osteoblasts

Page 96: Oncology oite-review-2012

Non-ossifying fibroma (fibrous cortical defect)

• Xrays: eccentric, multi-loculated sub-cortical lesions with a central lucency and a scalloped sclerotic margin

• Histology: highly cellular (including giant cells & lipophages) with storiform fibrogenic areas

• Rx: observation (most regress spontaneously)– only definite indication to

treat is a pathologic fracture

• *Pearl - Jaffe-Campanacci syndrome: multiple NOF, cafe-au-lait spots, mental retardation, hypogonadism, ocular/cardio abnormalities.

Page 97: Oncology oite-review-2012

Non-ossifying fibroma (fibrous cortical defect)

• found mostly in children with 75% occurring in the second decade

• non-neoplastic process that occurs in the juxta-epiphyseal region of the long bones– ~90% involve the tubular

long bones– Common sites include the

femur (most commonly the distal femoral metaphysis 38%), the proximal and distal tibia (43%)

– most lesions occur around the knee

• Classic scenario: child who has a minor sports injury and a plain xray shows lesion 

Page 98: Oncology oite-review-2012

Round cell tumorsmyeloma, lymphoma, metastatic dz, small cell

osteosarcoma, Ewing’s sarcoma, infection

• <5 yo then think Neuroblastoma• 10-40 yo think of the destructive group

(OSA, Ewing’s, infection)• >40 yo think Metastatic Dz, Myeloma,

Lymphoma– Note that infection can be seen in any age

group

Page 99: Oncology oite-review-2012

Ewing’s Sarcoma

• Malignant round cell sarcoma affecting young patients 2-25.

• Commonly Diaphyseal• <5 yo, consider leukemia or metastatic

neuroblastoma.• >30 yo, consider metastatic carcinoma or

lymphoma• Third most common primary sarcoma of bone

1. osteosarcoma 2. Chondrosarcoma 3. Ewings

Page 100: Oncology oite-review-2012

Ewing’s Sarcoma• A type of primitive peripheral

neuroectodermal tumor (PNET)• Most common in the 1st and

2nd decades• Preferentially affects whites

more than blacks and Asians• 3:2 male to female ratio• Found in the lower extremity

more than the upper extremity, but any long tubular bone may be affected– most common sites are the

metaphysis and diaphysis of the femur followed by the tibia and humerus

• Clinical presentation includes pain and swelling of weeks or months duration– Erythema and warmth of

the local area are sometimes seen

Osteomyelitis is often the initial diagnosis based on fevers, leukocytosis, anemia and an increased ESR

Page 101: Oncology oite-review-2012

Ewing’s Sarcoma• Xrays: often a central lytic lesion

associated with a lamellated or "onion skin" periosteal reaction (caused by and splitting and thickening of the cortex by tumor cells)

– "onion-skin" appearance is often followed with a "moth-eaten" or mottled appearance and extension into soft tissue

Page 102: Oncology oite-review-2012

Ewing’s Sarcoma

Imaging:

• CT is helpful in defining bone destruction

• MRI is essential to elucidate the soft tissue involvement– T1-weighted images low intensity compared to the normal

high intensity of bone marrow. On T2 -weighted images the tumor is hyper intense compared to muscle

• Increased uptake on bone scan

Page 103: Oncology oite-review-2012

Ewing’s Sarcoma• Histology: densely packed uniform small

round blue cells in sheets (+CD99)– Glycogen is present within the cells

causing (+) reaction to periodic acid-schiff (PAS) stain

– Most are positive with HBA-71 or 0-13 stain which is an antibody to the product of myc

– distinguished from metastatic neuroblastoma by reticulin stain and urine vanillyl mandelic acid and homovanillic acid

– Rhabdomyosarcoma is ruled out if the specimen stains negatively with desmin, myoglobin and actin stains

– neural origin is supported by electron microscope findings of pseudorosettes

– This is further supported by the common finding in Ewing's sarcoma and primitive neuroectodermal tumors of choline acetyltransferase and the translocation t(11:22)

Page 104: Oncology oite-review-2012

Ewing’s Sarcoma• Treatment

– includes surgery, radiation and multi-drug chemotherapy

– XRT or chemo used preoperatively– Adjuvant chemotherapy follows surgery and

decreases recurrences

• Poor prognostic signs:– increased age – increased ESR at presentation– leukocytosis at presentation

• Distinguish from metastatic neuroblastoma by keratin stain

Page 105: Oncology oite-review-2012

Neuroblastoma• The second most common solid

malignancy of childhood (after brain tumors)– responsible for ~ 10% of all

pediatric neoplasms (and up to 1/2 of malignancies diagnosed in infancy)

• predominantly a tumor of early childhood (2/3 dx’d when < 5 yo)

• often begins in the nerve tissue of the adrenal glands

• Xrays: permeative lesion with medullary bone replacement and variable lysis

• Histology: densely cellular sheets of small cells with dark nuclei and scant cytoplasm that have an overall primitive appearance– Rosettes (Homer Wright

pseudorosettes) are often found - in them, tumor cells are arranged in circles around a central mass of neuropil

• The majority (about 90%) produce catecholamines which are an important diagnostic feature– elevated blood levels of

catecholamines and elevated urine levels of metabolites such as vanillymandelic acid VMA and homovanillic acid HVA are present

• Metastatic dz = dismal <5% 3 year survival and need to be treated with surgery, chemotherapy, and radiation

Page 106: Oncology oite-review-2012

Osteomyelitis• More common in young • The most common organism in all

pts = Staph Remember Neonates-group B strep/E coli, Sickle-Salmonella, shoe Puncture-Pseudomonas, after Surg-Staph epi

• Hematogenous osteo occurs most commonly in the distal femur, proximal tibia, proximal femur and proximal humerus

• May be caused by direct innoculation secondary to trauma or surgery– 5% of Open frx and 5% of acute

OM become chronic OM– Biopsy chronic cases—1%

chronic sinuses get SCC (Marjolin’s ulcer)

Page 107: Oncology oite-review-2012

Osteomyelitis• Xrays: Radiolucent and

radiodense lesions usually in metaphysis (may cross growth plate into epiphysis)– Chronicmottled bone,

periosteal rxn, bony deformation• “Brodie’s Abscess” =

Subacute OM—well defined lytic defect and mottled sclerosis of adjacent metaphysis

• Sequestrum (dead cortical bone) and Involucrum (new cortical bone b/c periosteal bld supply external)

Page 108: Oncology oite-review-2012

Osteomyelitis• On MRI scan, T1 weighted images

demonstrate infection as a low signal with ill defined margins. T2 images show infection as a bright signal..

• Histology: 3 featuresmixed cell population, new capillaries and tissue separation (edema)

• Rx: Surgical sampling or needle biopsy is necessary for diagnosis– Infected hardware should be

removed if the bone is healed and stable

– Acute osteomyelitis is treated with irrigation and debridement as necessary,  followed by four to six weeks of antibiotics

– Chronic osteomyelitis is best treated with thorough debridement, antibiotics,  and local flap coverage if necessary

Page 109: Oncology oite-review-2012

Hematopoietic Bone Lesions

• Lymphoma• Multiple myeloma• Solitary plasmacytoma• Osteosclerotic myeloma

Page 110: Oncology oite-review-2012

Lymphoma

• Bony involvement of Hodgkin’s disease (HD) is between 9.2%-30.3% of patients

• Primary Bone Involvement (primary lymphoma of bone) <1% -- Osseous lesions are most commonly due to hematogenous dissemination in stage IV disease or due to direct invasion from an adjacent lymph node leading to focal sclerosis

Page 111: Oncology oite-review-2012

Lymphoma• Xrays: extensive lytic/blastic

changes with 25-50% pts having CORITCAL THICKENING (which is also seen in Chondrosarcoma, Pagets, and Chronic OM)

• presents in the 3rd or 4th decades or later

• The sites of predilection are long bones and primarily diaphyseal affecting the medulla of the bone

• Multifocal in older age group• Bone scan is useful and MRI is

very sensitive in the localization of subtle marrow disease– hypointense signal on T1

weighted spin echo sequences and high signal on T2 weighted gradient echo sequences

Page 112: Oncology oite-review-2012

Lymphoma• Histology: Bone marrow taken

over by lymphocytes of all shapes and sizes (usually large non-cleaved B-cell)

• Often, a diagnosis of “histologic exclusion”– Given lytic lesion in pt

>40yo on histologic slide see NO glands (mets), NO plasma cells (MM), NO cartilage (CS), and NO storiform pattern (MFH)

• Rx: Pts w/ one or multiple bone lesions usually respond well to combined modality treatment, including chemo and local XRT

• Surgery should be considered for lesions that present a risk of pathological fracture.

Page 113: Oncology oite-review-2012

Lymphoma

• Primary is usually Non-Hodgkins lymphoma

• Mottled, blastic bone lesion• VERY hot on bone scan• Histology- mixed round cell inflitrate• Treatment- radiation and chemotherapy• Mets, MM, and Lymphoma usually do not

require surgery (except for structural insufficiency)

Page 114: Oncology oite-review-2012

Multiple Myeloma

• Plasma cell malignancy in patients >50yo.• Plasma cells produce immunoglobulins• Bone destruction by RANKL• Bisphosphonates reduce the number of skeletal

events in patients with MM.• Numerous Punched out lesions• 30% of bone scans are cold: need to get skeletal

survey• Serum Protein Electrophoresis (SPEP)- M spike-

Immunoglobulin G (50%). Immunoglobulin A (25%)• Treatment is chemo and radiation

Page 115: Oncology oite-review-2012

Multiple Myeloma• Most common primary malignant

tumor• Median age: 68 y for men/70 y for

women• Involves spine, skull, ribs, sternum

and pelvis but may affect any bone with hematopoietic red marrow

• Normocytic, normochromic anemia secondary to marrow failure and an increased ESR

• SPEP—may see “goal post sign” – Specifically, SPEP>75% sens in

dx while UPEP adds ~20%• Single greatest prognostic factor is

Renal Function• Median survival 2 yrs

Page 116: Oncology oite-review-2012

Multiple Myeloma• Xrays: diffuse osteopenia with

multiple “punched out” radiolucent lesions w/no periosteal reaction

• Bone scan can fail to have increased uptake in 25% pts suggesting a skeletal survey should always be done

• Histology: Uniform population of monoclonal plasma cells (>30% BM)

• Rx: palliative chemo or BMT– Only patients with complete

remission experience any bony healing

– Bisphosphonates used to inhibit bone resorption

– Surgery & XRT 2-3 wks later for impending/path frx

Page 117: Oncology oite-review-2012

Plasmacytoma

• A local disease of MM• Treatment is radiation• Better prognosis than MM.

Page 118: Oncology oite-review-2012

Osteosclerotic Myeloma

• Poems syndrome• Associated with neuropathy

Page 119: Oncology oite-review-2012

Tumor Like Conditions of Bone

• Aneurysmal Bone Cyst• Unicameral Bone Cyst (Simple Bone Cyst)• Histiocytosis (Langerhans’ Cell

Histiocytosis• Fibrous Dysplasia

Page 120: Oncology oite-review-2012

Aneurysmal Bone Cyst

• Vertebrae and long bones are typical location

• Lytic and expansile lesion in patients less than 20 years old.

• Fluid-fluid lines on x-ray, better seen on MRI

• Will see RBC in histology• Treatment: Curretage and bone graft• 25% recur

Page 121: Oncology oite-review-2012

Aneurysmal Bone Cyst• found most commonly during

the second decade • most common location is the

metaphysis of the lower extremity long bones (>than the upper extremity)– vertebral bodies also may

be involved • 30% have identifiable pre-

existing lesion• translocation involving the

16q22 and 17p13 chromosomes has been identified in the solid variant and extraosseous forms of aneurysmal bone cyst

Page 122: Oncology oite-review-2012

Aneurysmal Bone Cyst• Present with PAIN (vs UBC

—fracture)• Xrays: purely lytic,

expansile, eccentric metaphyseal lesion that erodes cortex (“soap bubble”)– Fluid-fluid levels

• Histology: blood-filled spaces with NO endothelial lining

• Rx: biopsy to rule out telengiectatic OSA then extended curettage/grafting– Recurrence statistically

related to young age and open growth plates, and may be less likely following wide excision

Page 123: Oncology oite-review-2012

Aneurysmal Bone Cyst• Present with PAIN (vs UBC

—fracture)• Xrays: purely lytic,

expansile, eccentric metaphyseal lesion that erodes cortex

• Fluid-fluid levels on CT/MR

• Histology: blood-filled spaces with NO endothelial lining

• Rx: biopsy to rule out telengiectatic OSA then extended curettage/grafting– Recurrence statistically

related to young age and open growth plates, and may be less likely following wide excision

– White space and blood

Page 124: Oncology oite-review-2012

Aneurysmal Bone Cyst

• Common in second decade • Most common location is the

metaphysis of the lower extremity long bones (>than the upper extremity)– vertebral bodies also

may be involved • 30% have identifiable pre-

existing lesion• Translocation involving the

16q22 and 17p13 chromosomes has been identified in the solid variant and extraosseous forms of aneurysmal bone cyst

• Telangiectatic Osteosarcoma DDX 1/1000

Page 125: Oncology oite-review-2012

Unicameral Bone Cyst

• Also known as Simple Bone Cyst

• Most commonly found in children between 5-15 yo (85% pts < 20 yo)

• Most common site: prox humerus (50%) followed by prox femur (25%)

• Xrays: radiolucent, centered lesion with thinned cortices and metaphyseal expansion to width of physis– “fallen leaf” sign = fx

Page 126: Oncology oite-review-2012

Unicameral Bone Cyst• Usually asymptomatic and

only present when pathologic frx occurs

• Histology: thin fibrous membrane lines lumen

• Rx: aspiration confirmation then steroid injection(s)– If pathologic frxlet heal

first before injecting– In lower ext, may consider

open curettage/grafting; however, recurrence rates reported up to 40%

• MRI all cysts to diagnose fluid inside

Page 127: Oncology oite-review-2012

How to tell UBC from ABC

• UBC– Minimal pain unless frx– No h/o trauma unless frx– Fluid-filled cavity– Slightly expansile– Xray with few loculations– MRI with minimal fluid-fluid

levels– (+)membrane lining– White space

on path

• ABC– Pain even without frx– May have had direct blow– Blood-filed cavity– Very expansile– Xray with many loculations– MRI with prominent fluid-fluid

levels– No membrane lining

(mesenchymal tissue lining)– White space

& blood on path

Page 128: Oncology oite-review-2012

Histiocytosis (Langerhans’ cell Histiocytosis

• Continum- Eosinophilic granuloma—Hand-Schuller-Christian Disease—Letterer-Siwe disease.

Page 129: Oncology oite-review-2012

Eosinophilic Granuloma

• Affects multiple bones and occuring in the youth.

• Punched out lesion on x-ray• Vertebra plana• Treament low dose radiation, bone graft

with curretage, steroid injection, or observation

Page 130: Oncology oite-review-2012

Eosinophilic Granuloma• part of a spectrum of Langerhan's

cell histiocytosis, formerly known as histiocytosis X

• localized lesion in bone or lung and occurs most commonly in children aged 5 to 10

• makes up 60-80% of all cases of Langerhan's cell histiocytosis

• uncommon in blacks• found in the skull, mandible, spine

and long bones• In spine, may cause vertebra plana

(Calve’s disease)• male to female ratio is two to one• Letterer-Siwe disease is a fulminant

systemic disease that comprises 10% of Langerhan's cell histiocytosis, occurs in children under 3 years old and is rapidly fatal

• Hand-Schuller-Christian disease (HSC) is a chronic disseminated form of Langerhan' s histiocytosis– occurs in older patients– triad of HSC is diabetes

insipidus, exopthalmos and skull lesions

Page 131: Oncology oite-review-2012

Eosinophilic Granuloma• XRAY: well-circumscribed, central

radiolucent lesion in metaphysis or diaphysis of long bone with radiodense rim– “hole within a hole” pattern

• Histology: sheets of Langerhan's cells, multinucleated giant cells, and eosinophils– LC as Eosinophilic histiocytes with

coffee bean shape nuclei – identifiable under EM: racket shaped

cytoplasmic inclusion bodies called Birbeck's granules

– Confirmed by CD1a stain• Rx: With localized disease, often a

biopsy alone is enough to incite healing– Other treatment modalities of EG

include curettage, excision, steroid injection, radiation and observation

– Chemotherapy is recommended for systemic disease

Page 132: Oncology oite-review-2012

McCune-Albright syndrome

• Precocious puberty• Café au lait spots• Poly-ostotic fibrous dysplasia• Chinese letters histology• Treatment- observe vs. internal fixation• “Fibrous dysplasia heals with fibrous

dysplasia”

Page 133: Oncology oite-review-2012

Malignant Fibrous Histiocytoma• Most frequent soft tissue tumor in

adults• Highest incidence in 5th decade• Found in the extremities 75% of

the time (50% lower ext)– In order: Distal femur, prox tibia,

prox femur, prox humerus• #1 sarcoma to complicate a pre-

existing condition (Paget’s, bone infarct, radiation, AVN, surgery, NOF)– 20% Secondary to another

condition – These lesions are ususally more

aggressive (presenting as Stage IIb sarcoma)

Page 134: Oncology oite-review-2012

Malignant Fibrous Histiocytoma• Clinically, p/w rapidly enlarging

mass• XRAY: extensive, permeative

lesion, usually central in metaphyseal or diaphyseal region– “mottled bone”– Associated soft tissue mass

• CT scan is helpful in determining cortical erosion

• MRI is a must: intermediate T1/high T2

• Increased uptake on Bone scan

Page 135: Oncology oite-review-2012

Malignant Fibrous Histiocytoma

• Histology: Malignant multi-nucleated giant cells with “storiform” spindle cell fibrogenic areas (create “cartwheels”)– Stains (+) for CD68 and

Lysosome– 5 subtypes: pleomorphic-

storiform (most common 50-60%), myxoid, giant cell, inflammatory, angiomatoid

• Rx: wide resection and chemo

Page 136: Oncology oite-review-2012

Bone tumors of unknown origin

• Giant cell tumor of bone• Ewing’s Sarcoma (COVERED IN “ROUND CELL TUMORS”)

• Adamantinoma

Page 137: Oncology oite-review-2012

Giant Cell tumor of Bone

• Aggressive benign lesion in 20-40 yos, especially distal radius and proximal tibia.

• If ulnar side of distal radius, this osteosarcoma

• Imaging- well marginated lytic lesion, subchondral, eccentric, closed physis.

• Histology: multi-nucleated giant cells• Treatment- aggressive curretage with

bone graft, with a reccurence rate of 2-5%

Page 138: Oncology oite-review-2012

Giant Cell Tumor• XRAY: eccentric, expanding

zone of well-defined radiolucency at ends of bones (no reactive sclerosis)

• Histology: giant cells with nuclei appearing similar to those of the surrounding nuclei

• Rx: Intralesional excision by "extended" curettage is the treatment of choice– Curettage alone is associated with a

high recurrence rate, and this can be decreased with the addition of chemical cautery using phenol, multiple freeze-thaw cycles using liquid nitrogen, and treating the walls of the cavity with a high-speed rotary burr

– Curettage alone is thought to lead to recurrence in 50% of cases

– Recurrence after extended curettage is ~10%

– The tumor cavity may be filled with cement or bone graft, according to the surgeon's preference

Page 139: Oncology oite-review-2012

Giant Cell Tumor of Bone• occur most often during the

third decade• usually found in the long

bones, most often the distal femur, proximal tibia, and distal radius

• Whether tumor arises in the epiphysis or distal metaphysis is a matter of controversy, but giant cell tumors only occur after the epiphyseal plates have closed – A diagnosis of GCT in a

patient with open growth plates should be questioned

• Pts p/w progressive pain• 2% met to lungs (10-30% of

met pts die)– Therefore, Chest CT for

GCT

Page 140: Oncology oite-review-2012

Adamantinoma

• Tibia lesion in patients >20 years old, can involve tibia in up to 50%.

• Precursor of Osteofibrous dysplasia (Campanacci disease)

• Histology- fibrous tissue with epithelioid tissue

• Treatment- wide en bloc excision, consider intercalary tibia resection.

• 40% recur • Lung mets 25%

Page 141: Oncology oite-review-2012

Adamantinoma• extremely rare, low-grade

malignant tumor of epithelial origin

• 90% in the diaphysis of the tibia

• usually occurs in the 2nd to 5th decade of life

• 20% of cases have metastases late in the course of the disease

• May mimic osteofibrous dysplasia on xray

Page 142: Oncology oite-review-2012

Adamantinoma• XRAY: multiple eccentric, well-

circumscribed lucent defects with intervening sclerosis in the tibia (“soap bubble”)

• Histology: biphasic--blue epithelial nests of cells within fibrous background– Epithelial origin, +keratin

• Rx: wide surgical excision– insensitive to radiation and

may metastasize to lungs, lymph nodes and abdominal organs by both hematogenous and lymphatic routes

– Chemotherapy is not used

Page 143: Oncology oite-review-2012

Chordoma• rare malignant tumor that arises

from notochord remnants• account for 1-4% of all bone

tumors• occur in older adults with the

highest prevalence in the 5th to 7th decade

• the ratio of male to female is 2:1• due to their origin in the

notochord, chordomas occur in the mid-line of the axial skeleton– 50% occur in the

sacrococcygeal region and 33% occur at the base of the skull

– other rare sites include transverse processes of vertebrae and the paranasal sinuses

• p/w low back pain, bowel/bladder dysfunction, headaches

Page 144: Oncology oite-review-2012

Chordoma• XRAY: solitary mid-line lesion with

bony destruction– often have an accompanying

soft tissue mass and ~50% have focal calcifications

• CT and MRI scans help demonstrate the soft tissue component, calcifications and epidural extension

• Chordomas have reduced uptake on bone scan

• Histology: lobules of “physaliferous” cells in fibrous septa– Stain (+) for cytokeratin, S-100,

vimentin, and epithelial membrane antigen

• Rx: Wide surgical excision is desirable but rarely feasible based on the anatomic location of the tumor – sexual function and sphincter

control may be compromised after surgery

– Radiation is used if complete resection is impossible

– Chemotherapy can be used for late stage disease

• Death from Local Complications

Page 145: Oncology oite-review-2012

Testable facts s/p SacrectomyBowel and Bladder Function After Sacral Resection (Todd et al., CORR 4/2002)

• Resection Spared Level Normal Bowel Normal Bladder• Bilateral S2-S5 Both S1 0/10 (0%) 0/10 (0%)• Bilateral S3-S5 Both S2 2/5 (40%) 3/12 (25%)• Bilateral S4-S5 Both S3 4/4 (100%) 9/13 (69%)• Variable Unilateral S3 2/3 (67%) 3/5 (60%)• Unilateral S1-S5 Contralateral S1-S5 7/8 (87%) 8/9 (89%)

Page 146: Oncology oite-review-2012

“Other” tumors/lesions OITE Questions

Page 147: Oncology oite-review-2012

51. A 12 yo boy sustained a fracture in the distal ulna twice in the past year. He denies any major fall or injury. A radiograph and a biopsy speciman are shown in Figures 17a and 17b. What is the most likely diagnosis?

1. Chondroblastoma

2. Giant Cell Tumor

3. Unicameral bone cyst

4. Fibrous dysplasia

5. Nonossifying fibroma

OITE QUESTION

Page 148: Oncology oite-review-2012

Empty White Space

Page 149: Oncology oite-review-2012

51. A 12 yo boy sustained a fracture in the distal ulna twice in the past year. He denies any major fall or injury. A radiograph and a biopsy speciman are shown in Figures 17a and 17b. What is the most likely diagnosis?

1. Chondroblastoma

2. Giant Cell Tumor

3. Unicameral bone cyst

4. Fibrous dysplasia

5. Nonossifying fibroma

OITE QUESTION

Page 150: Oncology oite-review-2012

OITE QUESTION

A 35-year-old woman has increasing shoulder pain. A radiograph, sagittal T2-weighted MRI scan, and CT scan are shown in Figures 236a through 236c. A biopsy specimen is shown in Figure 236d. What is the most likely diagnosis?1- Infection2- Tuberculosis3- Giant cell tumor4- Aneurysmal bone cyst5- Telangiectatic osteosarcoma

Page 151: Oncology oite-review-2012

Blood and White Space

Page 152: Oncology oite-review-2012

OITE QUESTION

A 35-year-old woman has increasing shoulder pain. A radiograph, sagittal T2-weighted MRI scan, and CT scan are shown in Figures 236a through 236c. A biopsy specimen is shown in Figure 236d. What is the most likely diagnosis?1- Infection2- Tuberculosis3- Giant cell tumor4- Aneurysmal bone cyst5- Telangiectatic osteosarcoma

Preferred Response: 4Recommended Reading(s):Kaila R, Ropars M, Briggs TW, et al: Aneurysmal bone cyst of the pediatric shoulder girdle:A case series and literature review. J Pediatr Orthop B 2007;16:429-436.Schwartz HS (ed): Orthopaedic Knowledge Update: Musculoskeletal Tumors 2. Rosemont,IL, American Academy of Orthopaedic Surgeons, 2007, pp 87-102

Page 153: Oncology oite-review-2012

OITE QUESTION

An 8-year-old girl has had right elbow pain for the past 2 months. Radiographs, MRI scans, and a biopsy specimen are seen in Figures 250a through 250e. What is the most likely diagnosis?1- Osteomyelitis2- Chondroblastoma3- Simple bone cyst4- Eosinophilic granuloma5- Aneurysmal bone cyst

Page 154: Oncology oite-review-2012

Blood and White Space

Page 155: Oncology oite-review-2012

OITE QUESTION

An 8-year-old girl has had right elbow pain for the past 2 months. Radiographs, MRI scans, and a biopsy specimen are seen in Figures 250a through 250e. What is the most likely diagnosis?1- Osteomyelitis2- Chondroblastoma3- Simple bone cyst4- Eosinophilic granuloma5- Aneurysmal bone cyst

Page 156: Oncology oite-review-2012

242. A 55-year-old man has an isolated femoral fracture following a motor vehicle accident. A radiograph is shown in Figure 91. He is hemodynamically stable and his pain is adequately controlled. What is the next most apporpriate step in management?

1 – antegrade intramedullary nailing of femur2 – retrograde intramedullary nailing of the femur3 – placement of a distal femoral traction pin4 – wide resection5 – biospy and staging

OITE QUESTION

Page 157: Oncology oite-review-2012
Page 158: Oncology oite-review-2012

242. A 55-year-old man has an isolated femoral fracture following a motor vehicle accident. A radiograph is shown in Figure 91. He is hemodynamically stable and his pain is adequately controlled. What is the next most apporpriate step in management?

1 – antegrade intramedullary nailing of femur2 – retrograde intramedullary nailing of the femur3 – placement of a distal femoral traction pin4 – wide resection5 – biospy and staging

OITE QUESTION

Page 159: Oncology oite-review-2012

General Concepts review

Page 160: Oncology oite-review-2012

Staging review• Grade: Histology with aid of radiographic findings and clinical correlation

G 1: Low grade,   uniform cell type without atypia, few mitosesG 2: High grade, atypical nuclei, mitoses pronounced

• Site: T 1: Intracompartmental (Confined within limits of periosteum)      T 2: Extracompartmental (Breach in an adjacent joint cartilage, bone cortex (or periosteum) fascia lata, quadriceps, and joint capsule)

• Metastasis:  M 0: No identifiable skip lesions or distant metastases.    M 1: Any skip lesions, regional lymph nodes, or distant mets.

• Enneking's Staging System of Malignant Bone Tumors, CORR 1980

Ia       Low grade, intracompartmental G 1 T 1 M 0Ib       Low grade, extracompartmental G1 T 2 M 0IIa     High grade, intracompartmental G 2 T 1 M 0*IIb     High grade, extracompartmental G 2 T 2 M 0IIIa     Low or High grade, intracompart G 1-2 T 1 M 1                              w/ metastases.IIIb   Low or High grade, extracompart. G 1-2 T 2 M 1                              w/ metastases.Osteosarcoma Most Commonly Diagnosed at IIB

Page 161: Oncology oite-review-2012

Biopsy Pearls

• Do not do the biopsy unless you are the one who is going to take care of the patient– at least talk to the ortho oncologist at your

regional referral center before you biopsy• Never use a transverse incision• Hemostasis is vitally important• 2-12 times greater complication if not done at

treating hospital• FNA-70-75% accurate• Core needle biopsy 85% accurate• Open Biopsy- 96-98% accurate

Page 162: Oncology oite-review-2012

Tumor Resection Definitions

• Intralesional: through the tumor• Marginal: through the reactive zone• Wide: En Blco removal ( Standard)• Radical : Entire compartment removed

Page 163: Oncology oite-review-2012

Pelvic Resections

• Internal hemipelvectomy – keep the limb, +/- reconstruct with allograft hemipelvis (often need hemiarthroplasty in addition, as native femoral head may not fit allograft acetabulum, or native FH blood supply may be taken)

• External hemipelvectomy – take the limb

• Type I – ilium, no reconstruction needed• Type II - periacetabular• Type III – parasymphyseal region• Type IV – involving SI joints/sacrum

Page 164: Oncology oite-review-2012

Immunostains: Bone tumors

• S-100 = chordoma, EG• Keratin= Mets, chordoma, adamantinoma• CD1A = EG• CD20 = Lymphoma• CD99 = Ewing’s• Cd138 = Myeloma

Page 165: Oncology oite-review-2012

Ewing’s Sarcoma

• Chromosomal translocation T(11:22), found in 90% of patients.

• Positive CD 99 reactivity• Positive Vimentin stain (sarcoma stain)• Histology: Monotonous sheets of small

smudgy blue cells, pseudorosettes.• Lung mets are common• Treatment- Chemo, resection vs radiation,

chemo again.

Page 166: Oncology oite-review-2012

Immunostains: Soft Tissue tumors

• Keratin = synovial cell, epitheliod sarcoma• EMA = synovial cell • Smooth muscle actin = leiomyosarcoma• Desmin = Rhabdomyosarcoma• Myoglobin = Rhabdomyosarcoma• S100 = clear cell, nerve sheath tumor• CD34 = Angiosarcoma• Elastin = Elastofibroma

Page 167: Oncology oite-review-2012

Common Chromosomal translocations

• T(11:22) = Ewing’s sarcoma• T (X:18) = synovial sarcoma• T (2:13) = alveolar rhabdomyosarcoma• T (12:22) = clear cell sarcoma• T(9:22) = chondrosarcoma• T(12;16) = myxoid liposarcoma

Page 168: Oncology oite-review-2012

Tumor Suppression Genes

• RB-1– Regulates gene expression– Both must be gone (recessive)– 35% of osteosarcomas

• P53

- Prevent entrance into S-phase

- One bad copy and you have the tumor

- 20-65% of Osteosarcomas

Page 169: Oncology oite-review-2012

Oncogenes

• Induce uncontrolled growth• Resessive• FAK, erb-2• Chemotherapy for these tumors

Page 170: Oncology oite-review-2012

Chemotherapy

• Induces programmed cell death• Eliminated micromets in lungs• Improves survival rate with >95% kill• Reduces recurrence• Bleomycin= concern for pulmonary fibrosis• Adriamycin=concern for cardiomyopathy

Page 171: Oncology oite-review-2012

Immunohistochemistry Review• Keratins: seen in epithelial cells. Classic marker of

metastatic carcinomas. Also seen in sarcomas which show some epithelial differentiation including adamantinoma, epithelioid sarcoma (most common sarcoma found in hand), and synovial sarcoma (most common sarcoma found in foot).

Page 172: Oncology oite-review-2012

Immunohistochemistry Review• Vimentin: typical stain of all tumors of mesothelial origin.

(+) for almost all sarcomas. (-) in carcinomas.

• Desmin: typically found in muscle cells and tumors with myo-differentiaition. These include rhabdomyosarcoma, leiomyosarcoma, and leiomyomas. Occasionally, (+) for desmoids and primitive neuroectodermal tumors.

• Actin: as with desmin, indicative of myogenous differentiation.

• S-100: most commonly associated with neural, chondroid, or melanocytic differentiation.

• Factor VIII related antigen: (+) in lesions with vascular differentiation like hemangiomas.

Page 173: Oncology oite-review-2012

Immunohistochemistry Review

• Metastatic cancer Keratin• Lymphoma CD20 (B cell)• Ewing’s CD99• Synovial Sarcoma vimentin, EMA• Nerve tumors S-100 • Langerhans cell histiocytosis (EG)

S100,CD1A• Chordoma Keratin, S100,

vimentin, EMA• Myeloma CD138• Adamantinoma Keratin

Page 174: Oncology oite-review-2012

WARNING: TEST TOPICS

• CT Chest/Abd/Pelvis to Stage• Osteosarcoma – Chemo/Surgery/Chemo• Chondrosarcoma - Surgery• Metastatic disease (renal cell bleeds, lung in

hand) – Mechanical • Plasmcytoma / Multiple Myeloma -

Mechanical• Fibrous Displasia - Mechanical• Enchondroma - Mechanical• Giant Cell Tumor of Bone – Locally

aggressive, Mechanical, Mets

Page 175: Oncology oite-review-2012

OITE Questions

Page 176: Oncology oite-review-2012

OITE 2008 – Basic Science / Onc

• 15 A 19-year-old man has the mass shown in Figure 5a. MRI reveals a heterogeneous mass. A biopsy specimen is shown in Figure 5b. What is the most likely diagnosis?

• 1- Synovial sarcoma• 2- Epithelioid sarcoma• 3- Liposarcoma• 4- Lipoma• 5- Desmoid tumor

Page 177: Oncology oite-review-2012

OITE 2008 – Basic Science / OncQ15. Figs 5a and 5b

Page 178: Oncology oite-review-2012

OITE 2008 – Basic Science / Onc

• 15 A 19-year-old man has the mass shown in Figure 5a. MRI reveals a heterogeneous mass. A biopsy specimen is shown in Figure 5b. What is the most likely diagnosis?

• 1- Synovial sarcoma• 2- Epithelioid sarcoma• 3- Liposarcoma• 4- Lipoma• 5- Desmoid tumor

Page 179: Oncology oite-review-2012

OITE 2008 – Basic Science / Onc

Recognizing synovial sarcomaGrossly -- well circumscribed, firm, gray-pink tumorRadiographically -- focal calcifications Microscopically – Many are histologically similar to synovial tissue and are characterized by a biphasic growth pattern consisting of plump epithelioid cells that form cords, nests, and glandular spaces, and areas of uniform spindle cells (in plump fascicles with hyalinization and distinct lobulation accompanied by mast cells, occasional osseous or cartilaginous metaplasia, and focal whorling) that resemble fibrosarcomas. May be monophasic without epithelial cells.

Synovial sarcoma• Most prevalent between 15-40 years of age. • More common in males and tends to affect the extremities (85%-95%)• The natural history of primary synovial sarcoma of the extremity is related to tumor

size (> 5 cm) and invasion of bone and neurovascular structures• Treatment: surgical excision with a wide or radical margin and limb preservation when

possible; Perioperative radiation if unable to get wide margin

Lewis JJ, Antonescu CR, Leung DH, et al: Synovial sarcoma: A multivariate analysis of prognostic factors in 112 patients with primary localized tumors of the extremity. J Clin Oncol 2000;18:2087-2094.Thompson RC Jr, Garg A, Goswitz J, et al: Synovial sarcoma: Large size predicts poor outcome. Clin Orthop Relat Res 2000;373:18-24.

Page 180: Oncology oite-review-2012

OITE 2008 – Basic Science / Onc

• 31 A 34-year-old woman has a painful right knee mass. A radiograph, MRI scans, and H & E biopsy specimens (20X and 100X) are shown in Figures 10a through 10d. What is the next most appropriate step in management?

• 1- Chemotherapy alone• 2- Arthrotomy with intralesional resection• 3- Extra-articular wide resection• 4- Above-knee amputation• 5- CT of the chest and a bone scan

Page 181: Oncology oite-review-2012

OITE 2008 – Basic Science / Onc

Q31. Figs 10a & 10b

Page 182: Oncology oite-review-2012

OITE 2008 – Basic Science / Onc

• Q31. Figs 10c & 10d

Page 183: Oncology oite-review-2012

OITE 2008 – Basic Science / Onc

• A 34-year-old woman has a painful right knee mass. A radiograph, MRI scans, and H & E biopsy specimens (20X and 100X) are shown in Figures 10a through 10d. What is the next most appropriate step in management?

• 1- Chemotherapy alone• 2- Arthrotomy with intralesional resection• 3- Extra-articular wide resection• 4- Above-knee amputation• 5- CT of the chest and a bone scan

Page 184: Oncology oite-review-2012

OITE 2008 – Basic Science / Onc

• PVNS– PVNS is a proliferation of synovial villi and nodules that may occur

locally within a joint or diffusely. Proliferation of synovial tissue can affect the whole of the synovium (diffuse), or a localised area (nodular). 

– Microscopically – subsynovial nodular proliferation of large round, polyhedral, or spindle cells with prominent cytoplasm and pale nuclei; phagocytic histiocyte-like cells, lipid-laden foam cells and multinucleated giant cells are interspersed with hemosiderin-laden cells (look for iron deposition – pathopneumonic of PVNS)

– Most common presentation – Progressive and insidious onset of pain and swelling of a joint

• Knee is most commonly affected, but it is also seen in the hip and shoulder

• Typically in 3rd to 4th decade – Treatment – Complete open or arthroscopic synovectomy

Page 185: Oncology oite-review-2012

OITE 2008 – Basic Science / Onc

• A 10-year-old girl has had pain in the right hip for the past 2 weeks. She denies any history of trauma or other diseases. Imaging studies and biopsy specimens are shown in Figures 17a through 17e. What is the most appropriate treatment?

• 1- Preoperative chemotherapy, wide resection, and postoperative chemotherapy

• 2- Hip arthrotomy, irrigation, and intravenous antibiotics for 6 weeks

• 3- In situ pinning with a 7.3-mm cannulated screw• 4- Radiation therapy• 5- Crutch walking and nonsteroidal anti-inflammatory drugs

Page 186: Oncology oite-review-2012

OITE 2008 – Basic Science / Onc

Q46. Fig 17a

Page 187: Oncology oite-review-2012

OITE 2008 – Basic Science / OncQ46. Fig 17 b & c

Page 188: Oncology oite-review-2012

OITE 2008 – Basic Science / Onc

Q46. Fig 17 d & e

Page 189: Oncology oite-review-2012

OITE 2008 – Basic Science / Onc

• A 10-year-old girl has had pain in the right hip for the past 2 weeks. She denies any history of trauma or other diseases. Imaging studies and biopsy specimens are shown in Figures 17a through 17e. What is the most appropriate treatment?

• 1- Preoperative chemotherapy, wide resection, and postoperative chemotherapy

• 2- Hip arthrotomy, irrigation, and intravenous antibiotics for 6 weeks

• 3- In situ pinning with a 7.3-mm cannulated screw• 4- Radiation therapy• 5- Crutch walking and nonsteroidal anti-inflammatory drugs

Page 190: Oncology oite-review-2012

OITE 2008 – Basic Science / Onc

• Recognizing Osteosarcoma– Grossly: large tender soft tissue mass typically over

metaphyses of long bones– Radiologically: destruction of the normal trabecular

bone pattern, indistinct margins, no endosteal bone response, a mixture of radiodense and radiolucent areas, with periosteal new bone formation, lifting of the cortex, and Codman's triangle

– Microscopically: malignant sarcomatous stroma, w/ production of osteoid and bone • 3 main types: osteoblastic, chondroblastic, fibroblastic

Page 191: Oncology oite-review-2012

OITE 2008 – Basic Science / Onc

• 53 A 5-year-old girl has knee pain, a low grade fever, and a limp. A radiograph and MRI scan are shown in Figures 19a and 19b, and a biopsy specimen is shown in Figure 19c. Molecular genetic analysis identifies an EWS-FLI1 gene rearrangement. In addition to a CT scan of the chest and a bone scan, what other study is indicated for complete tumor staging?

• 1- Spinal tap• 2- MRI of the brain• 3- Bone marrow biopsies• 4- Sentinel lymph node biopsy• 5- Serum alkaline phosphatase

Page 192: Oncology oite-review-2012

OITE 2008 – Basic Science / OncQ53. Figs 19 a – c.

Page 193: Oncology oite-review-2012

OITE 2008 – Basic Science / Onc

• 53 A 5-year-old girl has knee pain, a low grade fever, and a limp. A radiograph and MRI scan are shown in Figures 19a and 19b, and a biopsy specimen is shown in Figure 19c. Molecular genetic analysis identifies an EWS-FLI1 gene rearrangement. In addition to a CT scan of the chest and a bone scan, what other study is indicated for complete tumor staging?

• 1- Spinal tap• 2- MRI of the brain• 3- Bone marrow biopsies• 4- Sentinel lymph node biopsy• 5- Serum alkaline phosphatase

Page 194: Oncology oite-review-2012

OITE 2008 – Basic Science / Onc

• Recognizing Ewing’s Sarcoma

Grossly – rapid growing painful mass that tender on exam

Radiographically –highly aggressive appearance with permeative or “moth-eaten” bone destruction occurring in the metadiaphysis of long bones. Codman’s triangle often present. Periosteal reaction (classically: onion skin appearance) and soft tissue mass without calcifications are common.

Microscopically – Small blue cell tumor; can’t differentiate from other small blue cell tumors therefore need BM Bx

• Simon MA, Springfield D (eds): Surgery for Bone and Soft Tissue Tumors. Philadelphia, PA, Lippincott-Raven, 1998, pp 287-297.

• Carvajal R, Meyers P: Ewing's sarcoma and primitive neuroectodermal family of tumors. Hematol Oncol Clin North Am 2005;19:501-525.

Page 195: Oncology oite-review-2012

85. A 15 yo cross country runner reports shin pain that is present during running, at rest, and at night. The pain is relieved by ibuprofen. Radiographs, a bone scan, and a CT scan are shown in Figures 32a through 32c. What is the most likely diagnosis?

1. Cortical desmoid

2. Osteoblastoma

3. Stress fracture

4. Enchondroma

5. Osteoid osteoma

Page 196: Oncology oite-review-2012
Page 197: Oncology oite-review-2012

85. A 15 yo cross country runner reports shin pain that is present during running, at rest, and at night. The pain is relieved by ibuprofen. Radiographs, a bone scan, and a CT scan are shown in Figures 32a through 32c. What is the most likely diagnosis?

1. Cortical desmoid

2. Osteoblastoma

3. Stress fracture

4. Enchondroma

5. Osteoid osteoma

Page 198: Oncology oite-review-2012

92.A 45 yo man has a long-standing history of an ankle mass. He now reports that the mass has enlarged and become quite painful over the past several months. A radiograph and axial MRI scans are shown in Figures 35a and 35b. What is the next most appropriate step in management?

93.Below knee amputaton

94.Neoadjuvant chemotherapy

95.Radiation Therapy

96.Chest imaging and a bone scan

97.Limb salvage with an allograft arthrodesis

Page 199: Oncology oite-review-2012
Page 200: Oncology oite-review-2012

92.A 45 yo man has a long-standing history of an ankle mass. He now reports that the mass has enlarged and become quite painful over the past several months. A radiograph and axial MRI scans are shown in Figures 35a and 35b. What is the next most appropriate step in management?

93.Below knee amputaton

94.Neoadjuvant chemotherapy

95.Radiation Therapy

96.Chest imaging and a bone scan

97.Limb salvage with an allograft arthrodesis

Page 201: Oncology oite-review-2012

143. An 11 yo boy falls during a soccer game and is unable to finish the match. The radiographs shown in figures 62a and 62b reveal a radiolucent lesion of the tibia but no fracture. What is the most appropriate management?

1. Repeat radiographs in 3 to 6 months

2. Needle biopsy and radiofrequency ablation under CT direction

3. MRI, bone scane, chest CT, biopsy, and chemotherapy

4. Curettage and bone grafting

5. En bloc resection with autogenous tricortical bone grafting

Page 202: Oncology oite-review-2012
Page 203: Oncology oite-review-2012

143. An 11 yo boy falls during a soccer game and is unable to finish the match. The radiographs shown in figures 62a and 62b reveal a radiolucent lesion of the tibia but no fracture. What is the most appropriate management?

1. Repeat radiographs in 3 to 6 months

2. Needle biopsy and radiofrequency ablation under CT direction

3. MRI, bone scane, chest CT, biopsy, and chemotherapy

4. Curettage and bone grafting

5. En bloc resection with autogenous tricortical bone grafting

Page 204: Oncology oite-review-2012

159. A 12 yo boy is seen in the ED at 11pm with severe thigh pain. A radiograph is shown in Figure 69. A decision is made to transfer his care to the local orthopaedic oncologist in the morning. What should be done prior to transfer?

1. Splinting and pharmacologic pain control

2. Biopsy

3. Spanning external fixator

4. Femoral traction pin

5. Crutch training

Page 205: Oncology oite-review-2012
Page 206: Oncology oite-review-2012

159. A 12 yo boy is seen in the ED at 11pm with sever thigh pain. A radiograph is shown in Figure 69. A decision is made to transfer his care to the local orthopaedic oncologist in the morning. What should be done prior to transfer?

1. Splinting and pharmacologic pain control

2. Biopsy

3. Spanning external fixator

4. Femoral traction pin

5. Crutch training

Page 207: Oncology oite-review-2012

OITE 2008—Orthopedic Diseases

162. A 34-year-old man has had progressive back, rib, and right thigh pain for the past 2 months. A bone scan is shown in figure 72. Further work-up includes routine laboratory analysis, including PSA; radiographs of the involved bones, and CT of the chest, abdomen, and pelvis. A primary is not discovered. The prognosis for this patient is most similar to patients with bone metastases from which of the following primary sites?

1. Breast

2. Prostate

3. Kidney

4. Thyroid

5. Lung

Page 208: Oncology oite-review-2012

OITE 2008—Orthopedic Diseases

Figure 72:

Page 209: Oncology oite-review-2012

OITE 2008—Orthopedic Diseases

162. A 34-year-old man has had progressive back, rib, and right thigh pain for the past 2 months. A bone scan is shown in figure 72. Further work-up includes routine laboratory analysis, including PSA; radiographs of the involved bones, and CT of the chest, abdomen, and pelvis. A primary is not discovered. The prognosis for this patient is most similar to patients with bone metastases from which of the following primary sites?

1. Breast

2. Prostate

3. Kidney

4. Thyroid

5. Lung

Page 210: Oncology oite-review-2012

OITE 2008—Orthopedic Diseases

Schwartz HS (ed): OKU: MSK Tumors 2. Rosemont, IL, American. AAOS, 2007, pp 353-357.

“In 3-4% of patients with metastatic carcinoma, the primary site of origin is unknown.”

Breast, lung, thyroid, kidney, prostate classically met to bones (“BLT w/ Kosher Pickle” if you recall USMLE Step I).

“If the lesion appears to primarily involve the cortex of a bone, or is located distal to the knee or elbow, it is more likely to originate from an occult primary carcinoma of the lung than from other sites.”

“In one study, radiographs of the chest identified the lung as the primary tumor site in 43% of patients.”

Page 211: Oncology oite-review-2012

OITE 2008—Orthopedic Diseases

167. A 54-year-old man with a history of alcoholism and prostate carcinoma has mild knee pain after falling. Radiographs are shown in Figures 74a nd 74b. What is the most likely radiographic diagnosis?

1. Chondrosarcoma

2. Bone infarct

3. Osteogenic sarcoma

4. Metastatic carcinoma of uncertain origin.

5. Metastatic prostate carcinoma.

Page 212: Oncology oite-review-2012

OITE 2008—Orthopedic Diseases

Figure 74a: Figure 74b:

Page 213: Oncology oite-review-2012

OITE 2008—Orthopedic Diseases

167. A 54-year-old man with a history of alcoholism and prostate carcinoma has mild knee pain after falling. Radiographs are shown in Figures 74a nd 74b. What is the most likely radiographic diagnosis?

1. Chondrosarcoma

2. Bone infarct

3. Osteogenic sarcoma

4. Metastatic carcinoma of uncertain origin.

5. Metastatic prostate carcinoma.

Page 214: Oncology oite-review-2012

OITE 2008—Orthopedic Diseases

Bone Infarction vs. Enchondroma• Linear or serpiginous pattern more suggestive of

infarction; popcorn or floculent pattern w/ arcs/swirls matrix suggestive of chondroid matrix/enchondroma.

Bone Infarct Enchondroma

Page 215: Oncology oite-review-2012

OITE 2008—Orthopedic Diseases

173. A 24-year-old man has a painful mass on his left humerus. A radiograph, MRI scan, and a biopsy specimen are shown in Figures 77a through 77c. What is the most likely diagnosis?

1. Myositis ossificans

2. Osteochondroma

3. Periosteal osteosarcoma

4. Periosteal chondroma

5. Sarcoma

Page 216: Oncology oite-review-2012

OITE 2008—Orthopedic Diseases

Figure 77a: Figure 77b:

Page 217: Oncology oite-review-2012

OITE 2008—Orthopedic Diseases

Figure 77c:

Page 218: Oncology oite-review-2012

OITE 2008—Orthopedic Diseases

173. A 24-year-old man has a painful mass on his left humerus. A radiograph, MRI scan, and a biopsy specimen are shown in Figures 77a through 77c. What is the most likely diagnosis?

1. Myositis ossificans

2. Osteochondroma

3. Periosteal osteosarcoma

4. Periosteal chondroma

5. Sarcoma

Page 219: Oncology oite-review-2012

OITE 2008—Orthopedic Diseases

Periosteal chondroma: atlasgeneticsoncology.org

Periosteal chondroma is a painful cartilaginous lesion that arises from surface of cortex deep to the periosteum, producing broad based cartilaginous mass that may extend into soft tissues; often develops after adolescence. Characteristically erodes and induces sclerosis of the contiguous cortical bone.

Pathology: It persists as mass of mature cartilage. Low power microscopy shows well circumscribed lobulated hyaline masses. Cellularity can vary, from hypo- to hyper-cellularity. The cartilage looks more active than enchondroma and the lesion may be confused with chondrosarcoma.

Page 220: Oncology oite-review-2012

OITE 2008—Orthopedic Diseases

174. A 29-year-old woman has a right shoulder mass. Figures 78a through 78c show the radiograph, CT scan, and H&E biopsy specimen. These findings are associated with which of the following conditions?

1. Multiple hereditary exostoses

2. Hyperparathyroidism

3. Renal failure

4. Retinoblastoma

5. Hemangiomatosis

Page 221: Oncology oite-review-2012

OITE 2008—Orthopedic Diseases

Figure 78a: Figures 78b:

Page 222: Oncology oite-review-2012

OITE 2008—Orthopedic Diseases

Figure 78c:

Page 223: Oncology oite-review-2012

OITE 2008—Orthopedic Diseases

174. A 29-year-old woman has a right shoulder mass. Figures 78a through 78c show the radiograph, CT scan, and H&E biopsy specimen. These findings are associated with which of the following conditions?

1. Multiple hereditary exostoses

2. Hyperparathyroidism

3. Renal failure

4. Retinoblastoma

5. Hemangiomatosis

Page 224: Oncology oite-review-2012

OITE 2008—Orthopedic Diseases

204. A 60-year-old man has left shoulder pain. A review of systems reveals hematuria. A radiograph of the affected area is shown in Figure 89a, and MRI scans are shown in Figures 89b and 89c. Examination reveals a pulsatile mass. CT of the chest, abdomen, and pelvis is likely to reveal which of the following findings?

1. Spiculated lung mass

2. Thickened colon

3. Renal mass

4. Hilar adenopathy

5. Prostate irregularity

Page 225: Oncology oite-review-2012

OITE 2008—Orthopedic Diseases

Figure 89a: Figure 89b:

Page 226: Oncology oite-review-2012

OITE 2008—Orthopedic Diseases

Figure 89c:

Page 227: Oncology oite-review-2012

OITE 2008—Orthopedic Diseases

204. A 60-year-old man has left shoulder pain. A review of systems reveals hematuria. A radiograph of the affected area is shown in Figure 89a, and MRI scans are shown in Figures 89b and 89c. Examination reveals a pulsatile mass. CT of the chest, abdomen, and pelvis is likely to reveal which of the following findings?

1. Spiculated lung mass

2. Thickened colon

3. Renal mass

4. Hilar adenopathy

5. Prostate irregularity

Page 228: Oncology oite-review-2012

OITE 2008—Orthopedic Diseases

Rougraff BT: Evaluation of the patient with carcinoma of unknown origin metastatic to bone. Clin Orthop Relat Res 2003;415:S105-S109.

Breast, lung, thyroid, kidney, prostate classically met to bones (“BLT w/ Kosher Pickle” if you recall USMLE Step I).

• This patient has hematuria, which immediately raises the suspicion of renal cell carcinoma.

• Renal cell is also very vascular and therefore a large lesion can be pulsatile.• Note that according to above reference, most mets to bone of unknown

origin are either lung (most common) or renal, possibly because these organs are harder to examine on physical exam or tumors may grow larger before becoming symptomatic.

Page 229: Oncology oite-review-2012

OITE 2008 Orthopaedic Diseases

214. A 43-year-old woman reports a 2-month history of increasing knee pain. She has no history of malignancy. Radiographs, MRI scans, and a biopsy specimen are showin in Figures 92a through 92e. What is the most likely diagnosis?

1. Chordoma

2. Giant cell tumor

3. Metastatic carcinoma

4. Osteosarcoma

5. Aneurysmal bone cyst

Page 230: Oncology oite-review-2012
Page 231: Oncology oite-review-2012

OITE 2008 Orthopaedic Diseases

214. A 43-year-old woman reports a 2-month history of increasing knee pain. She has no history of malignancy. Radiographs, MRI scans, and a biopsy specimen are showin in Figures 92a through 92e. What is the most likely diagnosis?

1. Chordoma

2. Giant cell tumor (Correct location but no giant cells)

3. Metastatic carcinoma

4. Osteosarcoma

5. Aneurysmal bone cyst

1. Dorfman HD, Czerniak B: Bone Tumors. St Louis, MO, Mosby, 1998, pp 1009-1029.2. Marcove RC, Arlen M: Atlas of Bone Pathology. Philadelphia, PA, JB Lippincott, 1992, pp 522-530.

Page 232: Oncology oite-review-2012

OITE 2008 Orthopaedic Diseases

1. Chordoma (Wrong Location, Usually in Sacrum, base of skull and rare spine; Correcta age; malignant slow growing neoplasm of notochord remnant; Physaliphorous Cells – Large cells with central nuclei, numerous vacuoles in eosinophilic cytoplasm)

2. Giant cell tumor (Correct location but no giant cells)

3. Metastatic carcinoma – (Pathology shows clear cell variant of Renal Carcinoma; Mets are the most common diagnosis; just have to know it is not the other answers)

4. Osteosarcoma (Not the correct age; Usually does not cross the physis; No osteoid on path)

5. Aneurysmal bone cyst (Common in spine and flat bones although 15% occur in the femur; Large spaces marked by septa filled with blood; Giant Cells may be present; Occasional osteoclast lining of septa)

1. Dorfman HD, Czerniak B: Bone Tumors. St Louis, MO, Mosby, 1998, pp 1009-1029.2. Marcove RC, Arlen M: Atlas of Bone Pathology. Philadelphia, PA, JB Lippincott, 1992, pp 522-530.

Page 233: Oncology oite-review-2012

OITE 2008 Orthopaedic Diseases

226. A 56-year-old man with a history of neurofibromatosis reports burning pain in his foot. An MRI scan of the thigh reveals the mass shown in Figure 96A. A biopsy is most likely to reveal the histology shown in which of the following figures?

1. Figure 96b

2. Figure 96c3. Figure 96d

4. Figure 96e5. Figure 96f

Page 234: Oncology oite-review-2012

1. Figure 96b

2. Figure 96c

3. Figure 96d

4. Figure 96e

5. Figure 96f

Page 235: Oncology oite-review-2012

226. A 56-year-old man with a history of neurofibromatosis reports burning pain in his foot. An MRI scan of the thigh reveals the mass shown in Figure 96A. A biopsy is most likely to reveal the histology shown in which of the following figures?

1. Figure 96b

2. Figure 96c3. Figure 96d

4. Figure 96e5. Figure 96f

1. Weiss SW, Goldblum JR: Benign tumors of peripheral nerves, in Enzinger FM, Weiss SW (eds): Soft Tissue Tumors. St Louis, MO, Mosby, 2001, pp 1111-1207.2. Schwartz HS (ed): Orthopaedic Knowledge Update: Musculoskeletal Tumors 2. Rosemont, IL, American Academy of Orthopaedic Surgeons, 2007, pp 273-275

OITE 2008 Orthopaedic Diseases

Page 236: Oncology oite-review-2012

1. Round blue cells with myxoid features can been seen in types of chondrosarcoma

2. Osteoid with malginant spindle cell stroma seen in Osteosarcoma

3. Wavy Spindle Cells are seen in Neurofibromas

4. No stroma and low cystoplasm to nuclear ration can be seen in Ewing’s Scarcoma

5. Vacuoles can be seen in Chordomas and Fatty Tumors

Page 237: Oncology oite-review-2012

OITE 2008 Orthopaedic Diseases

236. A 28-year-old woman notes a growing mass in her hand for the past several weeks. Examination reveals a nontender mass in her palm with no skin changes or ipsilatearl adenopathy. An axial MRI scan is shown in Figure 98a and a biopsy specimen is shown in Figure 98b. Immunohistochemistry analysis is positive for epithelial membrane antigen, cytokeratin and sporadic S-100. What is the most likely diagnosis?

1. Glomus tumor

2. Squamous cell carcinoma

3. Ganglion

4. Giant cell tumor of the tendon sheath

5. Synovial sarcoma

Page 238: Oncology oite-review-2012
Page 239: Oncology oite-review-2012

236. A 28-year-old woman notes a growing mass in her hand for the past several weeks. Examination reveals a nontender mass in her palm with no skin changes or ipsilatearl adenopathy. An axial MRI scan is shown in Figure 98a and a biopsy specimen is shown in Figure 98b. Immunohistochemistry analysis is positive for epithelial membrane antigen, cytokeratin and sporadic S-100. What is the most likely diagnosis?

1. Glomus tumor

2. Squamous cell carcinoma

3. Ganglion

4. Giant cell tumor of the tendon sheath

5. Synovial sarcoma

1. Weiss SW, Goldblum JR, et al: Malignant soft-tissue tumors of uncertain type, in Strauss M, Gery L (eds): Enzinger and Weiss's Soft Tissue Tumors, ed 4. St Louis,MO, Mosby, 2001, pp 1483-1509.2. Pradhan A, Cheung YC, Grimer RJ, et al: Soft-tissue sarcomas of the hand: Oncological outcome and prognostic factors. J Bone Joint Surg Br 2008;90:209-214.

OITE 2008 Orthopaedic Diseases

Page 240: Oncology oite-review-2012

1. Glomus tumor – related to glomus bodies which help with temperature regulation, paroxysmal pain, worse with cold, Dx with MRI

2. Squamous cell carcinoma – very rare in the hand, path would likely have keratinized layer of skin, will not be the OITE answer

3. Ganglion – lesion not associated with, MRI shows homogenous fluid filled cystic structure

4. Giant cell tumor of the tendon sheath – Benign lesion associated with tendon sheath, giant cells are usally present on path, must differenciat from syncovial cell sarcoma on path, treatment is marginal excision, 30% reoccur

5. Synovial sarcoma – S-100+, EMA and Cytokeratin +, Spindle cells on path, Associate with tendon sheaths, 80% occur in the extremities, used to be called tendosynovial sarcoma

OITE 2008 Orthopaedic Diseases

Synovial Sarcoma

Giant Cell Tumor of tendon sheath

www.bweems.com/gctten.html

Page 241: Oncology oite-review-2012

OITE 2008 Orthopaedic Diseases

252. Figure 102 shows the radiograph of a 40-year-old man who reports worsening knee pain and a limp. A bone scan reveals no other sites of disease. What is the most likely diagnosis?

1. Chondroblastoma

2. Low-grad intramedularry osteogenic sarcoma

3. Giant cell tumor

4. Ewing’s sarcoma

5. Enchondroma

Page 242: Oncology oite-review-2012
Page 243: Oncology oite-review-2012

OITE 2008 Orthopaedic Diseases

252. Figure 102 shows the radiograph of a 40-year-old man who reports worsening knee pain and a limp. A bone scan reveals no other sites of disease. What is the most likely diagnosis?

1. Chondroblastoma (Patient is too old, diagnosed < 20 y/o)

2. Low-grade intramedularry osteogenic sarcoma (Does not cross the physis)

3. Giant cell tumor (Correct age, crosses the physis)

4. Ewing’s sarcoma (Does not cross the physis)

5. Enchondroma (Does not cross the physis)

1. Turcotte RE: Giant cell tumor of bone. Orthop Clin North Am 2006;37:35-51.2. Greenspan A, Jundt G, Remagen W (eds): Differential Diagnosis in Orthopaedic Oncology, ed 2. Philadelphia, PA, Lippincott Williams & Wilkins, 2007, pp 387-457.

Page 244: Oncology oite-review-2012

OITE 2008 Orthopaedic Diseases

273. A 28-year-old man with a history of osteogenic sarcoma 10 years ago was treated with standard multi-agent chemotherapy that included doxorubicin. Currently, he is disease free and needs to undergo revision of a distal femur replacement. What test should be performed as part of his preop work-up?

1. Liver ultrasound

2. Echocardiography

3. Bone densitometry

4. Bone marrow aspiration

5. Skin tuberculin testing

Page 245: Oncology oite-review-2012

OITE 2008 Orthopaedic Diseases273. A 28-year-old man with a history of osteogenic sarcoma 10 years

ago was treated with standard multi-agent chemotherapy that included doxorubicin. Currently, he is disease free and needs to undergo revision of a distal femur replacement. What test shoulder be performed as part of his preop work-up?

1. Liver ultrasound (many are hepatotoxic, follow with LFT’s, liver mets response to chemo can be followed with US)

2. Echocardiography (Doxorubicin is cardiotoxic)

3. Bone densitometry (Methotrexate)

4. Bone marrow aspiration (L-phenylalanine mustard (L-PAM) leukemia)

5. Skin tuberculin testing – can be offered before chemo so prophylaxis can be started before they are immunocompromized

Page 246: Oncology oite-review-2012

OITE 2008—Science

136. Which of the following is diagnosed by urine protein electrophoresis?

1- Monoclonal heavy chains of myeloma

2- Monoclonal light chains of myeloma

3- Diffuse B-cell lymphoma

4- Primary lymphoma of bone

5- Amyloidosis

Page 247: Oncology oite-review-2012

OITE 2008—Science

136. Which of the following is diagnosed by urine protein electrophoresis?

1- Monoclonal heavy chains of myeloma

2- Monoclonal light chains of myeloma3- Diffuse B-cell lymphoma

4- Primary lymphoma of bone

5- Amyloidosis

RECOMMENDED READINGS:

Dorfman HD, Czerniak B: Bone Tumors. St Louis, MO, Mosby, 1998, pp 664-728.

Page 248: Oncology oite-review-2012

Explanation

• Myeloma is an overproduction of Plasma cells ( B-cells) overproducing one kind of antibody.

• These antibodies are composed of light-chained and heavy-chained parts; the light-chained parts can be detected in high quantities in the urine, called Bence-Jones proteins, and are diagnositic for myeloma.

Page 249: Oncology oite-review-2012

OITE 2009 – Basic Science & Tumors

142. What predominant cell type is typically seen on a biopsy specimen in a patient with multiple myeloma?

1. Spindle cells

2. Giant cells

3. Osteoclasts

4. Lymphocytes

5. Plasma cells

Page 250: Oncology oite-review-2012

OITE 2009 – Basic Science & Tumors

142. What predominant cell type is typically seen on a biopsy specimen in a patient with multiple myeloma?

1. Spindle cells

2. Giant cells

3. Osteoclasts

4. Lymphocytes

5. Plasma cells

Page 251: Oncology oite-review-2012

OITE 2009 – Basic Science & Tumors

Reference: Fletcher C, Unni K, Mertens F (eds): Pathology and Genetics of Tumors of Soft Tissue and Bone.

Multiple myeloma

-characteristic cell = plasma cell

-most common neoplasm of bone

-6th & 7th decades of life

-characterized by osteolytic lesions,bone pain, hypercalcemia,monoclonal gammopathy,amyloid deposits (eg in kidney)

-targets = vertebrae, ribs, skull, pelvis, femur, clavicle, scapula

-diagnose w/ bone marrow aspirate; often cold on bone scan

-Poor prognosis (median survival 3 years)

Plasma cells in multiple myeloma

Page 252: Oncology oite-review-2012

OITE 2009 – Basic Science & Tumors

156. Cytogenetic analysis is a useful approach for detecting which of the following genetic abnormalities?

1. Point mutations

2. Frame-shift mutations

3. Microsatellite instability

4. Translocations

5. DNA cross-linking

Page 253: Oncology oite-review-2012

OITE 2009 – Basic Science & Tumors

156. Cytogenetic analysis is a useful approach for detecting which of the following genetic abnormalities?

1. Point mutations

2. Frame-shift mutations

3. Microsatellite instability

4. Translocations

5. DNA cross-linking

Page 254: Oncology oite-review-2012

51. A 12 yo boy sustained a fracture in the distal ulna twice in the past year. He denies any major fall or injury. A radiograph and a biopsy speciman are shown in Figures 17a and 17b. What is the most likely diagnosis?

1. Chondroblastoma

2. Giant Cell Tumor

3. Unicameral bone cyst

4. Fibrous dysplasia

5. Nonossifying fibroma

Page 255: Oncology oite-review-2012
Page 256: Oncology oite-review-2012

51. A 12 yo boy sustained a fracture in the distal ulna twice in the past year. He denies any major fall or injury. A radiograph and a biopsy speciman are shown in Figures 17a and 17b. What is the most likely diagnosis?

1. Chondroblastoma

2. Giant Cell Tumor

3. Unicameral bone cyst

4. Fibrous dysplasia

5. Nonossifying fibroma

Page 257: Oncology oite-review-2012

UBCs

1. Occur in young pts

2. Present with pain/path fx after minor trauma

3. XRAY: Central, lytic, cortical thinning and expansion but not usually beyond width of physis, fallen leaf sign

4. PATH: Thin fibrous membrane, giant cells, hemosiderin, lipophages, chronic inflammation

5. TREATMENT: Steroid injection vs. curettage & packing if in high risk location (prox femur)

Page 258: Oncology oite-review-2012

104. Synovial sarcoma is closely associated with translocation of which of the following genes?

1 – BCR-ABL

2 – EWS-FLI1

3 – PDGF-COL1A1

4 – SYT-SSX

5 – PAX3-FKHR

Page 259: Oncology oite-review-2012

104. Synovial sarcoma is closely associated with translocation of which of the following genes?

1 – BCR-ABL = (CML)

2 – EWS-FLI1 = (Ewings)

3 – PDGF-COL1A1 = (Dermatofibrosarcoma Protuberans)

4 – SYT-SSX

5 – PAX3-FKHR = (Alveolar rhabdomyosarcoma)

Page 260: Oncology oite-review-2012

251.Which of the following soft tissue sarcomas is most common in children?

1 – epitheloid sarcoma

2 – rhabdomyosarcoma

3 – synovial sarcoma

4 – liposarcoma

5 – extraskeletal Ewing’s sarcoma

Page 261: Oncology oite-review-2012

251.Which of the following soft tissue sarcomas is most common in children?

1 – epitheloid sarcoma

2 – rhabdomyosarcoma

3 – synovial sarcoma

4 – liposarcoma

5 – extraskeletal Ewing’s sarcoma

Page 262: Oncology oite-review-2012

• Five most common malignant bone tumors in adults >50– Metastases (PTBLK)– Lymphoma– Myeloma (most common primary bone tumor)– Chondrosarcoma (most common sarcoma

age>50)– Osteosarcoma (most common sarcoma)

• Ewings is most common malignant bone tumor in children/adolescent

• Rhabdomyosarcoma is most common soft tissue sarcoma in children

Page 263: Oncology oite-review-2012

65. For which of the following skeletal tumors is radiation therapy routinely used for definitive local control?

1- Osteogenic sarcoma

2- Ewing’s sarcoma

3- Chondrosarcoma

4- Pleomorphic sarcoma

5- Paget’s sarcoma

Page 264: Oncology oite-review-2012

65. For which of the following skeletal tumors is radiation therapy routinely used for definitive local control?

1- Osteogenic sarcoma

2- Ewing’s sarcoma

3- Chondrosarcoma

4- Pleomorphic sarcoma

5- Paget’s sarcoma

Page 265: Oncology oite-review-2012

Question 232 - What is the most current recommendation for definitive treatment of a 15 year-old boy who has a high grade osteosarcoma of the distal femur?   1. Surgical resection only

2. Raditation therapy only3. Radiation therapy and surgical resection 4. Chemotherapy only5. Chemotherapy and surgical resection

 

Page 266: Oncology oite-review-2012

Answer: 5 Chemotherapy and surgical resection combined has lead to a >80% 5 year survival rate. Surgical restriction alone has lead to a high recurrence and metastatic rates and poor (17.4%) survival rates. Radiation therapy alone did not control local recurrences and pulmonary metastasis. radiation and surgery also had high local recurrences and metastatic rates. Neoadjunctive chemotherapy has provided many improvements in treatment including elimination of micro metastases, necrosis of the primary tumor, reduction of tumor size and neovasculization and helps prevent local recurrences. Complete surgical resection is the mainstay of treatment of osteogenic sarcoma, the addition of neoadjunctive chemotherapy has proven to be the most effective.

Page 267: Oncology oite-review-2012

Question 228 - Which of the following terms best describe most osteosarcomas at the time of diagnosis?  1. Metastatic 2. Low-grade, intracompartmental 3. Low-grade, extracompartmental 4. High-grade intracompartmental 5. High-grade extracompartmental

Page 268: Oncology oite-review-2012

Answer: 5 DISCUSSION: Osteosarcoma is a difficult disease to cure because it is predominantlyhigh‑grade and extracompartmental at the time of presentation, 90% stage IIB. Thecommon presentation is child or young adult with complaints of pain around the knee.The proximal femur and humerus, as well as, the pelvis are not uncommon sites. Pre operative multiagent chemotherapy is given for 8‑12 weeks and then staging studies areperformed. Resection, if possible, is then performed and maintenance chemotherapy isgiven for 6‑12 months. Long term survival has increased from 10‑20% to 60‑70%, withincreased limb salvage.  REFERENCES: Enneking WF, Spanier SS, Goodman MA. Clin Orthop 1980;153:106-20. This question deals with the surgical staging of musculoskeletal tumors: STAGE GRADE SITEIA LOW INTRACOMPARTMENTALIB LOW EXTRACOMPARTMENTALIIA HIGH INTRACOMPARTMENTALIIB HIGH EXTRACOMPARTMENTALIII ANY METS Upon diagnosis, most osteosarcomas are high grade, extracompartmental lesions, (stage IIB).

Page 269: Oncology oite-review-2012

Question 186 - A 55 year old man with metastatic prostate cancer has a painful lesion of the midshaft of the humerus in which approx. 75% of the cortex is involved. Management should consist of. 1. an incisional biopsy. 2. a humeral cuff and a sling 3. closed interlocking nailing 4. radiation therapy to the humerus 5. plate fixation with bone fixation

Page 270: Oncology oite-review-2012

Answer: 3  Redmond reviewed 13 patients who had 16 pathological fractures of the shaft of the humerus secondary to metastatic disease. All but one fracture was stabilized with closed IM nailing. 14 extremities returned to nearly normal function in 3 weeks. The conclusion stated that IM nailing of the humerus for pathological fractures provides immediate stability and can be accomplished with a closed technique, brief operative time, minimum morbidity, with a resultant early return of function to the extremity.

Page 271: Oncology oite-review-2012

Question 89 - A 14-year-old boy has a 4-month history of aching pain in the distal thigh. Examination reveals a mass in the distal thigh. Figure 25a shows a plain radiograph, Figures 25b and 25c show MRI images, Figure 25d shows a bone scan, and Figure 25e shows a CT scan of the chest. The most likely diagnosis and Musculoskelatal Tumor Society (Enneking) stage is  1. osteosarcoma, stage IIB

2. osteosarcoma, stage III3. parosteal osteosarcoma, stage IIB4. periosteal osteosarcoma, stage IIB5. periosteal osteosarcoma, stage III

Page 272: Oncology oite-review-2012
Page 273: Oncology oite-review-2012
Page 274: Oncology oite-review-2012
Page 275: Oncology oite-review-2012
Page 276: Oncology oite-review-2012

Answer: 2 Osteosarcoma commonly occurs about the knee in children and young adults. Patients present primarily with pain. More than 90% of the intramedullary osteosarcomas are high grade and penetrate the cortex early to form a soft tissue mass. About 10-20% of patients have pulmonary metastases at presentation. Plain radiographs demonstrate a lesion in which there is both bone destruction and bone formation. MRI and CT scans are useful for defining the anatomy of the lesion in regard to intramedullary extension, involvement of neurovascular structures, and muscle invasion.Parosteal Osteosarcoma commonly occurs at the posterior aspect of the distal femur, proximal tibia, and proximal humerus. Patients present with a painless mass. The radiographic appearance is characteristic in that it demonstrates a heavily ossified, often lobulated mass arising form the cortex.Periosteal Osteosarcoma is a rare form of osteosarcoma and occurs most commonly in the diaphysis of long bones (femur and tibia). The radiographic appearance is fairly constant; a sunburst type lesion rests on a saucerized cortical depression. Staging system of the Musculoskelatal Tumor Society (Enneking System)Stage Description I-A Low grade, intracompartmental, no metastasesI-B Low grade, extracompartmental, no metastasesII-A High grade, intracompartmental, no metastasesII-B High grade, extracompartmental, no metastasesIII-A Any grade, intracompartmental, with metastasesIII-B Any grade, extracompartmental, with metastases Figure 25a demonstrates an intramedullary lesion, Figures 25b-c demonstrate a large lesion extending into the soft tissue, and figure 25e demonstrates lung metastasis. With mets it must be a grade III and the radiographic appearance is typical for osteosarcoma.

Page 277: Oncology oite-review-2012

Question 71 A 12 year old girl has a Ewing’s sarcoma of the proximal fibula with no metastatic disease or neurovascular involvment. Treatment should include   1. radiation therapy. 2. chemotherapy. 3. surgical resection. 4. radiation therapy and surgical resection. 5. chemotherapy and surgical resection.

Page 278: Oncology oite-review-2012

Answer: 5

Ewings sarcoma is the second most common primary bone tumor in children. It usually occurs in the first or second decades of life. Radiographically it has a highly aggressive appearance with permeative bone destruction occurring in the diaphysis of long bones. It may also occur, however, in flat bones. Periosteal reaction and soft tissue mass without calcifications are common. The periosteal reaction may have a sunburst or onion skin appearance. Radiographic differential diagnosis includes histiocytosis X, osteomyelitis, osteosarcoma, and medullary neuroblastoma.

Historically, the 5 year survival rate of the this tumor was found to be about 5-10%. In the past, the use of 5000-6000cGY was utilized to treat the entire bone that was affected. This provided good local control of the disease, but it did not prevent pulmonary metastasis. Also, the malice effects of radiation ie. skin burns, pathologic fractures, and damages to growth plates were noted to occur. Currently, several large series have shown that 5 year survival rates of 75-80% can be achieved with preoperative chemotherapy to shrink the lesion followed by wide excision of the mass. Occasionally, post-operative radiation may be performed, but the indications are still unclear.

Thus, answers 1, 2, and 3 are incomplete, answer 4 is an old therapeutic regimen, and answer 5 is the most correct answer for the question.

Page 279: Oncology oite-review-2012

Question 42 - Which of the following conditions has the highest rate of malignant change?  1. Ollier’s disease 2. Enchondromatosis 3. Maffucci’s syndrome 4. Multiple exostoses 5. Solitary osteochondroma

Page 280: Oncology oite-review-2012

Answer: 3  Enchondromatosis is known as Ollier’s disease, which carries a 25% chance of malignant transformation most commonly chondrosarcoma. A solitary exostosis carries a less than 1% chance of malignant transformation while multiple can be up to 25%. Maffucci’s syndrome is by definition multiple enchondromas with hemangiomas has a near 100% chance of malignant transformation, therefore is the correct response. 

Page 281: Oncology oite-review-2012

Question 1 - A 55-year-old woman reports a spontaneous onset of severe pain in her ribs. AP and lateral chest radiographs show severe osteopenia, two rib fractures, and three vertebral compression fractures. Laboratory studies show a hemoglobin level of 9.0g/dL and a monoclonal spike on serum protein electrophoresis. Which of the following imaging studies would be most helpful in establishing the diagnosis?  1. Skeletal survey

2. Technical bone scan3. Bone density determination4. MRI scan of the thoracic spine5. CT scan of the chest and abdomen

OITE 1997

Page 282: Oncology oite-review-2012

Answer: 1

The inital presentation of this patient suggests either mutiple myeloma (MM) or metastatic CA (insidious pain onset, fractures and fracture locations). The laboratory shows an anemia which can be consistent with both MM and metastatic CA (but is found in 62% of MM patients). The monoclonal spike on serum electrophoresis according to RA Kyle ("Multiple Myeloma: Review of 869 Cases"), is present in 76% of MM patients. Kyle also found 79% of MM patients show skeletal radiographic abnormalities.

The characteristic radiographic lesions of MM are punched-out lytic areas without osteoblastic changes, which may be sharply circumscribed. The vertebrae, skull, thoracic cage, pelvis, and proximal portions of humerus and femur are the most common sites of involvement of MM. The vertebral pedicles are rarely involved in myeloma, whereas they are frequently the site of metastatic CA.

"In our [the author's] experience, diphosphonate bone scans are inferior to conventional roentgenography (for MM)". "...in cases of carcinoma... the scan is superior". "Multiple osseous lesions will be demonstrated on the radiographic skeletal survey in 90% of patients... and will help determine [biopsy sites] (to diagnose MM vs. metastatic CA) " . "...radionuclide scans are usually considerably more sensitive than conventional radiographs for the early detection of skeletal metastases," according to Sim and Frassica. ("Metastatic Bone Disease and Myeloma" in Evarts, CM: Surgery of the Musculoskeletal System, ed. 2 p. 5022).

Since the preliminary diagnosis favors MM, the better response is skeletal survey to determine biopsy sites and not the bone scan.  

Page 283: Oncology oite-review-2012

Question 3-

What is the most common metastatic carcinoma to the hand?  1. Lung

2. Renal3. Breast4. Thyroid5. Prostate

Page 284: Oncology oite-review-2012

Answer: 1

Kann, SE, et al (in Instructional Course Lectures 46) note "40% of metastases to the hand (55 of 129 in four series) arose from a primary lung tumor. The other site of primary lesions are the breast, kidney, colon, thyroid, head and neck, and prostate."

Page 285: Oncology oite-review-2012

Question 4 An otherwise healthy 45-year old man has an intraosseous low-grade chondrosarcoma of the distal femur with no dedifferentiation or metastatic disease. Treatment should consist of  1. surgical resection only.

2. radiation therapy only.3. radiation therapy and surgical resection.4. chemotherapy only.5. chemotherapy and surgical resection.

Page 286: Oncology oite-review-2012

Answer: 1 

"The rate of DNA synthesis in even high-grade [chondrosarcomas] is very low...Thus, radiation or chemotherapeutic agents that destroy primary tumors by interfering with DNA synthesis would be predicted to have limited effect." (Mankin et al "Chondrosarcoma of Bone" in Evarts, CM: Surgery of the Musculoskeletal System, ed. 2)

Bauer, et al, (in Acta Orthop. Scand. 1995; 66(3): 283-288) analyzed 40 low grade chondrosarcomas of the extremities. Their ten year local recurrence rate was 0.09 and no metastases were found in 23 patients treated with intralesional curettage alone. The authors feel their results imply treatment with limited surgery would be beneficial compared with the morbidity associated with en bloc resection and reconstruction. 

Page 287: Oncology oite-review-2012

Question 114Which of the following tumors has the greatest potential to metastasize to the lung?

• 1- Osteoblastoma• 2- Enostosis• 3- Desmoplastic fibroma• 4- Giant cell tumor• 5- Enchondroma

OITE 2006--Ortho Diseases

Page 288: Oncology oite-review-2012

Question 114Which of the following tumors has the greatest potential to metastasize to the lung?

• 1- Osteoblastoma• 2- Enostosis• 3- Desmoplastic fibroma• 4- Giant cell tumor• 5- Enchondroma

OITE 2006--Ortho Diseases

Page 289: Oncology oite-review-2012

42. Which of the following bone tumors contains epithelial cells in addition to mesenchymal cells?

1. Osteosarcoma

2. Adamantinoma

3. Conventional chondrosarcoma

4. Osteoma

5. Giant cell tumor

Page 290: Oncology oite-review-2012

42. Which of the following bone tumors contains epithelial cells in addition to mesenchymal cells?

1. Osteosarcoma

2. Adamantinoma

3. Conventional chondrosarcoma

4. Osteoma

5. Giant cell tumor

Page 291: Oncology oite-review-2012

Adamantinoma

• Pathology: Basic histology consists of nests of epithelial or epitheliod cells surrounded by a fibrous stroma (see image). The epithelial component can take several different patterns (nests, islands, cords, or tubules). The most common pattern is a basaloid pattern – nests of epithelial cells with peripheral palisading. The epithelial cells are positive for cytokeratin, while the stromal cells are postitive for vimentin.

Desai SS, Jambhekar N, Agarwal M, et al: Adamantinoma of tibia: A study of 12 cases. J Surg Oncol 2006;93:429-433.

Page 292: Oncology oite-review-2012

89) Increased telemorase activity in chondrosarcoma, as determined by immunohistochemistry, has been shown to directly correlate with

1) Decreased tumor grade2) The rate of recurrence3) Technetium activity on bone scan4) Gadolinium uptake on MRI5) Tumor senescence.

Page 293: Oncology oite-review-2012

1) Decreased tumor grade

2) The rate of recurrence

3) Technetium activity on bone scan

4) Gadolinium uptake on MRI

5) Tumor senescence

Page 294: Oncology oite-review-2012

Martin JA, DeYoung BR, Gitelis S, et al. Telemorase reverse transcriptase subunit expression is associated with chondrosarcoma malignancy. Clin Orthop Rleat Res 2004; 426: 117-124.

-cartilage neoplasias include malignant chondrosarcomas and benign enchondromas. Aggressive chondrosarcomas are often difficult to distinguish from less aggressive ones, or benign enchondromas. They all express aggrecan rich type II collagen reinforced ECM that resembles normal cartilage architecture. Therefore the need to develop malignancy specific markers for these tumors.

Page 295: Oncology oite-review-2012

57. A 58 yo man has had increasing stiffness and swelling in the right knee for the past 9 months. Radiographs and a biopsy specimen are shown in figures 19A-C. What is the most likely diagnosis?

1. Tumoral Calcinosis

2. Synovial Chondromatosis

3. Synovial Hemangioma

4. Synovial Sarcoma

5. Periosteal Osetosarcoma

Page 296: Oncology oite-review-2012
Page 297: Oncology oite-review-2012

57. The answer is B. Synovial Chondromatosis.

The XR shows a knee with significant nodular cartilage and bone formation.

The biopsy clearly shows balls of chondrocytes coated in synovial lining cells.

Patients usually present as this patient did with a stiff joint with swelling, usually with mechanical symptoms.

Tx: Synovectomy + removal of loose bodies.

Page 298: Oncology oite-review-2012

63. An 11 yo girl has knee pain and a limp. A radiograph and an MRI are shown in figures 20A-B respectively. A biopsy specimen is shown in Figure 20C. Routine laboratory testing will most likely show an elevation in which of the following?

1 – Alkaline Phosphatase

2 – Total Bilirubin

3 – Hematocrit

4 – Lymphocytes

5 – Alpha Fetal Protein

Page 299: Oncology oite-review-2012
Page 300: Oncology oite-review-2012

63. The answer is 1 - alkaline phosphatase.

The image shows an osteogenic sarcoma. Due to significant bony turnover, ALK PHOS will be elevated.

* Total Bilirubin – elevated with involvement of hepatobiliary system.

•Hematocrit – can be elevated during hematologic cancers.•Lymphocytes – elevated with infection/hemeonc•AFP – elevated with germ cell tumors

Page 301: Oncology oite-review-2012

121. Figure 37 shows the radiograph of a 15 yo boy who has had knee pain for the past 3 months. What is the next most appropriate step in management?

1 – Close observation with a follow up radiograph in 6 weeks

2 – Biopsy

3 – CT scan of the chest, a bone scan, and a MRI of the femur

4 – Cultures and IV antibiotics

5 – Radiofrequency ablation

Page 302: Oncology oite-review-2012
Page 303: Oncology oite-review-2012

121. The answer is C. CT chest, bone scan, and MRI of the femur.

This boy has a lesion concerning for malignancy based on size of tumor, symptoms of continued pain, and effect of lesion on bone. •Standard staging w/u includes: Plain XR of limb CT/MRI entire bone: eval compartments/bone CXR/CT Chest: Eval for pulmonary mets Bone Scans: Eval for other bony lesions Lab tests: tailored for eval of other system involvement including LFTs, CBC/diff, ESR/CRP, Ca/Phos, Serum & Urine Electrophoresis, UA, peripheral bloodsmear, possible thyroid tests/mammagrams, poss CT abdomen/renal US, PSA when searching for primary tumor.

Page 304: Oncology oite-review-2012

OITE 2007 #186

• A 36-year-old woman has had knee pain for the past 4 months. Radiographs, MRI scans, and a biopsy specimen are shown in Figures 71a through 71e. What is the most likely diagnosis?– 1 Giant cell tumor of bone– 2 Pigmented villonodular synovitis– 3 Chondroblastoma– 4 Rheumatoid Nodule– 5 Osteomyelitis

Page 305: Oncology oite-review-2012

OITE 2007 #186

• Figures 71a through 71e

Page 306: Oncology oite-review-2012

OITE 2007 #186

• A 36-year-old woman has had knee pain for the past 4 months. Radiographs, MRI scans, and a biopsy specimen are shown in Figures 71a through 71e. What is the most likely diagnosis?– 1 Giant cell tumor of bone– 2 Pigmented villonodular synovitis– 3 Chondroblastoma– 4 Rheumatoid Nodule– 5 Osteomyelitis

Page 307: Oncology oite-review-2012

OITE 2007 #197

• Figure 74a and 74b show the radiograph and MRI scans of a 27-year-old woman who has had a 2-moth history of progressive left knee pain. An open biopsy yields the specimen shown in Figures 74c. What is the most appropriate treatment?– 1 Chemotherapy followed by proximal tibia resection with

prosthetic reconstruction– 2 Proximal tibia resection with prosthetic reconstruction– 3 Observation– 4 External beam radiation therapy– 5 Intralesional curettage with local adjuvant

Page 308: Oncology oite-review-2012

OITE 2007 #197

• Figure 74a, 74b & 74c

Page 309: Oncology oite-review-2012

OITE 2007 #197

• Figure 74a and 74b show the radiograph and MRI scans of a 27-year-old woman who has had a 2-moth history of progressive left knee pain. An open biopsy yields the specimen shown in Figures 74c. What is the most appropriate treatment?– 1 Chemotherapy followed by proximal tibia resection with

prosthetic reconstruction– 2 Proximal tibia resection with prosthetic reconstruction– 3 Observation– 4 External beam radiation therapy– 5 Intralesional curettage with local adjuvant

Page 310: Oncology oite-review-2012

OITE 2007 #203

• A 60-year-old woman has an enlarging painless mass in the upper thigh. A radiograph and T1 and T2 weight MRI scans are shown in Figures 75a – c. A biopsy specimen is in 75d. Staging studies do no reveal any evidence of metastatic disease. What is the next most appropriate step in management?– 1 Chemotherapy– 2 Radiation Therapy– 3 Mammography– 4 Embolization– 5 Surgery

Page 311: Oncology oite-review-2012

OITE 2007 #203

• Figures 75a – d

Page 312: Oncology oite-review-2012

OITE 2007 #203

• A 60-year-old woman has an enlarging painless mass in the upper thigh. A radiograph and T1 and T2 weight MRI scans are shown in Figures 75a – c. A biopsy specimen is in 75d. Staging studies do no reveal any evidence of metastatic disease. What is the next most appropriate step in management?– 1 Chemotherapy– 2 Radiation Therapy– 3 Mammography– 4 Embolization– 5 Surgery

Page 313: Oncology oite-review-2012

OITE 2007 #203• Chondrosarcoma is the second most common

primary malignant bone tumor• Pathology large atypical chondrocytes• “Although patients who have a high-grade lesion are

considered to be at high risk for metastasis, chondrosarcomas generally are treated with operative resection. Adjuvant radiation and chemotherapy have been reserved for patients who have a mesenchymal chondrosarcoma or a dedifferentiated chondrosarcoma or for those who have had inadequate operative treatment.”

• Lee FY. Mankin HJ. Fondren G. Gebhardt MC. Springfield DS. Rosenberg AE. Jennings LC. Chondrosarcoma of bone: an assessment of outcome. JBJS AM. 81(3):326-38, 1999 Mar.

• Rex A. W. Marco, Steven Gitelis, Gregory T. Brebach, and John H. Healey. Cartilage Tumors: Evaluation and TreatmentJAAOS, September/October 2000; 8: 292 - 304.

Page 314: Oncology oite-review-2012

OITE 2007 #208• A 62-year-old man with a history of nonmetastatic thyroid

carcinoma has right hip pain. Radiographs and a CT scan are shown in figure 76a – c. He undergoes placement of a locked IM rod to prevent pathologic fracture. A biopsy specimen is shown in Figure 76d. Staging reveals that this is an isolated lesion. Definitive management should definitely include which of the following?– 1 Iodine 121– 2 Radiation therapy to the right femur– 3 Physical therapy for ROM and WBAT– 4 Brachytherapy to the right proximal femur– 5 Wide Resection

Page 315: Oncology oite-review-2012

OITE 2007 #208

• 76a – d

Page 316: Oncology oite-review-2012

OITE 2007 #208• A 62-year-old man with a history of nonmetastatic thyroid

carcinoma has right hip pain. Radiographs and a CT scan are shown in figure 76a – c. He undergoes placement of a locked IM rod to prevent pathologic fracture. A biopsy specimen is shown in Figure 76d. Staging reveals that this is an isolated lesion. Definitive management should definitely include which of the following?– 1 Iodine 121– 2 Radiation therapy to the right femur– 3 Physical therapy for ROM and WBAT– 4 Brachytherapy to the right proximal femur– 5 Wide Resection

Page 317: Oncology oite-review-2012

231.A 15-year-old boy reports pain in his posterior thigh. Radiographs and a CT scan are shown in Figures 87a through 87c. A biopsy specimen is shown in Figure 87d. What is the most likely diagnosis?

1 – osteomyelitis

2 – aneurysmal bone cyst

3 – fibrous dysplasia

4 – nonossifying fibroma

5 – malignant fibrous histiocytoma

Page 318: Oncology oite-review-2012
Page 319: Oncology oite-review-2012
Page 320: Oncology oite-review-2012

231.A 15-year-old boy reports pain in his posterior thigh. Radiographs and a CT scan are shown in Figures 87a through 87c. A biopsy specimen is shown in Figure 87d. What is the most likely diagnosis?

1 – osteomyelitis

2 – aneurysmal bone cyst

3 – fibrous dysplasia

4 – nonossifying fibroma

5 – malignant fibrous histiocytoma

Page 321: Oncology oite-review-2012

242. A 55-year-old man has an isolated femoral fracture following a motor vehicle accident. A radiograph is shown in Figure 91. He is hemodynamically stable and his pain is adequately controlled. What is the next most apporpriate step in management?

1 – antegrade intramedullary nailing of femur2 – retrograde intramedullary nailing of the femur3 – placement of a distal femoral traction pin4 – wide resection5 – biospy and staging

Page 322: Oncology oite-review-2012
Page 323: Oncology oite-review-2012

242. A 55-year-old man has an isolated femoral fracture following a motor vehicle accident. A radiograph is shown in Figure 91. He is hemodynamically stable and his pain is adequately controlled. What is the next most apporpriate step in management?

1 – antegrade intramedullary nailing of femur2 – retrograde intramedullary nailing of the femur3 – placement of a distal femoral traction pin4 – wide resection5 – biospy and staging

Page 324: Oncology oite-review-2012

254.A 48-year-old man reports a mass in his thigh. He denies any history of significant trauma. MRI scans are shown in Figures 96a through 96c. What is the most likely diagnosis?

1 – round cell liposarcoma 2 – dedifferentiated liposarcoma 3 – synovial sarcoma 4 – intramuscular lipoma 5 – hematoma

Page 325: Oncology oite-review-2012
Page 326: Oncology oite-review-2012
Page 327: Oncology oite-review-2012

254.A 48-year-old man reports a mass in his thigh. He denies any history of significant trauma. MRI scans are shown in Figures 96a through 96c. What is the most likely diagnosis?

1 – round cell liposarcoma 2 – dedifferentiated liposarcoma 3 – synovial sarcoma 4 – intramuscular lipoma 5 – hematoma

Page 328: Oncology oite-review-2012

273.Chemotherapy has been shown to improve survival for all of the following malignancies EXCEPT

1 – osteosarcoma

2 – Ewing’s sarcoma

3 – chondrosarcoma

4 – lymphoma of bone

5 – rhabdomyosarcoma

Page 329: Oncology oite-review-2012

273.Chemotherapy has been shown to improve survival for all of the following malignancies EXCEPT

1 – osteosarcoma

2 – Ewing’s sarcoma

3 – chondrosarcoma

4 – lymphoma of bone

5 – rhabdomyosarcoma

Page 330: Oncology oite-review-2012

OITE 2006 – Ortho Diseases

7. A 12-year-old patient has an epiphyseal lesion. What is the most likely diagnosis?

1- Giant cell tumor

2- Aneurysmal bone cyst

3- Eosinophilic granuloma

4- Dysplasia epiphysealis hemimelica

5- Chondroblastoma

Page 331: Oncology oite-review-2012

OITE 2006 – Ortho Diseases

7. A 12-year-old patient has an epiphyseal lesion. What is the most likely diagnosis?

1- Giant cell tumor

2- Aneurysmal bone cyst

3- Eosinophilic granuloma

4- Dysplasia epiphysealis hemimelica

5- Chondroblastoma

Page 332: Oncology oite-review-2012

OITE 2006 – Ortho Diseases

23. An 80-year-old woman reports the sudden development of pain in the left distal thigh. She denies any history of trauma. Figures 9a through 9d show radiographs, a bone scan, and a biopsy specimen. What is the likely diagnosis?

1- Enchondroma

2- Osteosarcoma

3- Dedifferentiated chondrosarcoma

4- Metastatic breast carcinoma

5- Paget’s disease

Page 333: Oncology oite-review-2012

OITE 2006 – Ortho Diseases

Page 334: Oncology oite-review-2012

OITE 2006 – Ortho Diseases

Page 335: Oncology oite-review-2012

OITE 2006 – Ortho Diseases

23. An 80-year-old woman reports the sudden development of pain in the left distal thigh. She denies any history of trauma. Figures 9a through 9d show radiographs, a bone scan, and a biopsy specimen. What is the likely diagnosis?

1- Enchondroma

2- Osteosarcoma

3- Dedifferentiated chondrosarcoma

4- Metastatic breast carcinoma

5- Paget’s disease

Page 336: Oncology oite-review-2012

OITE 2006 – Ortho Diseases

23. An 80-year-old woman reports the sudden development of pain in the left distal thigh. She denies any history of trauma. Figures 9a through 9d show radiographs, a bone scan, and a biopsy specimen. What is the likely diagnosis?

1- Enchondroma

2- Osteosarcoma

3- Dedifferentiated chondrosarcoma

4- Metastatic breast carcinoma

5- Paget’s disease

Page 337: Oncology oite-review-2012

OITE 2006 Orthopedic Diseases

78. A 14 year old girl reports right thigh discomfort with activities. An AP radiograph is shown in figure 24a, and coronal T1-weighted and axial T2-weighted MRI scans are shown if figures 24b and 24c, respectively. A biopsy specimen is shown in Figure 24d. What is the treatment of choice for this tumor?

1- Surgery only2- Surgery and chemotherapy3- Surgery and radiation therapy

4- Chemotherapy only5- Chemotherapy and radiation therapy

Page 338: Oncology oite-review-2012

OITE 2006 Orthopedic Diseases

Page 339: Oncology oite-review-2012

OITE 2006 Orthopedic Diseases

78. A 14 year old girl reports right thigh discomfort with activities. An AP radiograph is shown in figure 24a, and coronal T1-weighted and axial T2-weighted MRI scans are shown if figures 24b and 24c, respectively. A biopsy specimen is shown in Figure 24d. What is the treatment of choice for this tumor?

1- Surgery only2- Surgery and chemotherapy3- Surgery and radiation therapy

4- Chemotherapy only5- Chemotherapy and radiation therapy

Page 340: Oncology oite-review-2012

OITE 2006 Orthopedic Diseases

79. A 25 year old man reports progressive wrist pain and swelling. An AP radiograph and biopsy specimen are shown in Figures 25a and 25b. Management should consist of:

1- below-elbow amputation2- bisphosphonate treatment3- nonsteroidal anti-inflammatory drugs4- chemotherapy followed by wide surgical excision5- intralesional resection and reconstruction

Page 341: Oncology oite-review-2012

OITE 2006 Orthopedic Diseases

Page 342: Oncology oite-review-2012

OITE 2006 Orthopedic Diseases

79. A 25 year old man reports progressive wrist pain and swelling. An AP radiograph and biopsy specimen are shown in Figures 25a and 25b. Management should consist of:

1- below-elbow amputation2- bisphosphonate treatment3- nonsteroidal anti-inflammatory drugs4- chemotherapy followed by wide surgical excision5- intralesional resection and reconstruction

Page 343: Oncology oite-review-2012

OITE 2006 Orthopedic Diseases

84. A 57 year old man was diagnosed with localized prostate carcinoma 3 years ago, with negative margins and negative lymph nodes. He now reports a 3-week history of severe right hip pain that is worse with weight-bearing. Radiographs are shown in Figures 26a and 26b. CT scans of the chest, abdomen, and pelvis, as well as a whole body bone scan, are negative for other lesions. What is the next most appropriate step for treatment?

1- Right hip hemiarthroplasty2- Locked IM rod placement in right femur3- Biopsy of the right femur4- Hip disarticulation5- Curettage, cementation, and plate fixation

Page 344: Oncology oite-review-2012

OITE 2006 Orthopedic Diseases

Page 345: Oncology oite-review-2012

OITE 2006 Orthopedic Diseases

84. A 57 year old man was diagnosed with localized prostate carcinoma 3 years ago, with negative margins and negative lymph nodes. He now reports a 3-week history of severe right hip pain that is worse with weight-bearing. Radiographs are shown in Figures 26a and 26b. CT scans of the chest, abdomen, and pelvis, as well as a whole body bone scan, are negative for other lesions. What is the next most appropriate step for treatment?

1- Right hip hemiarthroplasty2- Locked IM rod placement in right femur3- Biopsy of the right femur4- Hip disarticulation5- Curettage, cementation, and plate fixation

Page 346: Oncology oite-review-2012

OITE 2006 Orthopedic Diseases

96. A 12 year old sustained a knee injury in a fall off his bike. The patient reports that his knee pain improved the day after injury. Radiographs shown in Figures 30a and 30b reveal a lesion in the distal femur. Management should consist of

1- open biopsy and intralesional excision2- CT-guided needle biopsy and anti-microbial therapy3- curettage and autogenous iliac crest bone grafting4- anti-inflammatory drugs as needed5- observation and follow up

Page 347: Oncology oite-review-2012

OITE 2006 Orthopedic Diseases

Page 348: Oncology oite-review-2012

OITE 2006 Orthopedic Diseases

96. A 12 year old sustained a knee injury in a fall off his bike. The patient reports that his knee pain improved the day after injury. Radiographs shown in Figures 30a and 30b reveal a lesion in the distal femur. Management should consist of

1- open biopsy and intralesional excision2- CT-guided needle biopsy and anti-microbial therapy3- curettage and autogenous iliac crest bone grafting4- anti-inflammatory drugs as needed5- observation and follow up

Page 349: Oncology oite-review-2012

OITE 2006 Orthopedic Diseases

109. A 72 year old man with a history of smoking 40 packs of cigarettes per year underwent a successful left total hip replacement 10 years ago. He now reports a 2 month history of progressive right hip pain. An AP pelvic radiograph and CT scan are shown in Figures 35a and 35b. What is the next most appropriate step in management?

1- Immediate cemented right total hip arthroplasty2- Open reduction and internal fixation of the acetabular fracture3- Activity modification, IV bisphosphonates, and follow up in 6 weeks4- Technetium Tc 99m scan and CT of the chest, abdomen, and pelvis5- Radiation therapy

Page 350: Oncology oite-review-2012

OITE 2006 Orthopedic Diseases

Page 351: Oncology oite-review-2012

OITE 2006 Orthopedic Diseases

109. A 72 year old man with a history of smoking 40 packs of cigarettes per year underwent a successful left total hip replacement 10 years ago. He now reports a 2 month history of progressive right hip pain. An AP pelvic radiograph and CT scan are shown in Figures 35a and 35b. What is the next most appropriate step in management?

1- Immediate cemented right total hip arthroplasty2- Open reduction and internal fixation of the acetabular fracture3- Activity modification, IV bisphosphonates, and follow up in 6 weeks4- Technetium Tc 99m scan and CT of the chest, abdomen, and pelvis5- Radiation therapy

Page 352: Oncology oite-review-2012

Question 119:A 25 year-old man has had intermittent swelling in the left thenar eminence for the past few months. He describes the onset of a dull ache associated with swelling. An AP radiograph is shown in Figure 39. What is the most likely diagnosis?

• 1- Lipoma• 2- Hemangioma• 3- Synovial sarcoma• 4- Ewing’s family of tumor• 5- Giant cell tumor of the tendon sheath

OITE 2006--Ortho Diseases

Page 353: Oncology oite-review-2012

Question 119: Figure 39

OITE 2006--Ortho Diseases

Page 354: Oncology oite-review-2012

Question 119:A 25 year-old man has had intermittent swelling in the left thenar eminence for the past few months. He describes the onset of a dull ache associated with swelling. An AP radiograph is shown in Figure 39. What is the most likely diagnosis?

• 1- Lipoma• 2- Hemangioma• 3- Synovial sarcoma• 4- Ewing’s family of tumor• 5- Giant cell tumor of the tendon sheath

OITE 2006--Ortho Diseases

Page 355: Oncology oite-review-2012

Question 137Which of the following diseases of bone (when nonmetastatic at diagnosis) carries the worst prognosis for 5-year survival)?

• 1- Lymphoma• 2- Osteosarcoma• 3- Ewing’s sarcoma• 4- Paget’s sarcoma• 5- Conventional chondrosarcoma

OITE 2006--Ortho Diseases

Page 356: Oncology oite-review-2012

Question 137Which of the following diseases of bone (when nonmetastatic at diagnosis) carries the worst prognosis for 5-year survival)?

• 1- Lymphoma• 2- Osteosarcoma• 3- Ewing’s sarcoma• 4- Paget’s sarcoma• 5- Conventional chondrosarcoma

OITE 2006--Ortho Diseases

Page 357: Oncology oite-review-2012

Question 147:A 2-year-old boy has a limp and anterior bowing of the leg. A lateral radiograph is shown in Figure 52. What is the most likely diagnosis?

• 1- Unicameral bone cyst• 2- Osteofibrous dysplasia• 3- Adamantinoma• 4- Ewing’s sarcoma• 5- Metastatic neuroblastoma

OITE 2006--Ortho Diseases

Page 358: Oncology oite-review-2012

Question 147: Figure 52OITE 2006--Ortho Diseases

Page 359: Oncology oite-review-2012

Question 147:A 2-year-old boy has a limp and anterior bowing of the leg. A lateral radiograph is shown in Figure 52. What is the most likely diagnosis?

• 1- Unicameral bone cyst• 2- Osteofibrous dysplasia• 3- Adamantinoma• 4- Ewing’s sarcoma• 5- Metastatic neuroblastoma

OITE 2006--Ortho Diseases

Page 360: Oncology oite-review-2012

Question 161: A 16 year-old girl has had vague right hip pain for the past 2 months. An AP radiograph of the pelvis and a T1 fat suppression, gadolinium-enhanced MRI are shown in Figures 62a and 62b. A biopsy specimen is shown in Figure 62c. What characteristic genetic translocation is associated with this disease?

• 1- t(X;18)• 2- t(2;13)• 3- t(9;22)• 4- t(11;22)• 5- t(12;16)

OITE 2006--Ortho Diseases

Page 361: Oncology oite-review-2012

Question 161: Figure 62aOITE 2006--Ortho Diseases

Page 362: Oncology oite-review-2012

Question 161: Figure 62bOITE 2006--Ortho Diseases

Page 363: Oncology oite-review-2012

Question 161: Figure 62cOITE 2006--Ortho Diseases

Page 364: Oncology oite-review-2012

Question 161: A 16 year-old girl has had vague right hip pain for the past 2 months. An AP radiograph of the pelvis and a T1 fat suppression, gadolinium-enhanced MRI are shown in Figures 62a and 62b. A biopsy specimen is shown in Figure 62c. What characteristic genetic translocation is associated with this disease?

• 1- t(X;18)• 2- t(2;13)• 3- t(9;22)• 4- t(11;22)• 5- t(12;16)

OITE 2006--Ortho Diseases

Page 365: Oncology oite-review-2012

Question 16 Figure 4a shows a pigmented lesion on the right side of the neck of a 41-year-old man. The patient's history reveals that he had multiple bone lesions during childhood and juvenile-onset diabetes mellitus. Figure 4b and 4c show radiographs of his knee and leg. What is the most likely diagnosis?  1. Ollier’s disease 2. Neurofibromatosis 3. McCune-Albright syndrome 4. Multiple hereditary exostoses 5. Multiple nonossifying fibromas

Page 366: Oncology oite-review-2012
Page 367: Oncology oite-review-2012
Page 368: Oncology oite-review-2012

Answer: 3  The classic triad seen with McCune-Albright Syndrome is precocious puberty, polyostotic fibrous dysplasia (seen in fig 4B and 4C) and café au lait spots (seen in fig 4A). The fibrous dysplasia is metaphyseal or diaphyseal and associated bowing is common. Ollier's disease is associated with multiple enchondromatoses, but does not include café au lait spots. Neurofibromatosis is associated with café au lait spots, but they are generally on the trunk, pelvis and flexor creases of elbows and knees. Multiple cutaneous fleshy tumors are also frequently seen by late childhood. Source:Campbell's Operative Orthopedics Atlas of Orthopedic Pathology Merck Manual

Page 369: Oncology oite-review-2012

 Question 23 - A 10-year old boy has had intermittent pain in his right groin and proximal thigh for the past 6 months. Fingures 6a and 6b show plain radographs of the hip. Figure 6c shows an axial proton density MRI scan through the lesion, and Figure 6d shows representative tissue biopsy specimens at low power. What is the most likely diagnosis?  1. Chondroblastoma 2. Ewing’s sarcoma 3. Giant cell tumor 4. Simple bone cyst 5. Aneurysmal bone cyst

Page 370: Oncology oite-review-2012
Page 371: Oncology oite-review-2012
Page 372: Oncology oite-review-2012
Page 373: Oncology oite-review-2012

Answer: 5 Clinical Symptoms- pain is short in duration B. Radiographic Features- ballooned or cystic expansion of the bone no significant matrix the lesion affects the metaphysis of long bones no significant mineralizationMRI- well defined lesion often with lobulated contour and internal septation with multiple fluid levels. C. Histology- hemosiderin laden macrophages, multinucleated giant cells, fibrous stroma, and woven bone. Kransdorf MS., and Berquist TH: Musculoskeletal Neoplasms; MRI of The Musculoskeletal System 1996. pp 822-836. Springfield DS: Bone and Soft Tissue Tumors; Lovell and Winter’s Pediatric Orthpaedics 1996. pp50-51. Wold LE., McLeod RA., Sim FH., and Unni KK: Atlas Of Orthopedic Pathology 1990. pp232-237

Page 374: Oncology oite-review-2012

Question 67 A 12-year-old girl has had painful, unilateral toe walking for the past 12 months. Examination shows that her foot is fixed in equinus, and she has exquisite point tenderness over the proximal and medial aspect of the medial gastrocnemius muscle. A lateral radiograph of the knee is shown in figure 16a, and a T2-weighted axial MRI scan of the proximal leg is shown in figure 16b. A photomicrograph of biopsy material is shown in Figure 16c. What is the most likely diagnosis?  1. Rhabdomyosarcoma 2. Nodular Fascitis 3. Heterotopic Ossification 4. Soft-tissue hemangioma 5. Soft-tissue Ewing’s sarcoma

Page 375: Oncology oite-review-2012
Page 376: Oncology oite-review-2012
Page 377: Oncology oite-review-2012
Page 378: Oncology oite-review-2012

Answer: 4

Soft tissue hemangioma are common and many types are present at birth. Can occur deep in skeletal muscle and other soft tissue of the extremities. Tend to be bluish in color. Cavernous types frequently are visible on plain x-rays because of areas of calcifications that can be described as smoke rings. These are felt to be pathognomonic (Enneking). Biopsy shows vascular components that may difficult to tell from normal vascular tissues. MRI shows a heterogeneous lesion with numerous small blood vessels and fatty infiltrate. Nonoperative measures are used if can control discomfort. Operative treatment is wide resection, however local recurrence is high (Miller). Rhabdomyosarcoma is a malignant tumor of skeletal muscle, and is more common in children and adolescents. Also involved in high rate of lymph node metastasis. Treatment includes wide resection, radiotherapy and prolonged systemic polychemotherapy. Nodular fascitis consists of proliferating fibroblasts in myoid stroma with predominant vascular pattern. Occurs most frequently in forearm of young adults with deep fascia involved. Heterotopic ossification is usually from repetitive trauma-most commonly over diaphyseal long bones. Usually not attached to underlying bone. There is a zonal pattern with trabecular bone at peripherally and immature tissue at center. Ewing’s is rapid growing painful mass that tender on exam. Usually begins in central portion of bone and rapidly extends proximal and distal along canal and through vascular perforations. Bone is rapidly resorbed and as characteristic periosteal reaction (Codman’s triangle). (Enneking, Miller) Enzinger F, Weiss S (eds): Soft Tissue Tumors, ed 3. St Louis, MO, CV Mosby, 1995, pp605-609.

Page 379: Oncology oite-review-2012

Question 80 Figures 22a and 22b show plain xrays of a 33-year-old man who has had progressive pain in his nondominant left shoulder for the past 5 months. A proton density MRI is shown in Fig. 22c, and histiologic materials from the solid portion of the lesion are shown in Figures 22d and 22e. What is the most likely diagnosis?  1. Enchondroma 2. Giant cell tumor 3. Chondroblastoma 4. Chondromyxoid fibroma 5. Clear cell chondrosarcoma

Page 380: Oncology oite-review-2012
Page 381: Oncology oite-review-2012
Page 382: Oncology oite-review-2012
Page 383: Oncology oite-review-2012
Page 384: Oncology oite-review-2012
Page 385: Oncology oite-review-2012

Answer: 5 Ref.: Unni KK; Dahlin’s Bone Tumors 1996, pp. 71-108 -males 70%, females 30%-peak age; third decade-most common location: the proximal femur accounts for approx. 50%. nearly always

involves the epiphyseal region of a long bone.-clinical symptoms: pain is the most common at presentation-X-ray fractures: lesion has epiphyseal location in prox. femur or humerus 

Page 386: Oncology oite-review-2012

Question 165 - Figures 44a and 44b show the plain radiographs of a 12-year-old boy who has had left medial knee pain for the past 4 months. Figure 44c shows representative histologic material. What is the most likely diagnosis?  1. enchondroma 2. osteoblastoma 3. giant cell tumor 4. chondroblastoma 5. osteochondritis dessicans

Page 387: Oncology oite-review-2012
Page 388: Oncology oite-review-2012
Page 389: Oncology oite-review-2012
Page 390: Oncology oite-review-2012

Answer: 4 

Chondroblastoma is a lytic lesion commonly located eccentrically in the epiphysis of long bones, most often in skeletally immature individuals. The most common sites of origin are the proximal humerus, proximal and distal femur, and proximal tibia. The radiographic differential includes giant cell tumor crossing into the epiphysis. In this case however, the characteristic highly cellular chondral oval mononuclear cells seen on the histologic material confirm the diagnosis. Turcotte RE, Kurt AM, Sim FH, et al. Chondroblastoma. Human Pathology 1993; 24: 944-949.

Page 391: Oncology oite-review-2012

Question 219 - A 35-year old man has had increasing pain in the knee for the past 4 months. An AP radiograph of the knee is shown in Figure 58a, and low- and high-power photomicrographs of the biopsy specimen are shown in Figures 58b and 58c. What is the most likely diagnosis?  1. Osteosarcoma 2. Chondroblastoma 3. Giant Cell tumor 4. Aneurysmal bone cyst 5. Desmoplastic fibroma

Page 392: Oncology oite-review-2012
Page 393: Oncology oite-review-2012
Page 394: Oncology oite-review-2012
Page 395: Oncology oite-review-2012

Answer: 3 50% of Giant cell tumors usually occur about the knee, with 28% involving distal femur. It is a lytic lesion with periosteal reaction. Histologically giant cells scattered uniformly, and the nuclei of the mononuclear and giant cells are similar in appearance. Most common age is 3rd decade. In this case the radiographic finding are similar to osteosarcoma and Desmoplastic fibroma as well as Giant cell tumor, however, histologic study singles out Giant cell tumor.

(Atlas of orthopedic surgery)

Page 396: Oncology oite-review-2012

Question 222 - Figures 60a and 60b show the radiographs of the ankle and distal leg of an 11-year-old girl after she twisted her ankle while playing soccer. She has no history of ankle or leg pain. Examination reveals localized swelling and tenderness over the lateral ankle, and the tibia is not tender. The bone lesion identified in the tibia most likely is   1. osteoblastoma 2. osteoid osteoma 3. ossifying fibroma 4. fibrous dysplasia 5. nonossifying fibroma

Page 397: Oncology oite-review-2012
Page 398: Oncology oite-review-2012

Answer: 5 DISCUSSION: Nonossifying fibroma almost always develops in patients under the age of twenty and is usually discovered when a x‑ray is taken for an injury. Histologically, there is a triphasic cytological appearance, featuring oval round cells, spindle cells, and giant cells. It usually is a single well‑delineated cortical defect with circular or oval lesions and smooth, lobulated edges. Adjacent body is sclerotic without periosteal reaction. The lower extremity long tubular bones are the most common sites of occurrence. Treatment is usually not necessary, but occasionally a large lesion will require intralesional excision and bone grafting. REFERENCES: Marks KE, Bauer TW: Fibrous Tumors of Bone. Orthop Clin North Am 1989; 20(3):377‑93.

Page 399: Oncology oite-review-2012

Question 227

An 80 year-old man has had increasing hip pain and difficulty ambulating for the past 6 months. An oblique radiograph pf the hip is shown in figure 64a, and a technetium bone scan is shown in Figure 64b. Low-and high power photomicrographs are shown in Figure 64c and 64d. What is the most likely diagnosis?  1. Pagets sarcoma 2. Pagets disease 3. Fibrous dysplasia 4. Hyperparathyroidism 5. Metastatic carcinoma

Page 400: Oncology oite-review-2012
Page 401: Oncology oite-review-2012
Page 402: Oncology oite-review-2012
Page 403: Oncology oite-review-2012

Answer: 2 Paget's disease is generally discovered in the patient 50 years of age and older. This disease in more common in those of eastern and western European decent and rare in Asians, Scandinavians, and blacks. 20% of patients are asymptomatic and the diagnosis is made while being worked‑up for an unrelated complaint. Symptomatic patients complain of bone pain, skeletal deformities, changes in skin temperature, pathologic fractures and symptoms related to nerve compression. Radiographic appearance is that of focal bone resorption and formation in a disordered trabecular pattern, with enlarged cortices and overall bone size. Paget's disease appears hot on technetium bone scans. Treatment is indicated for those with increasing pain and/or deformity. Calcitonin and diphosphonates are the pharmacologic agents available. Pathologic fractures, disabling arthritis, and severe malalignment are indications for surgical intervention.

REFERENCES: Merkow RL, Lane JM: Paget's Disease of Bone. Orthop Clinics NorthAm 1990;21(1):171‑89.

Page 404: Oncology oite-review-2012

OITE 2008 – Basic Science / Onc

• 8 Figure 3a shows the radiograph of a 30-year-old woman who reports worsening left hip pain. Bone and CT scans do not reveal any other sites of disease. A biopsy specimen is shown in Figure 3b. The immunostaining reactivity for smooth muscle actin is shown in Figure 3c. What is the most likely diagnosis?

• 1- Ostegenic sarcoma• 2- Ewing's sarcoma• 3- Metastatic carcinoma• 4- Malignant fibrous histiocytoma of bone• 5- Leiomyosarcoma of bone

Page 405: Oncology oite-review-2012

Question 1 - A 55-year-old woman reports a spontaneous onset of severe pain in her ribs. AP and lateral chest radiographs show severe osteopenia, two rib fractures, and three vertebral compression fractures. Laboratory studies show a hemoglobin level of 9.0g/dL and a monoclonal spike on serum protein electrophoresis. Which of the following imaging studies would be most helpful in establishing the diagnosis?  1. Skeletal survey

2. Technical bone scan3. Bone density determination4. MRI scan of the thoracic spine5. CT scan of the chest and abdomen

OITE 1997

Page 406: Oncology oite-review-2012

Answer: 1

The inital presentation of this patient suggests either mutiple myeloma (MM) or metastatic CA (insidious pain onset, fractures and fracture locations). The laboratory shows an anemia which can be consistent with both MM and metastatic CA (but is found in 62% of MM patients). The monoclonal spike on serum electrophoresis according to RA Kyle ("Multiple Myeloma: Review of 869 Cases"), is present in 76% of MM patients. Kyle also found 79% of MM patients show skeletal radiographic abnormalities.

The characteristic radiographic lesions of MM are punched-out lytic areas without osteoblastic changes, which may be sharply circumscribed. The vertebrae, skull, thoracic cage, pelvis, and proximal portions of humerus and femur are the most common sites of involvement of MM. The vertebral pedicles are rarely involved in myeloma, whereas they are frequently the site of metastatic CA.

"In our [the author's] experience, diphosphonate bone scans are inferior to conventional roentgenography (for MM)". "...in cases of carcinoma... the scan is superior". "Multiple osseous lesions will be demonstrated on the radiographic skeletal survey in 90% of patients... and will help determine [biopsy sites] (to diagnose MM vs. metastatic CA) " . "...radionuclide scans are usually considerably more sensitive than conventional radiographs for the early detection of skeletal metastases," according to Sim and Frassica. ("Metastatic Bone Disease and Myeloma" in Evarts, CM: Surgery of the Musculoskeletal System, ed. 2 p. 5022).

Since the preliminary diagnosis favors MM, the better response is skeletal survey to determine biopsy sites and not the bone scan.  

Page 407: Oncology oite-review-2012

Question 3-

What is the most common metastatic carcinoma to the hand?  1. Lung

2. Renal3. Breast4. Thyroid5. Prostate

Page 408: Oncology oite-review-2012

Answer: 1

Kann, SE, et al (in Instructional Course Lectures 46) note "40% of metastases to the hand (55 of 129 in four series) arose from a primary lung tumor. The other site of primary lesions are the breast, kidney, colon, thyroid, head and neck, and prostate."

Page 409: Oncology oite-review-2012

Question 4 An otherwise healthy 45-year old man has an intraosseous low-grade chondrosarcoma of the distal femur with no dedifferentiation or metastatic disease. Treatment should consist of  1. surgical resection only.

2. radiation therapy only.3. radiation therapy and surgical resection.4. chemotherapy only.5. chemotherapy and surgical resection.

Page 410: Oncology oite-review-2012

Answer: 1 

"The rate of DNA synthesis in even high-grade [chondrosarcomas] is very low...Thus, radiation or chemotherapeutic agents that destroy primary tumors by interfering with DNA synthesis would be predicted to have limited effect." (Mankin et al "Chondrosarcoma of Bone" in Evarts, CM: Surgery of the Musculoskeletal System, ed. 2)

Bauer, et al, (in Acta Orthop. Scand. 1995; 66(3): 283-288) analyzed 40 low grade chondrosarcomas of the extremities. Their ten year local recurrence rate was 0.09 and no metastases were found in 23 patients treated with intralesional curettage alone. The authors feel their results imply treatment with limited surgery would be beneficial compared with the morbidity associated with en bloc resection and reconstruction. 

Page 411: Oncology oite-review-2012

Question 16 Figure 4a shows a pigmented lesion on the right side of the neck of a 41-year-old man. The patient's history reveals that he had multiple bone lesions during childhood and juvenile-onset diabetes mellitus. Figure 4b and 4c show radiographs of his knee and leg. What is the most likely diagnosis?  1. Ollier’s disease 2. Neurofibromatosis 3. McCune-Albright syndrome 4. Multiple hereditary exostoses 5. Multiple nonossifying fibromas

Page 412: Oncology oite-review-2012
Page 413: Oncology oite-review-2012
Page 414: Oncology oite-review-2012

Answer: 3  The classic triad seen with McCune-Albright Syndrome is precocious puberty, polyostotic fibrous dysplasia (seen in fig 4B and 4C) and café au lait spots (seen in fig 4A). The fibrous dysplasia is metaphyseal or diaphyseal and associated bowing is common. Ollier's disease is associated with multiple enchondromatoses, but does not include café au lait spots. Neurofibromatosis is associated with café au lait spots, but they are generally on the trunk, pelvis and flexor creases of elbows and knees. Multiple cutaneous fleshy tumors are also frequently seen by late childhood. Source:Campbell's Operative Orthopedics Atlas of Orthopedic Pathology Merck Manual

Page 415: Oncology oite-review-2012

Question 20 A 42-year-old health care professional has had knee pain for the past 2 months. An MRI scan of the knee reveals a large effusion with loculations and synovial thickening, and results of an open biopsy and culture are consistent with tuberculosis. Sensitivity tests show no resistance to antibiotics. Following debridement and synovectomy, appropriate antibiotic therapy should include  1. rifampin and pyridoxine. 2. rifampin and ethambutol hydrochloride. 3. isoniazid. 4. isoniazid and pyridoxine. 5. isoniazid, rifampin, pyrazinamide, and pyridoxine.

Page 416: Oncology oite-review-2012

Answer: 5 Watts and Lifeso, JBJS 1996;78A:288-298.

*Current Concepts Review discusses tuberculosis of bone and joints. The case presented shows classical findings on MRI. Successful medical treatment of tuberculosis requires the prolonged administration of a minimum of three drugs to which the organisms are susceptible, and at least one of these drugs must be bactericidal. Isoniazid is the most potent bactericidal drug available and is particularly effective against actively growing organisms. Rifampin and pyrazinamide are the most effective sterilizing drugs, and they are specifically effective against bacilli that are dormant and undergo periodic bursts of activity. Ethambutol is bacteriostatic. Streptomycin is bactericidal but must be given parenterally. Other drugs useful for multiple-drug-resistant organisms include ethionamide, cycloserine, kanamycin, capreomycin, and paraaminosalicylic acid. The current recommendation for treatment of adults with musculoskeletal tuberculosis is 300mg of isoniazid a day, 600mg of rifampin a day, and 20-30mg/kg of pyrazinamide a day. 10mg of pyridoxine a day is given as prophylaxis against possible isoniazid-induced neuropathy.

Page 417: Oncology oite-review-2012

 Question 23 - A 10-year old boy has had intermittent pain in his right groin and proximal thigh for the past 6 months. Fingures 6a and 6b show plain radographs of the hip. Figure 6c shows an axial proton density MRI scan through the lesion, and Figure 6d shows representative tissue biopsy specimens at low power. What is the most likely diagnosis?  1. Chondroblastoma 2. Ewing’s sarcoma 3. Giant cell tumor 4. Simple bone cyst 5. Aneurysmal bone cyst

Page 418: Oncology oite-review-2012
Page 419: Oncology oite-review-2012
Page 420: Oncology oite-review-2012
Page 421: Oncology oite-review-2012

Answer: 5 Clinical Symptoms- pain is short in duration B. Radiographic Features- ballooned or cystic expansion of the bone no significant matrix the lesion affects the metaphysis of long bones no significant mineralizationMRI- well defined lesion often with lobulated contour and internal septation with multiple fluid levels. C. Histology- hemosiderin laden macrophages, multinucleated giant cells, fibrous stroma, and woven bone. Kransdorf MS., and Berquist TH: Musculoskeletal Neoplasms; MRI of The Musculoskeletal System 1996. pp 822-836. Springfield DS: Bone and Soft Tissue Tumors; Lovell and Winter’s Pediatric Orthpaedics 1996. pp50-51. Wold LE., McLeod RA., Sim FH., and Unni KK: Atlas Of Orthopedic Pathology 1990. pp232-237

Page 422: Oncology oite-review-2012

Question 42 - Which of the following conditions has the highest rate of malignant change?  1. Ollier’s disease 2. Enchondromatosis 3. Maffucci’s syndrome 4. Multiple exostoses 5. Solitary osteochondroma

Page 423: Oncology oite-review-2012

Answer: 3  Enchondromatosis is known as Ollier’s disease, which carries a 25% chance of malignant transformation most commonly chondrosarcoma. A solitary exostosis carries a less than 1% chance of malignant transformation while multiple can be up to 25%. Maffucci’s syndrome is by definition multiple enchondromas with hemangiomas has a near 100% chance of malignant transformation, therefore is the correct response. 

Page 424: Oncology oite-review-2012

Question 50 - A 21 year old man has had increasing hip pain for the last 3 months that occurs primarily during weightlifting exercises. AP and oblique radiographs of his hip are shown in figures 13a and 13b. A CT scan of his hip is shown in figure 13c, and a T2-weighted coronal MRI scan is shown in fig.13d. Low and high power photomicrographs of the biopsy material are shown in 13e and 13f. What is the most likely diagnosis? 1. chondroblastoma 2. giant cell tumor 3. unicameral bone cyst 4. aneurysmal bone cyst 5. hyperparathyroidism

Page 425: Oncology oite-review-2012
Page 426: Oncology oite-review-2012
Page 427: Oncology oite-review-2012
Page 428: Oncology oite-review-2012
Page 429: Oncology oite-review-2012
Page 430: Oncology oite-review-2012
Page 431: Oncology oite-review-2012

Answer: 4 Physical exam - pain and swelling are the important features, and they vary in durationfrom weeks to a few years. Lesion tends to increase in size until therapy initiated.X-rays - a zone of rarefaction, which is usually well circumscribed and eccentric, is associated with an obvious soft-tissue extension. In the classic case, this soft tissue extension is produced by the bulging of periosteum and a resultant layer of radiographically visible new bone that delimits the periphery of the tumor. The lesional area tends to show trabeculation. Fusiform expansion may be produced when small bones such as ribs or the fibula are affected. They may grow rapidly and mimic a malignancy, especially telangiectatic osteosarcoma. Gross path - the cyst contains anastomosing cavernomatous spaces that ordinarily comprise the bulk of the lesion. The spaces are usually filled with unclotted blood; the blood may well up into, but not spurt from, the tumor when it is unroofed. The eggshell-thick layer of subperiosteal new bone, which delimits the lesion, is ordinarily discernible. With magnification, cavernous spaces that may be filled with blood are identified. The walls of the spaces contain spindled fibroblastic cells, multinucleated giant cells, and thin strands of bone.

Page 432: Oncology oite-review-2012

Question 67 A 12-year-old girl has had painful, unilateral toe walking for the past 12 months. Examination shows that her foot is fixed in equinus, and she has exquisite point tenderness over the proximal and medial aspect of the medial gastrocnemius muscle. A lateral radiograph of the knee is shown in figure 16a, and a T2-weighted axial MRI scan of the proximal leg is shown in figure 16b. A photomicrograph of biopsy material is shown in Figure 16c. What is the most likely diagnosis?  1. Rhabdomyosarcoma 2. Nodular Fascitis 3. Heterotopic Ossification 4. Soft-tissue hemangioma 5. Soft-tissue Ewing’s sarcoma

Page 433: Oncology oite-review-2012
Page 434: Oncology oite-review-2012
Page 435: Oncology oite-review-2012
Page 436: Oncology oite-review-2012

Answer: 4

Soft tissue hemangioma are common and many types are present at birth. Can occur deep in skeletal muscle and other soft tissue of the extremities. Tend to be bluish in color. Cavernous types frequently are visible on plain x-rays because of areas of calcifications that can be described as smoke rings. These are felt to be pathognomonic (Enneking). Biopsy shows vascular components that may difficult to tell from normal vascular tissues. MRI shows a heterogeneous lesion with numerous small blood vessels and fatty infiltrate. Nonoperative measures are used if can control discomfort. Operative treatment is wide resection, however local recurrence is high (Miller). Rhabdomyosarcoma is a malignant tumor of skeletal muscle, and is more common in children and adolescents. Also involved in high rate of lymph node metastasis. Treatment includes wide resection, radiotherapy and prolonged systemic polychemotherapy. Nodular fascitis consists of proliferating fibroblasts in myoid stroma with predominant vascular pattern. Occurs most frequently in forearm of young adults with deep fascia involved. Heterotopic ossification is usually from repetitive trauma-most commonly over diaphyseal long bones. Usually not attached to underlying bone. There is a zonal pattern with trabecular bone at peripherally and immature tissue at center. Ewing’s is rapid growing painful mass that tender on exam. Usually begins in central portion of bone and rapidly extends proximal and distal along canal and through vascular perforations. Bone is rapidly resorbed and as characteristic periosteal reaction (Codman’s triangle). (Enneking, Miller) Enzinger F, Weiss S (eds): Soft Tissue Tumors, ed 3. St Louis, MO, CV Mosby, 1995, pp605-609.

Page 437: Oncology oite-review-2012

Question 71 A 12 year old girl has a Ewing’s sarcoma of the proximal fibula with no metastatic disease or neurovascular involvment. Treatment should include   1. radiation therapy. 2. chemotherapy. 3. surgical resection. 4. radiation therapy and surgical resection. 5. chemotherapy and surgical resection.

Page 438: Oncology oite-review-2012

Answer: 5

Ewings sarcoma is the second most common primary bone tumor in children. It usually occurs in the first or second decades of life. Radiographically it has a highly aggressive appearance with permeative bone destruction occurring in the diaphysis of long bones. It may also occur, however, in flat bones. Periosteal reaction and soft tissue mass without calcifications are common. The periosteal reaction may have a sunburst or onion skin appearance. Radiographic differential diagnosis includes histiocytosis X, osteomyelitis, osteosarcoma, and medullary neuroblastoma.

Historically, the 5 year survival rate of the this tumor was found to be about 5-10%. In the past, the use of 5000-6000cGY was utilized to treat the entire bone that was affected. This provided good local control of the disease, but it did not prevent pulmonary metastasis. Also, the malice effects of radiation ie. skin burns, pathologic fractures, and damages to growth plates were noted to occur. Currently, several large series have shown that 5 year survival rates of 75-80% can be achieved with preoperative chemotherapy to shrink the lesion followed by wide excision of the mass. Occasionally, post-operative radiation may be performed, but the indications are still unclear.

Thus, answers 1, 2, and 3 are incomplete, answer 4 is an old therapeutic regimen, and answer 5 is the most correct answer for the question.

Page 439: Oncology oite-review-2012

Question 80 Figures 22a and 22b show plain xrays of a 33-year-old man who has had progressive pain in his nondominant left shoulder for the past 5 months. A proton density MRI is shown in Fig. 22c, and histiologic materials from the solid portion of the lesion are shown in Figures 22d and 22e. What is the most likely diagnosis?  1. Enchondroma 2. Giant cell tumor 3. Chondroblastoma 4. Chondromyxoid fibroma 5. Clear cell chondrosarcoma

Page 440: Oncology oite-review-2012
Page 441: Oncology oite-review-2012
Page 442: Oncology oite-review-2012
Page 443: Oncology oite-review-2012
Page 444: Oncology oite-review-2012
Page 445: Oncology oite-review-2012

Answer: 5 Ref.: Unni KK; Dahlin’s Bone Tumors 1996, pp. 71-108 -males 70%, females 30%-peak age; third decade-most common location: the proximal femur accounts for approx. 50%. nearly always

involves the epiphyseal region of a long bone.-clinical symptoms: pain is the most common at presentation-X-ray fractures: lesion has epiphyseal location in prox. femur or humerus 

Page 446: Oncology oite-review-2012

Question 89 - A 14-year-old boy has a 4-month history of aching pain in the distal thigh. Examination reveals a mass in the distal thigh. Figure 25a shows a plain radiograph, Figures 25b and 25c show MRI images, Figure 25d shows a bone scan, and Figure 25e shows a CT scan of the chest. The most likely diagnosis and Musculoskelatal Tumor Society (Enneking) stage is  1. osteosarcoma, stage IIB

2. osteosarcoma, stage III3. parosteal osteosarcoma, stage IIB4. periosteal osteosarcoma, stage IIB5. periosteal osteosarcoma, stage III

Page 447: Oncology oite-review-2012
Page 448: Oncology oite-review-2012
Page 449: Oncology oite-review-2012
Page 450: Oncology oite-review-2012
Page 451: Oncology oite-review-2012

Answer: 2 Osteosarcoma commonly occurs about the knee in children and young adults. Patients present primarily with pain. More than 90% of the intramedullary osteosarcomas are high grade and penetrate the cortex early to form a soft tissue mass. About 10-20% of patients have pulmonary metastases at presentation. Plain radiographs demonstrate a lesion in which there is both bone destruction and bone formation. MRI and CT scans are useful for defining the anatomy of the lesion in regard to intramedullary extension, involvement of neurovascular structures, and muscle invasion.Parosteal Osteosarcoma commonly occurs at the posterior aspect of the distal femur, proximal tibia, and proximal humerus. Patients present with a painless mass. The radiographic appearance is characteristic in that it demonstrates a heavily ossified, often lobulated mass arising form the cortex.Periosteal Osteosarcoma is a rare form of osteosarcoma and occurs most commonly in the diaphysis of long bones (femur and tibia). The radiographic appearance is fairly constant; a sunburst type lesion rests on a saucerized cortical depression. Staging system of the Musculoskelatal Tumor Society (Enneking System)Stage Description I-A Low grade, intracompartmental, no metastasesI-B Low grade, extracompartmental, no metastasesII-A High grade, intracompartmental, no metastasesII-B High grade, extracompartmental, no metastasesIII-A Any grade, intracompartmental, with metastasesIII-B Any grade, extracompartmental, with metastases Figure 25a demonstrates an intramedullary lesion, Figures 25b-c demonstrate a large lesion extending into the soft tissue, and figure 25e demonstrates lung metastasis. With mets it must be a grade III and the radiographic appearance is typical for osteosarcoma.

Page 452: Oncology oite-review-2012

Question 100 A 56 year old man who has a two year history of a progressive peripherial neuropathy has symmetric motor and sensory deficits in the lower extremities that are worse distally. Plain radiographs of the spine and pelvis show mulitple small sclerotic lesions in the pubic rami, left and right ilia, and the lumbosacral spine. Serum electrophoresis shows a monoclonal spike. What is the most likely diagnosis.  1. Metastatic lung cancer 2. Metastatic prostate cancer 3. Osteosclerotic myeloma 4. Non-Hodgkin’s lymphoma 5. Primary hyperparathyroidism

Page 453: Oncology oite-review-2012

Answer: 3

This is a classic presentation of osteosclerotic myeloma otherwise know as multiple myeloma. The major features of this disease are 97% have a homogenous monoclonal spike on serum and urine electrophoresis usually IgG. Sensorimotor polyneuropathy accompanies leading to severe weakness, atrophy, ataxia. Flat bones of the body typically show multiple sclerotic punch out lesions. Malignant plasma cells produce osteoclast activating factor that stimulate osteoclast proliferation and thus rapid bony destruction.

Harrisons 10th Edition Principles of Internal Medicine

Page 454: Oncology oite-review-2012

Question 112 Which of the following terms best describe most chondrosarcomas at intitial presentation?  1. Metastatic

2. Low-grade, intracompartmental3. Low-grade, extracompartmental4. High-grade, intracompartmental5. High-grade, extracompartmental

Page 455: Oncology oite-review-2012

Answer: 2 In a study by Memorial Sloan-Kettering Cancer Center most patients presenting with chindrosarcoma complain of pain 76% of the time as their primary symptom. Histologic grading is divided into Grade I-27%, Grade II-42% (Low grade) and Grade III-31% (high grade). Radiographically the lesion is centrally located 3/3 od the time.

Page 456: Oncology oite-review-2012

Question 123 A 36 year-old woman who has had intermittent pain in her knee for the past 8 months reports that over the past two months the pain has increased in frequency and intensity. Laboratory studies show that the CBC and ESR are within normal limits. AP and lateral radiographs are shown in Figures 33c and 33d. What is the most likely diagnosis? 1. Lymphoma 2. Osteomyelitis 3. Unicameral bone cyst 4. Aneurismal bone cyst 5. Eosinophilic Granuloma

Page 457: Oncology oite-review-2012
Page 458: Oncology oite-review-2012
Page 459: Oncology oite-review-2012
Page 460: Oncology oite-review-2012

Answer: 2 Although CBC and ESR were normal, this can happen with chronic osteo, the histological slides show osteo and rule out the other choices above. There is a fairly normal bone matrix (ruling out ABC or UBC) invaded with many small WBC’s in 33c. Slide 33d shows many PMN’s, not lymphocytes, or cosinophils. Making the only logical choice to be answer 4. The radiographs offer little help in differentiation, there is periosteal thickeneing and evidence of an involucrum with lytic lesions although. Ref- Dahlin DC, UnniKK: General aspects and data on 8452 cases, ed 4. Springfield, IL, Charles Thomas, 1986, pp. 448-452.

Page 461: Oncology oite-review-2012

Question 134 - A 14-year-old boy undergoes excisional biopsy of a 3-cm mass over the lateral aspect of the proximal forearm. No imaging studies were obtained prior to the biopsy. A photomicrograph of the biopsy specimen is shown in Figure 36. What is the most likely diagnosis?  1. Desmoid tumor 2. Rhabdomyosarcoma 3. Synovial sarcoma 4. Nodular fasciitis 5. Proliferative fasciitis

Page 462: Oncology oite-review-2012
Page 463: Oncology oite-review-2012

Answer: 3 

In figure 36 a biphasic pattern of spindle cells and epithelial cells should help to recognize a synovial sarcoma. Synovial sarcoma is most prevalent in adolescents and young adults between 15-40 years of age. It is more common in males and tends to affect the extremities (85%-95%). Enzinger F, Weiss S (eds): Soft Tissue Tumors, ed 3. St. Louis, MO, CV Mosby, 1995, pp 757-786.

Page 464: Oncology oite-review-2012

Question 139 A 56 year old man has had a slowly enlarging soft tissue mass in his left thigh for the past six months. Plain radiographs show only a soft tissue shadow with no mineralization or obvious bony involvement. The proton density MRI scans shown in figures 39A and 39B show a coronal view and axial view, respectively, of the thigh. At this time, management should include  1. excisional biopsy 2. incisional biopsy 3. resection with a wide margin 4. a repeat MRI scan in 3 months 5. a repeat clinical examination in 3 months

Page 465: Oncology oite-review-2012
Page 466: Oncology oite-review-2012
Page 467: Oncology oite-review-2012

Answer: 2 The MRI study shows a large heterogenous soft tissue mass confined to the lateral compartment of the thigh without evidence of bone involvement. Biopsy of this suspicious lesion is required for both diagnosis and staging. Soft tissue biopsies can be performed either as a needle biopsy, incisional biopsy, or excisional biopsy. Excisional biopsy is appropriate for small masses which are believed to be benign. If there is any question of malignancy, incisional biopsy should be performed through a longitudinal incision placed over the compartment involved in such a way that the biopsy tract can be removed at the time of the definitive procedure. The clinical history of a progressively enlarging mass with the MRI findings in figures 39a and b is highly suspicious and warrants a biopsy. Repeat MRI in 3 months or clinical observation would only delay treatment.

Ref. Enneking WF, Spanier SS, Goodman MA: A system for the surgical staging of musculoskeletal Sarcoma. Clin Orthop 1980; 153:106-120. General principles of tumors, in Crenshaw AH, (ed) Campbell’s Operative Orthopaedics, eighth Edition, 1992, pp228-230. Nelson TE, Enneking WF: Staging of bone and soft tissuesarcomas revisited, in Stauffer RN (ed): Advances in Operative Orthopaedics. St. Louis, MO, Mosby Year Book, 1994.

Page 468: Oncology oite-review-2012

Question 140 - A 10 year old boy of Mediterranean ancestry whose height is in the 25th percentile sustains a fracture of the distal femur following a mild fall. Radiographs reveal an impacted fracture of the distal femur as well as osteopenia in both femora and the pelvis. Laboratory studies show a hemoglobin level of 7 mg/dl. A complete hematologic evaluation is likely to reveal  1. hemoglobin S and C 2. hemoglobin S chains only 3. no hematologic abnormalities 4. increased total iron binding capacity 5. absence of or severely deficient beta globulin

Page 469: Oncology oite-review-2012

Answer: 5 The child described here most likely has thalassemia. This genetically determined hemoglobinopathy seen in patients of Mediterranean descent is characterized by a primary deficiency in the production of the beta chains of hemoglobin. The resulting abnormal hemoglobin leads to severe hemolytic anemia. The anemia and secondary hemochromatosis then affect most organ systems including the skeleton. The skeletal changes produced by thalessemia reflect compensatory hyperactivity and hypertrophy of the bone marrow, manifested by osteoporosis, widened medullary spaces, and thinned cortices with coarse reticulations. The clinical skeletal manifestations include arthralgias, delayed skeletal maturation, premature fusion of the epiphyses in the long bones resulting in short stature, and an increased incidence of fractures. The fractures heal more slowly than normal, and recurrent fractures at the same site have been reported. Hemaglobin S chains are indicative of sickle cell disease (HgbS-S) which occurs in 0.3-1.3% of North American Blacks. Sickle cell disease can lead to fractures with minor trauma due to cortical thinning secondary to marrow hyperplasia. The presence of hemoglobin C and S indicates sickle cell- hemoglobin C disease. These patients have musculoskeletal pain, frequently localized to the joints and have a slightly higher risk of osteonecrosis of the femoral and humeral head. An increased TIBC suggests iron deficiency anemia.  Ref. Dines DM, Canale VC, Arnold WD: Fractures in thalassemia. J Bone Joint urg 1976; 58A:662-666. Resnick D, (ed) Diagnosis of Bone and Joint Disorders, WB Saunders Co, 1995 pp.2107-2137.

Page 470: Oncology oite-review-2012

Question 160 -

What is the most common clinical indicator of reflex sympathetic dystrophy of the knee?  1. Effusion 2. Muscle atrophy 3. Atrophic hair changes 4. Disproportionate pain 5. Decreased range of motion

Page 471: Oncology oite-review-2012

Answer: 4 Disproportionate pain. O’Brien SJ, Ngeow J, Gibney MA, et al: Reflex sympathetic dystrophy of the knee: Causes, diagnosis, and treatment. American Journal of Sports Medicine 1995;23. pp. 655-659.Cooper DE, DeLee JC, Ramamurthy S: Reflex sympathetic dystrophy of the knee: Treatment using continuous epidural anaesthesia. Journal of Bone Joint Surgery 1989;71A:365-369.

Page 472: Oncology oite-review-2012

Question 165 - Figures 44a and 44b show the plain radiographs of a 12-year-old boy who has had left medial knee pain for the past 4 months. Figure 44c shows representative histologic material. What is the most likely diagnosis?  1. enchondroma 2. osteoblastoma 3. giant cell tumor 4. chondroblastoma 5. osteochondritis dessicans

Page 473: Oncology oite-review-2012
Page 474: Oncology oite-review-2012
Page 475: Oncology oite-review-2012
Page 476: Oncology oite-review-2012

Answer: 4 

Chondroblastoma is a lytic lesion commonly located eccentrically in the epiphysis of long bones, most often in skeletally immature individuals. The most common sites of origin are the proximal humerus, proximal and distal femur, and proximal tibia. The radiographic differential includes giant cell tumor crossing into the epiphysis. In this case however, the characteristic highly cellular chondral oval mononuclear cells seen on the histologic material confirm the diagnosis. Turcotte RE, Kurt AM, Sim FH, et al. Chondroblastoma. Human Pathology 1993; 24: 944-949.

Page 477: Oncology oite-review-2012

Question 177 A 45-year-old woman who has had increasing foot pain for the past 9 months has tenderness over the region of the cuboid. Oblique and lateral radiographs are shown in Figures 49a and 49b. Low-and high-power photomicrographs are shown in Figures 49c and 49d. What is the most likely diagnosis?  1. Chondroblastoma

2. Giant cell tumor3. Unicameral bone cyst4. Aneurysmal bone cyst5. Metastatic carcinoma

Page 478: Oncology oite-review-2012
Page 479: Oncology oite-review-2012
Page 480: Oncology oite-review-2012
Page 481: Oncology oite-review-2012
Page 482: Oncology oite-review-2012

Answer: 1

Chondroblastoma. Refer above for explanation. Note that there are islands of chondroid matrix. This is an unusal location for this tumor. Note that chicken wire calcification on the low power radiograph. This tumor usually occurs in patients less than 20 years old. ABC usually occurs in patients less than 20 years of age. It is eccentric , lytic, expansile area of bone destruction in the metaphysis. The eseential histo feature is cavernous blood filled without an endothelial lining. UBC most commonly occurs over the proximal humerus. The bone is often expanded. The lesion often appears trabeculated. Tratment can include methyprednisolone injection or curretage and bone grafting Bullough et al Atlas of orthopaedic Pathology.

Page 483: Oncology oite-review-2012

Question 186 - A 55 year old man with metastatic prostate cancer has a painful lesion of the midshaft of the humerus in which approx. 75% of the cortex is involved. Management should consist of. 1. an incisional biopsy. 2. a humeral cuff and a sling 3. closed interlocking nailing 4. radiation therapy to the humerus 5. plate fixation with bone fixation

Page 484: Oncology oite-review-2012

Answer: 3  Redmond reviewed 13 patients who had 16 pathological fractures of the shaft of the humerus secondary to metastatic disease. All but one fracture was stabilized with closed IM nailing. 14 extremities returned to nearly normal function in 3 weeks. The conclusion stated that IM nailing of the humerus for pathological fractures provides immediate stability and can be accomplished with a closed technique, brief operative time, minimum morbidity, with a resultant early return of function to the extremity.

Page 485: Oncology oite-review-2012

Question 190 - A 55-year old woman who has severe pain in her arm for the past 4 months reports that she felt a sudden snap in her arm after trying to open a tight jar lid. An AP radiograph of the humerus is shown in Figure 53a. A hight power photomictograph of the biopsy specimen is shown in Figure 53b. What is the most likely diagnosis?   1. Lymphoma 2. Multiple Myeloma 3. Hyperparathyroidism 4. Metatstatic bone disease 5. Mesenchymal chondrosarcoma

Page 486: Oncology oite-review-2012
Page 487: Oncology oite-review-2012
Page 488: Oncology oite-review-2012

Answer: 4 Dahlin DC, Unni KK: gineral aspects and data on 8452 cases, ed. 4. Springfield, IL, Charles Thomas, 1985, pp 408-413.Reference not found. In this age group (40-80 yrs.), the history alone suggests metastatic disease. The X-ray shows an aggressive, permeative, destructive, mixed blastic and lytic lesion with periosteal involvement and soft tissue extension in the proximal diaphyseal humerus associated with fracture. This is uncharacteristic for the other choices. Lymphoma and multiple myeloma are lytic. The high power H&E stain doesn’t have chondroid matrix so mesenchymal chondrosarcoma is unlikely. Hyperparathyroidism typically has osteoclasts and characteristic scalloping seen on microscopy. Plus no labs are given to support the diagnosis. Most of this came from various locations in Miller MD, Review of Otrthopaedics 2nd Ed.;WB Saunders, 1996.

Page 489: Oncology oite-review-2012

Question 205 Which of the following margins is achieved in a hip disarticulation performed as surgical treatment of a Musculoskeletal Tumor Society (Enneking) type IIA distal femoral osteogenic sarcoma?  1. Wide 2. Radical 3. Marginal 4. Intralesional 5. Wide-contained

Page 490: Oncology oite-review-2012

Answer: 2 Enneking WF: principles of Musculoskeletal Onocologic Surgery, in Evarts CM (ed): Surgery of the Musculoskeletal System, ed 2, New York, NY. Churchill Livingstone, 1990, pp 4647-4669

Radical margins are aceived when the entire tumor and its compartment (all surrounding muscles, ligaments and connective tissues) are removed. Marginal line of resection goes through the reactive zone of tumor, the reactive zone contains inflammatory cells, edema, fibrous tissue, and satellites of tumor cells and have 25 to 50 percent local recurrence. Wide margins are accomplished when the entire tumor is removed with a cuff of normal tissue and have less than 10 percent local recurrence. Intralesional margins go directly through the tumor and result in 100 percent local recurrence. I don't know what wide contaminated means.A radical margin is definitive for stage II lesions, but is no better than a wide-margin for stage I lesions. If it can be achieved with a logical procedure, radical local resection is just as effective as radial disarticulation for stage IIA lesions. Stage IIB lesions in most circumstances require disarticulation to achieve a radical margin.

Page 491: Oncology oite-review-2012

Question 219 - A 35-Year Old man has had increasing pain in the knee for the past 4 months. An AP Radiograph of the knee is shown in Figure 58a, and low- and high-power photomicrographs of the biopsy specimen are shown in Figures 58b and 58c. What is the most likely diagnosis?  1. Osteosarcoma 2. Chondroblastoma 3. Giant Cell tumor 4. Aneurysmal bone cyst 5. Desmoplastic fibroma

Page 492: Oncology oite-review-2012
Page 493: Oncology oite-review-2012
Page 494: Oncology oite-review-2012
Page 495: Oncology oite-review-2012

Answer: 3 50% of Giant cell tumors usually occur about the knee, with 28% involving distal femur. It is a lytic lesion with periosteal reaction. Histologically giant cells scattered uniformly, and the nuclei of the mononuclear and giant cells are similar in appearance. Most common age is 3rd decade. In this case the radiographic finding are similar to osteosarcoma and Desmoplastic fibroma as well as Giant cell tumor, however, histologic study singles out Giant cell tumor.

(Atlas of orthopedic surgery)

Page 496: Oncology oite-review-2012

Question 222 - Figures 60a and 60b show the radiographs of the ankle and distal leg of an 11-year-old girl after she twisted her ankle while playing soccer. She has no history of ankle or leg pain. Examination reveals localized swelling and tenderness over the lateral ankle, and the tibia is not tender. The bone lesion identified in the tibia most likely is   1. osteoblastoma 2. osteoid osteoma 3. ossifying fibroma 4. fibrous dysplasia 5. nonossifying fibroma

Page 497: Oncology oite-review-2012
Page 498: Oncology oite-review-2012

Answer: 5 DISCUSSION: Nonossifying fibroma almost always develops in patients under the age of twenty and is usually discovered when a x‑ray is taken for an injury. Histologically, there is a triphasic cytological appearance, featuring oval round cells, spindle cells, and giant cells. It usually is a single well‑delineated cortical defect with circular or oval lesions and smooth, lobulated edges. Adjacent body is sclerotic without periosteal reaction. The lower extremity long tubular bones are the most common sites of occurrence. Treatment is usually not necessary, but occasionally a large lesion will require intralesional excision and bone grafting. REFERENCES: Marks KE, Bauer TW: Fibrous Tumors of Bone. Orthop Clin North Am 1989; 20(3):377‑93.

Page 499: Oncology oite-review-2012

Question 227

An 80 year-old man has had increasing hip pain and difficulty ambulating for the past 6 months. An oblique radiograph pf the hip is shown in figure 64a, and a technetium bone scan is shown in Figure 64b. Low-and high power photomicrographs are shown in Figure 64c and 64d. What is the most likely diagnosis?  1. Pagets sarcoma 2. Pagets disease 3. Fibrous dysplasia 4. Hyperparathyroidism 5. Metastatic carcinoma

Page 500: Oncology oite-review-2012
Page 501: Oncology oite-review-2012
Page 502: Oncology oite-review-2012
Page 503: Oncology oite-review-2012

Answer: 2 Paget's disease is generally discovered in the patient 50 years of age and older. This disease in more common in those of eastern and western European decent and rare in Asians, Scandinavians, and blacks. 20% of patients are asymptomatic and the diagnosis is made while being worked‑up for an unrelated complaint. Symptomatic patients complain of bone pain, skeletal deformities, changes in skin temperature, pathologic fractures and symptoms related to nerve compression. Radiographic appearance is that of focal bone resorption and formation in a disordered trabecular pattern, with enlarged cortices and overall bone size. Paget's disease appears hot on technetium bone scans. Treatment is indicated for those with increasing pain and/or deformity. Calcitonin and diphosphonates are the pharmacologic agents available. Pathologic fractures, disabling arthritis, and severe malalignment are indications for surgical intervention.

REFERENCES: Merkow RL, Lane JM: Paget's Disease of Bone. Orthop Clinics NorthAm 1990;21(1):171‑89.

Page 504: Oncology oite-review-2012

Question 228 - Which of the following terms best describe most osteosarcomas at the time of diagnosis?  1. Metastatic 2. Low-grade, intracompartmental 3. Low-grade, extracompartmental 4. High-grade intracompartmental 5. High-grade extracompartmental

Page 505: Oncology oite-review-2012

Answer: 5 DISCUSSION: Osteosarcoma is a difficult disease to cure because it is predominantlyhigh‑grade and extracompartmental at the time of presentation, 90% stage IIB. Thecommon presentation is child or young adult with complaints of pain around the knee.The proximal femur and humerus, as well as, the pelvis are not uncommon sites. Pre operative multiagent chemotherapy is given for 8‑12 weeks and then staging studies areperformed. Resection, if possible, is then performed and maintenance chemotherapy isgiven for 6‑12 months. Long term survival has increased from 10‑20% to 60‑70%, withincreased limb salvage.  REFERENCES: Enneking WF, Spanier SS, Goodman MA. Clin Orthop 1980;153:106-20. This question deals with the surgical staging of musculoskeletal tumors: STAGE GRADE SITEIA LOW INTRACOMPARTMENTALIB LOW EXTRACOMPARTMENTALIIA HIGH INTRACOMPARTMENTALIIB HIGH EXTRACOMPARTMENTALIII ANY METS Upon diagnosis, most osteosarcomas are high grade, extracompartmental lesions, (stage IIB).

Page 506: Oncology oite-review-2012

Question 231 What is the most likely reason that blood for a homologous transfusion that tested negative for the HIV-antibody can carry a low but definite risk of HIV-transmission to recipients?  1. There are many mutations of the HIV virus

2. The test for HIV-antibody is not very accurate3. The virus may hide in the wall of red blood cells4. The virus may hide in the wall of white blood cells5. There is a delay between infection with HIV and the development of a detectable antibody

Page 507: Oncology oite-review-2012

Answer: 5 There is a delay between infection with HIV and the development of a detectable antibody. Blood donated in the “window period” may ultimately lead to HIV infection because no antibody formation has occurred thus detection is not possible. HIV mutations may occur but antibodies should develop unless an immunodeficient condition exists. HIV testing is very sensitive and very accurate with modern techniques. HIV does not “hide” in the walls of wbc’s and rbc’s. HIV binds to the cell surface, the viral capsid is released into the cell cytoplasm and ultimately viral DNA is integrated into the nucleus and host DNA. (AIDS, 3rd Ed. V. DeVita Jr.)

Page 508: Oncology oite-review-2012

Question 232 - What is the most current recommendation for definitive treatment of a 15 year-old boy who has a high grade osteosarcoma of the distal femur?   1. Surgical resection only

2. Raditation therapy only3. Radiation therapy and surgical resection 4. Chemotherapy only5. Chemotherapy and surgical resection

 

Page 509: Oncology oite-review-2012

Answer: 5 Chemotherapy and surgical resection combined has lead to a >80% 5 year survival rate. Surgical restriction alone has lead to a high recurrence and metastatic rates and poor (17.4%) survival rates. Radiation therapy alone did not control local recurrences and pulmonary metastasis. radiation and surgery also had high local recurrences and metastatic rates. Neoadjunctive chemotherapy has provided many improvements in treatment including elimination of micro metastases, necrosis of the primary tumor, reduction of tumor size and neovasculization and helps prevent local recurrences. Complete surgical resection is the mainstay of treatment of osteogenic sarcoma, the addition of neoadjunctive chemotherapy has proven to be the most effective.

Page 510: Oncology oite-review-2012

Question 243 - The radiograph shown in Figure 67 most likely represents which of the following disease processes?  1. Sickle Cell anemia 2. Rheumatoid arthritis 3. Ankylosing spondylitis 4. Degenerative disk disease 5. Diffuse idiopathic skeletal hyperostosis

Page 511: Oncology oite-review-2012
Page 512: Oncology oite-review-2012

Answer: 3 Notice the vertebral squaring and syndesmophytes that are characteristic of AS. There is also symmetric blurring of the SI joints. Sickle cell and RA do not have these type of changes in the axial skeleton. DDD has disc space narrowing and this x-ray has maintained height. DISH can give similar findings, but does not have the SI changes and apophyseal ankylosing.

Page 513: Oncology oite-review-2012

Question 264 - Figure 76 shows the radiographs of a 5-year-old girl who has pain in her left shoulder as a result of a fall from a swing. Management should now include…  1. a biopsy. 2. a CT scan. 3. an MRI scan. 4. a sling and swathe. 5. curretage and bone grafting.

Page 514: Oncology oite-review-2012
Page 515: Oncology oite-review-2012

Answer: 4  The patient has a pathologic fracture through a unicameral bone cyst of her proximal humerus. As this is a non-weight bearing bone, and a relatively benign lesion you treat the fracture. Healing of the fracture will occasionally stimulate healing of the cyst. If not then the cyst can be treated with curretage and bone grafting once the fracture has healed.

(Green, Swiontkowski, Pathologic Fractures in Children – Skeletal Trauma in Children, 1st Edition, 1992.)

Page 516: Oncology oite-review-2012

Question 11

Which of the following techniques is most commonly used to classify tissue type in soft-tissue tumors? 1. karyotyping2. flow cytometry3. transmission electron microscopy4. immunohistochemistry staining5. scanning electron microscopy

OITE 1998

Page 517: Oncology oite-review-2012

Answer: 4

Microscopy allows for visualization of the cells; however, special staining is required for identification and classification of specific cells. Karyotyping refers only to the number and structure of chromosomes in a cell. Flow cytometry is used to determine the amount of DNA in a cell, and are predictive for the degree of malignancy. Staining methods are currently the most commonly used method to classify tissue type. Enzinger FM, Weiss SW (eds): Soft Tissue Tumors, ed 2. St Louis, MO, CV Mosby, 1983, pp 5-9, 83-101.

Page 518: Oncology oite-review-2012

Question 15 -

A 55 year-old man has multiple lytic lesions in the humeri, clavicles, and scapulae. Which of the following diagnostic studies best confirms a diagnosis of multiple myeloma? 1. CT scan of the chest2. Bone marrow biopsy3. Complete blood cell count4. Lateral radiograph of the skull5. Erythrocyte sedimentation rate

Page 519: Oncology oite-review-2012

Answer: 2 Myeloma is a malignant bone tumor derived from plasma cells. It is associated with abnormalities of protein synthesis. It is the most common primary malignant tumor of bone. The disease is most common between the ages of 50 and 80. Patients most commonly present with bone pain, usually in the spine and ribs. Biopsy is necessary to establish the diagnosis in a solitary lesion and is still the gold standard for diagnosis of any lesion. Definitive diagnosis of multiple myeloma is established by bone marrow aspiration. However, multiple myeloma may also be diagnosed with considerable confidence based on radiographs and lab data. Bence Jones proteins may be found in the urine. Serum protein abnormalities may cause formation of a rouleaux in the peripheral blood smear. Serum and urine protein electrophoresis are usually abnormal. Answers #3, #4, and #5 may provide some information but the definitive diagnosis is based on bone marrow aspiration. Answer #1 provides little information. Campbell's Operative Orthopaedics. Ninth edition. Volume 1, pp 726-727.

Page 520: Oncology oite-review-2012

Question 17 -

Analysis of which of the following proteins is used to establish the diagnosis of Becker muscular dystrophy? 1. Myosin2. Troponin3. Tropomyosin4. Fibrillin5. Dystrophin

Page 521: Oncology oite-review-2012

Answer: 5 Becker muscular dystrophy is an X-linked inherited disorder present in approximately 1 in 30,000 live male births. The responsible gene is located on the xp21 region of the X chromosome including sixty-five exons that encode for the protein dystrophin. Duchenne muscular dystrophy is also related to a mutation of the dystrophin gene. Muscle biopsies for dystrophin have been extremely successful for identifying these dystrophies and distinguishing them from other clinically similar autosomal recessive myopathies. (Ref: Shapiro, Hoffman)

Page 522: Oncology oite-review-2012

Question 20 - Which of the following radiographic findings would be characteristic of the knee joints of a patient with neuropathic osteoarthropathy of the knee? 1. Fragmentation and subluxation of the normal joint articulation2. Varus deformity with medial subchondral sclerosis3. Preferential narrowing of the medial tibiofemoral compartment4. Narrowing of the medial, lateral, and patellofemoral compartments5. Bone proliferation at the patellar tendon and ligament insertions

Page 523: Oncology oite-review-2012

Answer: 1 Neuropathic osteoarthropathy (a.k.a. Charcot joint) develops most often in weight-bearing joints. The most likely cause is diabetes mellitus, but it is also associated with syphilis, leprosy, yaws, congenital insensitivity to pain, spina bifida, myelomeningocele, syringomyelia, aerodystrophic neuropathy, amyloid neuropathy, peripheral neuropathy of alcoholism, spinal cord injury, peripheral nerve injury, post-transplant neuropathy, and intraarticular steroid injections. The loss of sensation to the joint is followed by severe degenerative changes, osteophyte formation, articular and subchondral fractures, and often calcification of surrounding soft tissues. In the knee, this is a tricompartmental disease and will not selectively affect one compartment over another.

Page 524: Oncology oite-review-2012

Question 45 -

A skeletal survey is more accurate than a bone scan for detecting skeletal involvement in which of the following neoplastic diseases? 1. Ewing’s sarcoma2. Osteogenic sarcoma3. Multiple myeloma4. Metastatic prostate carcinoma5. Metastatic breast carcinoma

Page 525: Oncology oite-review-2012

Answer: 3 Multiple Myeloma lesions are cold on bone scan and because of this a skeletal survey is more useful. Miller, Review or Orthopaedics; page 292

Page 526: Oncology oite-review-2012

Question 59 - A 60-year-old man with no history of cancer has a destructive lesion in the proximal femur. He has a long history of tobacco use, but stopped smoking 5 years ago. A needle biopsy specimen of the lesion shows adenocarcinoma. Which of the following studies will most likely pinpoint the source of the primary tumor? 1. CT scan of the chest2. technetium bone scan3. bone marrow aspiration4. serum protein electrophoresis5. lateral skull radiograph

Page 527: Oncology oite-review-2012

 Answer: 1 Skeletal metastasis of unknown origin - ...plain radiographs of the chest established the diagnosis of carcinoma of the lung in seventeen patients (43%)....... Unlike skeletal metastasis of known origin - most often breast or prostate - a metastasis of unknown origin usually originates in the lung or kidney. In the present series, the most common occult primary site was the lung (63%) and the second most common was the kidney (10%). Rougraff BT, Kneisl JS, Simon MA: Skeletal metastasis of unknown origin: A prospective study of a diagnostic strategy. LBLS 1993; 75-A:1276-1281.

Page 528: Oncology oite-review-2012

Question 60 -

What factor is most commonly associated with malignant transformation of a giant cell tumor? 1. high-grade histology of the initial tumor2. multiple local recurrences after curettage3. previous treatment of the tumor with cryotherapy4. previous treatment of the tumor with radiation therapy5. extraosseous extension into two or more adjacent compartments

Page 529: Oncology oite-review-2012

Answer: 4 Most authors have agreed that radiation therapy should be avoided in the treatment of giant cell tumor, as there is a high prevalence of sarcomatous degeneration. Gitelis S, Wilkins R, Conrad EU II: Benign bone tumors, in Pritchard DJ (ed): Instructional Course Lectures 45. Rosemont, IL, American Academy of Orthopaedic Surgeons, 1996, pp425-446. (p440)

Page 530: Oncology oite-review-2012

Question 69 -

Crush fractures of the vertebral body are a particularly common problem in type 1 (postmenopausal) osteoporosis because 1. Trabecular bone is preferentially resorbed in this high bone turnover state2. Loss of water content in the disk increases impact load to the vetrebral bodies3. Stress is imposed by the relative stiffness of the arthrtic facet joints4. Increased energy demands are imposed by decreased circulation to the vertebral body5. The thick cortical bone found in the vertebral body resorbs rapidly following estrogen withdrawal

Page 531: Oncology oite-review-2012

Answer: 1 

The spine is composed primarily of trabecular bone, compared with cortical bone, it has a high surface-to-volume ratio. Because metabolic activity (remodeling) occurs on bone surfaces, trabecular bones in general and the vertebral bodies in particular are resorbed preferentially in times of skeletal loss. Osteoporosis is characterized by trabeculae of decreased size and number. It has also been demonstrated in osteoporosis that there is a thinning of the cortex as well as a change in the shape of the trabecular bone from plates to narrow bars. In the phases of bone loss, vertebral body density declines before a similar loss is detected in cortical areas. The body accommodates bone loss by redistribution. A 10 percent shift of bone mass outward from the epicenter through an enlargement of the bone diameter will compensate for 30 percent decrease in the bone mass against applied bending and torque stresses but not against axial loading. This differential resorption explains the timing and patterns of the fracture syndromes seen in osteoporosis. The incidence of vertebral crush fractures rises immediately after menopause, secondary to high trabecular content and estrogen deficiency. (precise mechanism of effects of estrogen withdrawal unclear) Bernstein J, Lane JM: Metabolic bone disorders of the spine, in Rothman RH, Simeone FA (eds): The Spine, ed 3. Philadelphia, PA, WB Saunders, 1992, pp. 1381-1427.

Page 532: Oncology oite-review-2012

Question 77 - Which of the following types of sarcoma of the bone is most sensitive to external beam radiation? 1. Ewings tumor2. Parosteal osteosarcoma3. Dedifferentiated chondrosarcoma4. Low grade intramedullary chondrosarcoma5. High grade intramedullary osteosarcoma

Page 533: Oncology oite-review-2012

Answer: 1 Parosteal osteosarcoma occurs on the surface of the metaphyseal regions of the distal femur or the proximal humerus most commonly. The treatment is wide surgical resection versus limb salvage. Dedifferentiated chondrosarcoma has a moth eaten appearance and may occur as a transformation of chondrosarcoma. Treatment is resection and prognosis is poor. Low-grade intramedullary chondrosarcoma is also treated with surgical resection. High-grade intramedullary osteosarcoma is usually treated with pre-op chemo and resection. The only tumor listed where radiation is an option is Ewings tumor.

Page 534: Oncology oite-review-2012

Question 102 -

Flow cytometry of tumors measures the  1. Size of cells2. Amount of DNA in cells3. Nucleus-cytoplasm ratio4. Specific DNA sequences5. Specific messenger RNA sequences

Page 535: Oncology oite-review-2012

Answer: 2

Flow cytometry is a method of quantitating components or structural features of cells primarily by optical means. Ploidy and cell cycle analysis of cancers is the major diagnostic use. Cells are passed single file through a laser beam by continuous flow and several parameters are measured including Cell Diameter, proportional quantity of granular (DNA) within the cell, and using fluorescent probes the total DNA or a specific DNA/mRNA sequence can be counted. In examining tumors the amount of DNA in each cell is important for determining neoplasia.

Ref. www.bio.umass.edu/mcfacs/intro.htm, the U. of Massachusetts web site on flow cytometry.

Page 536: Oncology oite-review-2012

Question 111 -

A 47-year-old woman who reports mild, aching pain in her knee has no history of trauma. Examination of the knee is normal. Figure 23a shows the AP radiograph. A bone scan shows increased uptake at this site only. Figure 23b shows the CT scan, and Figure 23c shows the histology from the CT scan-guided needle biopsy. Treatment should include 1. extended curettage and polymethylmethacrylate cementation2. extra-articular resection of the knee and an allograft arthrodesis3. wide resection of the proximal tibia and custom prosthetic replacement4. prophylactic internal fixation and postoperative irradiation5. excision of the lateral condyle and reconstruction with a hemicondylar allograft

Page 537: Oncology oite-review-2012
Page 538: Oncology oite-review-2012
Page 539: Oncology oite-review-2012
Page 540: Oncology oite-review-2012

Answer: 1 Patients with giant cell tumors typically present with local pain, swelling and tenderness. Radiographs usually reveal a lesion destructive of both medullary and cortical bone. The lesion is characterized by an expanding zone of radiolucency that is located eccentrically in the end of a long bone in an adult.The modern technique for the removal of a giant cell tumor involves wide decortication of all the bone overlying the area of the tumor. The cavity is filled with methylmethacrylate bone cement and covered with demineralized bone matrix to stimulate the restoration of strong cortical boundaries. The other procedures are much more invasive and not necessary to treat a low-grade neo-plastic lesion such as a giant cell tumor. Fewer complications and better functional results have been found after intralesional excision and insertion of methylmethacrylate than other techniques. Gitelis S, Wilkins R, Conard EU II: Benign bone tumors. In Pritchard DJ: Instructional Course Lectures 45,1996.

Page 541: Oncology oite-review-2012

Question 123 Which of the following types of osteosarcoma is associated with the best prognosis for long-term survival? 1. Parosteal2. Periosteal3. High-grade intramedullary4. Osteosarcoma occurring in Paget’s disease5. Osteosarcoma occurring in irradiated bone

Page 542: Oncology oite-review-2012

Answer: 1 

p. 194 (parosteal): Early adequate treatment [of parosteal osteosarcoma] should lead to cure in most patients. A long-term survival rate of 80%-90% is to be expected for patients who have parosteal osteosarcomas without dedifferentiation.”

p. 163 (in Paget’s disease): “Although long-term survival is rare for patients with this type of sarcoma, four patients have survived more than 10 years.”

p. 164 (in irradiated bone): “The location of these tumors in unresectable locations such as the skull, clavicle, scapula, and spine explains the traditionally poor prognosis.” Unni, Dahlin’s Bone Tumors, 5th ed., 1996.

Page 543: Oncology oite-review-2012

Question 135 -

Which of the following metastatic tumors to bone carries the greatest risk of complications from intraoperative bleeding? 1. Breast2. Prostate3. Gastrointestinal4. Kidney5. Multiple myeloma

Page 544: Oncology oite-review-2012

Answer: 4 Just remember that renal cell CA is extremely vascular, and that an preop embolization should be performed prior to the procedure in order to minimize the blood loss.

Page 545: Oncology oite-review-2012

Question 142 - A form of renal osteodystrophy that is characterized by pure osteomalacia is caused by  1. Secondary hyperparathyroidism2. Phosphate retention secondary to uremia3. Insufficient renal synthesis of 1, 25 dihydroxy vitamin D4. Aluminum deposition in bone from oral phosphate binders5. Persistent acidosis aggravating the negative calcium balance

Page 546: Oncology oite-review-2012

Answer: 4 There are many causes of rickets and osteomalacia. Renal osteodystrophy is a common complication of chronic renal failure and is one of the most common causes of osteomalacia. Pure osteomalacia is caused by the aluminum in phosphate binders used to treat hyperphosphatemia in renal failure. Desferoxamine is an effective chelator of aluminum in patients with biopsy documented aluminum-associated osteomalacia. Pure osteomalacia also can be caused by hypophosphatemia. The other choices are part of the mechanism of bone changes in renal osteodystrophy.

(Simon: Orthopedic Basic Science, 1994 pp168-172)

Page 547: Oncology oite-review-2012

Question 146

Figure 30 shows the current radiograph of a 32-year-old woman who had a giant cell tumor of the distal radius that was treated with curettage/burring and packed with polymethylmethacrylate 2 years ago. What is the most likely diagnosis? 1. Osteomyelitis2. Malignant degeneration3. Stress fracture4. Local recurrence of the giant cell tumor5. Bone resorption due to methylmethacrylate

Page 548: Oncology oite-review-2012
Page 549: Oncology oite-review-2012

Answer: 4  Number four is the correct answer because recurrence is the most likely cause of the lytic zone. Since cement resists invasion by the tumor, lysis of the surrounding bone is inevitably produced in a recurrence. A 1-2 mm lytic zone can be found normally, greater than 5 mm lytic zone is positive for recurrence and the 3 – 5 mm lytic zone is very suspicious and should be followed by MRI and image guided needle biopsy. No other sign of osteomyelitis or stress fractures are noted. 

Page 550: Oncology oite-review-2012

Question 153

Lymphatic metastasis is a common feature of which of the following lesions? 1. Liposarcoma2. Nodular fasciitis3. Rabdomyosarcoma4. Malignant fibrous histiocytoma5. Extra-abdominal desmoid tumor

Page 551: Oncology oite-review-2012

Answer: 3

Rhabdomyosarcoma is a high grade malignancy with a rapid growth pattern. Local recurrence and distant metastasis after inadequate excision occurs in almost all instances and is uniformly fatal. The primary site of metastasis is the lung. Lymph node metastasis occurs in about > 10% of the patients, (synovial cell sarcoma = 25%). Whereas M.F.H. only occasionally metastasizes to regional lymph nodes, and liposarcoma is slow growing and recurrences or local metastasis is < 10%. Reference: Enneking, W.F. et al. Clinical Musculoskeletal Pathology , University of Florida, pp. 16-1-16-46.

Page 552: Oncology oite-review-2012

Question 170

Which of the following surgical options after resection of a sarcoma about the knee would require a patient to expend the greatest amount of energy while walking? 1. Arthrodesis2. Rotationplasty3. Above-knee amputation4. Osteoarticular allograft5. Endoprosthesis (custom arthroplasty)

Page 553: Oncology oite-review-2012

Answer: 3 The answer to this question is based on a study by Harris which tested the effeciency , rate, and percent of max rate in ambulation, in amputees, those with arthrodeses and arthroplasties. They found that Above knee amputees expended the most energy, followed by arthrodesis patients, and then arthroplasty patients.

Reference: Harris IE, Leff AR, Gitelis S. et al: Function after amputation, arthrodesis,arthroplasty for tumors about the knee. JBJS 1990; 72A:1477‑1485.

Page 554: Oncology oite-review-2012

Question 186 - Which of the following metastatic carcinomas has the worst long-term prognosis? 1. Lung2. Breast3. Prostate4. Thyroid5. Renal

Page 555: Oncology oite-review-2012

Answer: 1 The answer is the lung, which has a dismal long-term prognosis. However, not included in the question, but reviewed in the cited reference and worthy of noting is Gastric CA. This has the worst prognosis with a 15% survival rate of 6 months after spinal metastasis is detected. Recommendations are palliative care rather than surgical intervention.Pulmonary has 6-month & 1-year survival rates of 15% & 22% respectively. Surgical intervention will be based on the overall condition of the patient and when indicated the extent of surgical intervention should be as small as possible.Breast & prostate CA have better survival rates than the others, and are often hormonally sensitive tumors. Making endocrine therapy initially crucial with the good long term survival of prostate CA patients. When intervention is indicate, it should be combined with other modalities (Chemo, Etc.) and a more extensive surgery including decompression and fixation, but also resection of the metastatic lesion and reconstruction.Breast CA rarely progresses with symptoms due to spinal metastasis, likely due to symptoms being controlled with hormone and radiation therapy.Renal CA has a poor 6 months survival rate of 51% which is close to lung CA, but a sharp decline in deaths from 6-36 mos results in a high (40%) 3-year survival rate. These should be categorized into rapid growing & slow growing with surgical intervention based on this classification.  Reference: Tatsui H, et al. Survival rates of patients with metastatic spinal cancer after scintigraphic detection of abnormal radioactive accumulation. Spine 1996;21:2143-48.

Page 556: Oncology oite-review-2012

Question 187 - The Magnetic resonance imaging signal characteristics of a high-grade soft-tissue sarcoma are best described as 1. T1-low, T2-low.2. T1-low, T2-high.3. T1-moderate, T2-low.4. T1-high, T2-low.5. T1-high, T2-high.

Page 557: Oncology oite-review-2012

Answer: 2 After plain radiographs of the affected area have been obtained, magnetic resonance imaging modality is the best imaging modality for detecting and characterizing the lesion, regarding definition of normal muscle, fascial boundaries, and the tumor mass. Although MR imaging is not specific in determining whether lesions are benign or malignant, it can be useful in evaluation other characteristics, such as size, pattern of growth, integrity of natural boundaries, and homogeneity. Intravenous contrast agents are not necessary to evaluate neurovascular structures. Both the T1 & T2 weighted images are essential to detect and characterize soft tissue lesions.Most Tumors have long T1 and T2 relaxation times, therefore, in most instances signal intensity alone is of limited benefit. Exceptions are lipoma, hematoma, intra-lesional hemorrhage. Hemorrhage may occur in some soft tissue lesions, especially sarcomas, leading to the low T1 and high T2 intensity sound on MR. Miller, indicates that Water, CSF, acute hemorrhage and soft tissue tumors appear dark on T1 and light on T2. Reference: Soft-Tissue Tumors: Diagnosis, Evaluation, and Management. JAAOS 1994;2:202-211. Sundaram, M, The Role of MRI in evaluation of Muskuloskeletal Tumors, Radiology: Diagnosis-Imaging_Intervention Vol 5: 104; 6-10.

Page 558: Oncology oite-review-2012

Question 210 -

The radiograph shown in Figure 50a and the CT scan shown in Figure 50b reveal a lesion in the left femoral neck of a 12-year-old boy who has pain in the left hip. The most likely cause of the osteopenia of the left proximal femur is 1. disuse osteopenia2. paraendocrine effect of the tumor3. abnormally increased density on the right side4. side effect of the treatment of the lesion5. extensive tumor involvement of the left hip

Page 559: Oncology oite-review-2012
Page 560: Oncology oite-review-2012
Page 561: Oncology oite-review-2012

Answer: 1 Figure 50A: AP pelvis with a 1 cm lucency in the calcar region of the femoral neck and diffuse osteopenia of the proximal femur. Figure 50B: CT scan of the same patient showing a well demarcated lcm lesion in the femoral neck with an obvious nidus. Careful evaluation of the radiographs is critical in this question. First the obvious osteoid osteoma must be recognized. From there answers 2 and 3 can be eliminated. Neither radiograph reveals any sign that treatment has occurred, and the CT scan shows no signs of extensive involvement, thus answer 5 should also be eliminated. The real key is believing that such diffuse osteoporosis could occur in such a young child, as Jones described in his article from 1969.  Jones G: Radiological appearances of disuse osteoporosis. Clin Radiol 1969;20:345‑353.

Page 562: Oncology oite-review-2012

Question 224 - The diagnostic distinction between a benign enchondroma and a low-grade intramedually chondrosarcoma is based primarily on the 1. clinical history and radiographic findings.2. technetium bone scan3. flow cytometry pattern of extracted chondrocytes4. immunohistochemical staining patterns of a biopsy specimen5. histologic features of a biopsy specimen stained with hematoxylin-cosin 

Page 563: Oncology oite-review-2012

Answer: 1 "The diagnostic strategies for benign bone tumors center on the initial radiographic presentation." … "For example calcified, lytic phylangeal lesion strongly suggests the presence of cartilage that is consistent with enchondroma."

Page 564: Oncology oite-review-2012

Question 246 - Figures 66a and 66b show the radiographs of an 8-year-old girl who has a firm, immobile mass of her middle finger at the proximal interphalangeal joint. Figure 66c shows the histopathology of the biopsy specimen. What is the most likely diagnosis? 1. Fracture healing 2. Chondrosarcoma3. Periosteal chondroma4. Periosteal osteosarcoma5. Dysplasia epiphysealis hemimelica

Page 565: Oncology oite-review-2012
Page 566: Oncology oite-review-2012
Page 567: Oncology oite-review-2012

Answer: 3 

Periosteal chondroma is a benign chondroid tumor that arises under or in the periosteum on the surface of cortical bone. Has a scalloping of the cortex with a well defined margin between the tumor and bone. The tumor bed shows a variable amount of sclerosis. Also a variable amount of calcification exists. Histologically, as in this case, there is chondroid matrix. The predilection for the proximal end of long bones, particularly the humerus, and the bones of the hands, has been noted. This tumor must be differentiated from periosteal chondrosarcoma, which penetrates the cortical bone and is generally larger with a soft tissue mass. Histology will also show more polymorphism and hyperchromatic nuclei of chondrosarcoma.  Boriani. JBJS 1983;65A:205-212.

Page 568: Oncology oite-review-2012

Question 262 A 45-year-old woman has a deep soft-tissue mass in the thigh. The MRI scan Shows a 12-cm heterogenous mass that is deep to the fascia, and the CT scan Shows three small (5 to 10 mm) peripheral, noncalcified pulmonary nodules. What is the stage of disease according to the staging system of the Musculoskeletal Tumor Society? 1. 12. 23. I4. II5. III

Page 569: Oncology oite-review-2012

Answer: 5 The system adopted by the Musculoskeletal Tumor Society is based on three factors: Grade (G), Site (T), and metastases (M). Each of these factors is stratified by components that influence both prognosis and response to treatment. Grade is anassessment of biologic aggressiveness of the legion. There are three stratifications ofGrade: G0 = benign, G1 = low grade malignant (few mitoses, moderate differentiation,and distinct matrix), G2 = high grade malignant (frequent mitoses, poorly differentiatedcells, and sparse immature matrix). Site is the anatomic setting of the legion, determinedby radiographic imaging. There are three strata T0 = intracapsular (does not extend beyond the compartmental boarders of its origin and remains completely within its capsule) T1 = extracapsular, intracompartmental, (extends beyond its capsule butremains within its compartment), T2 = extracapsular, extracompartmental, (extendsbeyond its compartmental boundaries). Metastasis has two strata M0 = no evidence of regional or distant mets and M1 = regional or distant mets. Stages for benign tumors are listed as 1,2 and 3 whereas stages for malignant tumors are I, II, and III. 

Page 570: Oncology oite-review-2012

Stage Grade Site Mets Benign tumors Stage 1, latent G0 T0 M0 Stage 2, active G0 T0 M0 Stage 3, aggressive G0 T0 M0Malignant Tumors Stage I, low grade A. Intracompartmental G1 T1 M0 B. Extracompartmental G1 T2 M0 Stage II, high grade A. intracompartmental G2 T1 M0 B. extacompartmental G2 T2 M0Stage III, metastaticA. intracompartmentaldistant mets G1-2 T1 M1B. extracompartmental distant mets G1-2 T2 M1By the nature of the radiographic appearance of this person’s tumor andassuming we are dealing with a malignant tumor with metastases, this has to be a stage III tumor.

Page 571: Oncology oite-review-2012

Question 269 -

A patient who has a malignant bone tumor of the hip undergoes resection and hip arthrodesis. While walking, mean oxygen consumption would be approximately what percent of that observed in someone who has a normal hip? 1. 50%2. 100%3. 130%4. 200%5. 300%

Page 572: Oncology oite-review-2012

Answer: 3

“Unilateral arthrodesis has been shown to increase O2 consumption in normal gait by 32%.” 

Page 573: Oncology oite-review-2012

Question 8 Figure 2 shows the plain AP radiograph of a 20-year-old man who has had a painful swelling in his left wrist for the past 4 months. A bone scan shows a solitary lesion. A radiograph and CT scan of the chest are normal. Prior to biopsy, what is the most likely diagnosis? 1. Telangiectatic osteosarcoma2. Chondromyxoid fibroma3. aneurysmal bone cyst4. giant cell tumor5. chondroblastoma

OITE 1999

Page 574: Oncology oite-review-2012
Page 575: Oncology oite-review-2012

Answer: 4

Bones involved are the distal femur, proximal tibia, distal radius. other long bones. On x‑ray a lytic lesion in the epiphysis without surrounding sclerosis or periosteal bone reaction. Often has a "soap bubble" pattern. Treatment is en bloc excision, if possible, but usual treatment is currettage.

Page 576: Oncology oite-review-2012

Question 17 -  Which of the following is considered the most common primary carcinoma to metastasize to bones distal to the elbow and knee? 1. Breast2. Prostate3. Kidney4. Lung5. Thyroid

Page 577: Oncology oite-review-2012

Answer: 4 Axial skeletal involvement is more common than appendicular involvement, and metastatic lesions below the elbow and knee are relatively uncommon. Autopsy studies on patients with metastatic disease below the knee and elbow revealed that lung was the most common primary site. The tibia was the most common bone affected. Pain, local soft-tissue extension and pathologic fractures were the most common presenting symptoms. Reference: Leeson MC, Makley JT, Carter JR: Metastatic skeletal disease distal to the elbow and knee. Clin Orthop 1986;206:94-99.

Page 578: Oncology oite-review-2012

Question 25  Which of the following signal sequences describes the MRI scan characteristics of normal tendons? 1. Low on T1-weighted images and low on T2-weighted images2. Low on T1-weighted images and high on T2-weighted images3. High on T1-weighted images and low on T2-weighted images4. High on T1-weighted images and moderate on T2-weighted images5. High on T1-weighted images and high on T2-weighted images

Page 579: Oncology oite-review-2012

Answer: 1

T1 sequenced images are weighted toward fat and T2 sequenced images are weighted toward water. Consequently, normal tendons, ligaments, and cortical bone all appear low signal on all pulse sequences. Beaty JH (ed): OKU 6. Rosemant IL, AAOS, 1999, 9981-87.

Page 580: Oncology oite-review-2012

Question 36 -  Which of the following conditions will most likely eventually develop in a newborn girl with achondroplasia? 1. Degenerative disease of the hips2. Spinal stenosis3. Atlantoaxial instability4. Cardiomyopathy5. Patellar subluxation

Page 581: Oncology oite-review-2012

Answer: 2

Achondroplasia is the most common type of short-limb disproportionate dwarfism. The primary defect found in achondroplastic dwarfs is abnormal enchondral bone formation. Periosteal and intramembranous ossification is normal. Spinal deformities are the most common and potentially disabling problem for the achondroplastic dwarf. The spinal canal is developmentally narrowed, particularly in the lower lumbar segments. Stenosis of the spinal canal and intervertebral foramen is secondary to short, thickened pedicles, interpedicular narrowing, thickened laminae, and inferior facets. Additional factors, such as intervertebral disc herniation, degenerative spondylolysis, excessive lumbar lordosis, or anterior wedging of the vertebral bodies from a thoracolumbar kyphosis, narrow the canal further. Reference: Bassett GS: The osteochondrodysplasias, in Morrissey RT, Weinstein SL 9eds): Lovell and Winter’s Pediatric Orthopaedics, ed 4. Lippincott-Raven, 1996, pp 203-223. 

Page 582: Oncology oite-review-2012

Question 43 -  A 75-year-old man has a destructive lesion of the proximal femur with soft-tissue extension. A technetium bone scan shows no other lesions, and a CT scan of the chest is negative. A needle biopsy of the soft-tissue component shows high-grade chondrosarcoma. According to the staging system of the Musculoskeletal Tumor Society, the stage of the lesion is 1. IB2. IIB3. III4. 25. 3

Page 583: Oncology oite-review-2012

Answer: 2

Grade is an assessment of the biological aggressiveness of the lesion, which is a blend of histology, radiographic assessment, and clinical reflection. G0=benign, G1=low grade malignant, G2=high grade malignant Chondrosarcoma is considered high grade malignant. The anatomic setting or Site (T) has a direct relationship to the prognosis and the choice of surgical procedure. These are determined primarily by clinical and radiographic techniques. T1=intracapsular, T2= extracapsular, intracompartmental, T3=extracapsular, extracompartmental. In most staging systems for carcinomas metastatic involvement is stratified as either regional (N) or distant (M). For sarcomas both have the same ominous prognosis and both are designated (M). M0=no regional or distant mets, M1=regional or distant mets present. The three factors of grade, site, and metastasis are combined to form the criteria for the progressive stages of benign and malignant lesion. Benign lesions are designated by the Arabic numerals 1,2,and 3 corresponding to latent, active, or aggressive lesions. Stages of malignant lesions are designated by the Roman numerals I,II, or III, and these stages are synonymous with low-grade, high-grade, and metastatic. The three states of sarcomas are further stratified into A or B depending on whether the lesion is anatomically intracompartmental (A) or extracompartmental (B).

Page 584: Oncology oite-review-2012

Question 48  Figure 8 shows the AP radiograph of the sacroiliac joints of a 29-year-old man who has had increasing lower back pain and stiffness for the past 4 months. Management consisting of exercise and nonsteroidal anti-inflammatory drugs has provided only minimal relief. Examination reveals decreased lumbar mobility in all directions, normal sensibility, and no weakness. The most useful physical finding for confirming the diagnosis is 1. a positive straight leg raising test.2. limitation of chest expansion to 1" or less.3. the presence of urethritis.4. bilateral tightness of the hamstrings.5. unilateral absence of the Achilles reflex.

Page 585: Oncology oite-review-2012
Page 586: Oncology oite-review-2012

Answer: 2

The AP radiograph of the patient's sacroiliac joints shows sclerosis in the bilateral SI joints indicative of sacroiliitis. This is a hallmark finding in ankylosing spondylitis as are the rest of the symptoms described in the question. Patients with AS often have other joint involvement such as pain from inflammation of the costovertebral joints which inhibits chest expansion. Eventual ankylosis of these joints may result in severe fixed limitations of chest expansion. A positive straight leg-raising test is a tension sign that is suggestive of a herniated disc, yet the patient has normal sensibility and no weakness in his lower extremities. This would also exclude unilateral absence of the Achilles reflex. The presence of urethritis would be suggestive of Reiter's. Bilateral tightness of the hamstrings may be a finding but is not a useful finding for confirming the diagnosis. Reference: Frymoyer JW (ed): The Adult Spine: Principles and Practice. Philadelphia, PA, WB Saunders, 1992, pp 699-705.El-Khoury GY, Kathol MH, Brandser EA: Seronegative spondyloarthropathies. Radiol Clin North Am 1996;34:343-357.

Page 587: Oncology oite-review-2012

Question 63 -  Figure 13 shows the AP radiograph of both knees of a 55-year-old man who reports an acute exacerbation of right knee pain for the past 36 hours. Examination reveals a palpable effusion, moderate warmth, and pain on range of motion. A synovial fluid aspiration will most likely show 1. A WBC greater than 100,000/mm32. Gram-negative intracellular diplococcus3. Positive birefringent crystals4. Negative birefringent crystals5. Elevated complement levels

Page 588: Oncology oite-review-2012
Page 589: Oncology oite-review-2012

Answer: 3

Figure 13 shows chondrocalcinosis of bilateral knee menisci, which may be caused by several disorders, including calcium pyrophosphate deposition (CPPD), ochoronosis, hyperparathyroidisim, hypothyroidisim, and hemechromatosis. CPPD (a.k.a. pseudogout) is a common disorder of pyrophosphate metabolism that occurs in older patients and occasionally causes acute attacks, usually in the lower extremities, especially the knee. Short, blunt (rhomboid-shaped) rods that are weakly positively birefringent are demonstrated following aspiration. Negative birefringent crystals are seen in gouty knee aspirates. Answers 1 and 2 are seen in septic knee aspirates, which with this patient’s symptoms, should be in the differential diagnosis. However, the radiograph is more suggestive of pseudogout condition.

Page 590: Oncology oite-review-2012

Question 74  Which of the following benign lesions may occasionally metastasize to the lungs. 1. Nonossifying fibroma2. Osteofibrous dysplasia3. Chondromyxoid fibroma4. Chondroblastoma5. Periosteal chondroma

Page 591: Oncology oite-review-2012

Answer:  4

Chondroblastoma is a rare benign neoplasm of fetal-type cartilage differentiation.  Over 50% of the lesions occur in patients between the ages of 15 and 25, with 70% occurring in the long bones.  Radiologically, Chondroblastoma is usually a well-circumscribed lytic lesion that, when in a long bone, involves the epiphysis. Histologically, Chondroblastoma is composed of sheets of stromal cells, scattered multinucleated giant cells, and varying amounts of chondroid matrix. The stromal cells, resembling fetal chondroblasts, are distinctive. This benign, slow growing neoplasm is best treated with curettage and bone grafting. Rarely, these are locally destructive.  Another rare behavior is pulmonary metastasis, the so-called“benign metastasizing Chondroblastoma.” Growth of these rare metastatic foci is very slow, and they are usually treated successfully by pulmonary wedge resections. McCarthy EF, Frassixa FJ (eds):  Pathology of Bone and Joint Disorders with Clinical and Radiological Correlation.  Philadelphia, PA, WB Saunders, 1998, p221.

Page 592: Oncology oite-review-2012

Question 95  In which of the following pediatric neoplasms can involvement of the synovium result in arthralgias? 1. Ewing’ tumor2. Osteosarcoma3. Eosinophilic granuloma4. Leukemia5. Chondrosarcoma

Page 593: Oncology oite-review-2012

Answer: 4

Bone and joint symptoms are common presenting complaints in children with acute leukemia. These include bone tenderness, swelling, and arthralgias. Arthralgias, which symmetrically involve multiple joints, are caused by leukemic infiltration of the synovial membrane. Synovial involvement is minimal to nonexistent in the other forms of cancer above.

(Pathology of Bone and Joint Disorders with Clinical and Radiologic Correlation, 1998, 127-128; Annals of Internal Medicine, 1963)

Page 594: Oncology oite-review-2012

Question 104  Which of the following devices provides the best prophylactic stabilization of an impending pathologic fx caused by a lytic defect in the mid-diaphysis of the femur? 1. plate and screws alone2. plate and screws with cement3. Rush rod with cement4. Ender’s rods with cement5. locked intramedullary nail 

Page 595: Oncology oite-review-2012

Answer: 5

Impending pathologic fx of the femoral shaft can usually be treated by conventional closed IM rodding techniques. The use of reconstruction nails, locked prox. And distally, has served to reduce the common complication of progressive fem collapse with telescoping of the fx fragments and prox migration of the rod. Prox placement of screws into the fem head offers more secure fixation than conventional transverse or antegrade screw placement and may protect the femoral head from subsequent fx.

Both impending lesions and complete pathologic fx of femoral head and neck should be managed with replacement arthroplasty. Pathologic fx in supracondylar and condylar regions of femur are unusual and difficult to tx. If there is sufficient bone stock, use of conventional internal fixation devices augmented with methylmethacrylate will usually achieve stability. Tx principles are the same regardless of location. A construct should ideally provide enough stability to allow immediate full weight bearing with enough durability to last the patient’s expected lifetime.

Page 596: Oncology oite-review-2012

Question 112 -  Figures 25a and 25b show the AP and lateral radiographs, and Figures 25c and 25d show the T1 and T2 weighted MRI scans of a 42 year old man who has a painful mass of the ankle. Figure 25e shows a histopathologic specimen. What is the most likely diagnosis? 1. Aneurysmal bone cyst2. Giant cell tumor3. Hemangioma4. Osteomyelitis5. Intraosseuos ganglion

Page 597: Oncology oite-review-2012
Page 598: Oncology oite-review-2012
Page 599: Oncology oite-review-2012
Page 600: Oncology oite-review-2012

Answer: 5

Inraosseous ganglions are also known as subchodral bone cysts. They typically occur in the 2nd – 6th decades of life and are more common in males. On plain radiographs inraosseous ganglia appear as well defined, oval or round osteolytic areas with a thin rim of sclerotic bone. On MRI, there is a low signal on T1 and high on T2. Histological features include stellate or fibroblast-like cells that produce a large amount of mucoid ground substance.Aneurysmal bone cysts are eccentric, lytic, expansile areas of bone destruction on x-ray. The essential histologic features are cavernous blood filled spaces without an endothelial lining. This is not seen in Fig. 25e.Giant cell tumors reveal a purely lytic destructive lesion in the metaphysis. The basic proliferating cell has a round or oval shaped nucleus. Numerous mitotic figures are also seen. Hemangiomas are soft tissue tumors that can occur in a cutaneous, subcutaneous, or intramuscular location. Plain radiographs may reveal small phleboliths. On MRI, heterogeneous lesions with numerous small blood vessels are seen.There multiple forms of osteomyelitis and most are easily recognized clinically, radiographically and histologically. Most are located in the metaphyseal region and show varying degrees of a lytic process with some degree of sclerosis. Helwig U, Lang, Baczynski,: The intraosseous ganglion: A clinical pathologic report on 42 cases. Arch Orthop Trauma Surg 1994;114: 14-17.M. Forest, Tomeno, Vanel, Orthopedic Surgical Pathology, 1998, pp547-552.

Page 601: Oncology oite-review-2012

Question 118 -  Which of the following soft-tissue tumors has the following MRI scan signal characteristics: high signal on T1-weighted images and intermediate to high

signal on T2-weighted images?

 1. Synovial sarcoma2. Extra-abdominal desmoid tumor3. Malignant fibrous histiocytoma4. Lipoma5. High-grade pleomorphic liposarcoma

Page 602: Oncology oite-review-2012

Answer: 4

Magnetic resonance imaging has become the most useful modality for the definition of soft tissue masses. Although the MR image can detect soft tissue masses with a very high sensitivity, it is not possible to accurately predict the histology or whether a lesion is benign or malignant. The two exceptions to this general rule are lipomas and hemangiomas. Lipomas often are very homogeneous and have signal characteristics that exactly match those of the surrounding fat, high signal on T1-weighted images and intermediate to high on

T2-weighted images, thus establishing the diagnosis.

 Reference: Frassica FJ, Frassica DA: Soft tissue tumors: Diagnosis, evaluation, and management. J AM Acad Orthop Surg 1994;2:202-211

Page 603: Oncology oite-review-2012

Question 132  Figures 31a and 31b show the radiograph and biopsy specimen of a 57-year-old man with no history of trauma who has had painful swelling of the distal phalanx of the little finger for the past 2 months. What is the most likely diagnosis? 1. Osteomyelitis2. Epidermal inclusion cyst3. Metastatic lung carcinoma4. Metastatic kidney carcinoma5. Metastatic prostate carcinoma 

Page 604: Oncology oite-review-2012
Page 605: Oncology oite-review-2012
Page 606: Oncology oite-review-2012

Answer: 3

“Acrometastases” is a term first used by German authors about 25 years ago to describe metastatic lesions occurring in the hands and feet. The filtering mechanism of the lungs may partially explain why the lung tumor cells bypass the lung-filtering activity by directly invading pulmonary vessels, reaching the left heart and then spreading to remote regions. Therefore, lung tumors theoretically are more likely to be associated with acrometastatic tumors. Carcinomas of any origin can spread to bone, the most common are carcinomas of lung, breast, prostate, kidney, or thyroid. Carcinomas of the prostate, breast, and kidney secondary to “batson’s plexus” is a major factor in the development of spinal metastasis. Pelvic bone metastasis of prostate cancer is common secondary to proximity. The figures do not show the characteristics of osteomyelitis of cortical destruction and formation of periosteal new bone. Epidermal inclusion cysts do not pertain to figures shown. Reference: Lombardi RM, Amadio PC: Acrometastases, in Sim FH (ed): Diagnosis and Management of Metastatic Bone Disease: A multidisciplinary Approach. New York, NY, Raven Press, 1988, pp. 237-243.

Page 607: Oncology oite-review-2012

Question 145 -  A 73-vear-old woman has had aching pain in the right ankle region for the past 4 months. Results of serum and urine protein electrophoresis show normal migration. Figures 34a through 34c show the plain lateral radiograph of the talus, sagittal TI-weighted MRI scan, and biopsy specimen. What is the most likely diagnosis? 1. Metastatic carcinoma2. Lymphoma3. Multiple Myeloma4. Fibrosarcoma5. Malignant fibrous histiocytoma

Page 608: Oncology oite-review-2012
Page 609: Oncology oite-review-2012
Page 610: Oncology oite-review-2012
Page 611: Oncology oite-review-2012

Answer: 2

The referenced films show an x-ray with a lytic lesion and periosteal elevation of the talus, an MRI with decreased signal on T1, and a photomicrograph with abundant mononuclear cells in various stages of division. Given the negative SPEP/UPEP, the only possible correct response is Lymphoma. Bone tumors caused by Lymphoma have the above characteristics and are mostly found in the Femur, Pelvis, and Skull. Involvement of the talus is rare. Reference: Huvos, Bone Tumors 2d Edition, 625-637 

Page 612: Oncology oite-review-2012

Question 157 -  Figures 38a through 38c show the plain radiograph and coronal T1- and T2 -

weighted MRI scans of a 30-year-old man who has a painful lump over the distal deltoid on the lateral aspect of his arm. The mass is marginally excised, and a histologic section at the periphery of the mass is shown in Figure 38d. Management should include 1. observation.2. serial follow-up staging studies.3. radiation therapy.4. wide reexcision, followed by radiation therapy.5. chemotherapy.

Page 613: Oncology oite-review-2012
Page 614: Oncology oite-review-2012
Page 615: Oncology oite-review-2012
Page 616: Oncology oite-review-2012
Page 617: Oncology oite-review-2012

Answer: 1

Osseous soft tissue tumors are either extraskeletal osteosarcomas or myositis ossificans. If they specifically show you the periphery of the biopsy specimen, you can bet it will be myositis ossificans because that is where the characteristic features of trabecular osteoid, rimming osteoblasts and remnants of the muscle tissue will be found. The osteoscarcoma would have the osteoid in the center and be much more disorganized. So once you've figured out this is MO, you know to just watch it. If you excise it, it will often recur, so don't bother. There is nothing to stage, and no adjuvant therapy is recommended for this benign process. Enzinger FH, Weiss, SW: Osseous soft tissue tumors, in Enzinger, FM, Weiss SW (eds): Soft Tissue Tumors, ed 3. St Louis, MO, Mosby, 1995, pp 1013‑1037. Huvos, AG: Miscellaneous tumors of soft tissue and bone, in Huvos AG (ed): Bone Tumors. Diagnosis, Treatment and Prognosis, ed 2. Philadelphia, PA, WB Saunders, 1991, pp 745‑753

Page 618: Oncology oite-review-2012

Question 161 -  A 42-year-old woman has had persistent aching pain in her right arm for the past 9 months that now awakens her from sleep. Management consisting of oral narcotics has failed to provide relief. Figures 39a and 39b show the plain radiographs of the right humerus, and Figure 39c shows the biopsy specimen. What is the most likely diagnosis? 1. Chondrosarcoma2. Ewing’s sarcoma3. Enchondroma4. Osteosarcoma5. Malignant fibrous histiocytoma

Page 619: Oncology oite-review-2012
Page 620: Oncology oite-review-2012
Page 621: Oncology oite-review-2012

Answer: 1The radiographs show a long bone diaphyseal intramedullary process, maybe some lytic stuff going on inferiorly, calcification in the proximal portion, some erosion into the endosteal surface on the close‑up, no real periosteal reaction, and no cortical expansion‑ The micrograph looks kind of like cartilage, with cells and matrix, and the cells don't look too nasty‑ don't see giant cells or multinucleated cells all over the place. The other key feature is the process is painful. This is all typical of chondrosarcomas. Ewing's typically have periosteal reaction, and the biopsy should be more cellular. Enchondromas are typically not painful, they don't erode endosteal surfaces, and the biopsy would be less cellular. The radiograph doesn't look anything like an osteosarcoma‑ not aggressive enough, no periosteal reaction, etc. MFH typically won't have the lytic areas, and the micro would be very different‑ much more cellular, lots of atypia, giant cells. Unni KK: Chondrosarcoma (primary, secondary, dedifferentiated, and clear cell), in Unni KK (ed): Dahlin's Bone Tumors: General Aspects and Data on 11,087 cases, ed 5. Philadelphia, PA, Lippincott‑Raven 1996, pp 71‑108. Wold LE, McLeod RA, Sim FH, Unni KK (eds): Atlas of Orthopedic Pathology. Philadelphia, PA, WB Saunders 1990, Ch 3, 10, 15, 23, 35.

Page 622: Oncology oite-review-2012

Question 166 -  Figures 41a through 41c show the axial T1- and T2-weighted images and the biopsy specimen of a 53-year-old patient who has a painless mass of the left proximal thigh. A radiograph and CT scan of the chest and a bone scan are normal. Management should consist of 1. radiation therapy and chemotherapy2. radiation therapy and wide excision3. radiation therapy and debulking4. wide excision alone5. marginal excision

Page 623: Oncology oite-review-2012
Page 624: Oncology oite-review-2012
Page 625: Oncology oite-review-2012
Page 626: Oncology oite-review-2012

Answer: 2

Using the TNM staging system, tumors are staged as I (low-grade G1), stage II (intermediate), stage III (high grade), and any tumor with spread (mets) is Stage IV. Using the AJCC system: tumors < 5 cm are T1, those > 5 cm are T2;

Any patient with a soft tissue mass should receive a CXR to initially screen. Bone scans or CT’s are used to detect metastases. Biopsy specimens are used to determine the type of soft tissue mass. This biopsy revealed fibroblastic type of nuclei, nuclei of various shapes and sizes, and occasional mitotic figures. The screening exams were negative for metastasis. The MRI revealed the mass to be fibrous (~ muscle on T1, bright on T2). Radiation therapy (followed by wide excision) is necessary for large tumors to improve chances for local control. Surgery is the mainstay for treating soft tissue sarcomas. Radiation therapy can be done either pre-operatively or post-operatively, however, pre-operatively it will shrink the tumor mass, and often cause the formation of a pseudocapsule around the mass, helping to further isolate/localize the tumor and reduce the amount of tissue resected (permitting limb-sparing surgery).  Chang, et al: Clinical evaluation and treatment of soft tissue tumors, in Enzinger and Weiss: Soft-Tissue Tumors, ed 3., 1995, pp 17-38

Page 627: Oncology oite-review-2012

Question 176 -  Figure 45 shows the AP radiograph of the pelvis of a 42-year-old woman who reports the insidious onset of anterior pelvic pain over the past 2 years. History reveals that she had a hysterectomy 7 years ago. Examination reveals localized tenderness over the symphysis pubis, and laboratory studies show an erythrocyte sedimentation rate of 2 mm/hr (normal up to 20 mm/hr). Microscopic examination of the involved area would most likely reveal 1. osteomyelitis.2. osteomalacia.3. osteonecrosis.4. neoplasia.5. chronic inflammation.

Page 628: Oncology oite-review-2012
Page 629: Oncology oite-review-2012

Answer: 5

5 Ref. JBJS 1989;71A:358‑364 Osteitis pubis is a painful, inflammatory, but non‑infectious condition involving the periosteum, bone, cartilage, and ligamentous structures in the region of the symphysis pubis (2,7,28). Because of the variety of pathological changes with which osteitis pubis may be associated, there has been considerable confusion in the literature as to the precise cause of the disease. In 1924, Beer{3 } first described osteitis pubis, which developed after urological procedures. Since that time, numerous cases have been reported in association with pregnancy, rheumatological disorders, trauma, infection, and athletic exertion, as well as after urological and gynecological operations.In the referenced article on patients with osteitis pubis: all had localized tenderness to palpation over the symphysis pubis, and anteroposterior radiographs, made with the patient standing, showed the bilateral involvement of the symphysis pubis that is characteristic of osteitis pubis. In the four patients who had been first seen less than six months after the onset of symptoms, the radiographs showed rarefaction and erosion of the adjacent pubic bones forming the symphysis and widening of the symphyseal joint space . In the six patients who were first seen more than six months after the onset of symptoms, the radiographs showed sclerosis of the adjacent pubic bones and narrowing of the symphyseal joint space.

Page 630: Oncology oite-review-2012

Question 177 -  A 72-year-old man reports a progressive increase in pain in his right hip, where he has had very mild aching pain for several years. Figure 46a shows the plain AP radiograph of the pelvis, and Figures 46b and 46c show the coronal T1- and T2-weighed MRI scans. A biopsy specimen from the femoral neck region is shown in Figure 46d. What is the most likely diagnosis? 1. Dedifferentiated chondrosarcoma2. Telangiectatic osteosarcoma3. Pagetoid osteosarcoma4. Metastatic carcinoma5. Bone infarct-associated osteosarcoma

Page 631: Oncology oite-review-2012
Page 632: Oncology oite-review-2012
Page 633: Oncology oite-review-2012
Page 634: Oncology oite-review-2012

Answer: 3 Explanation: AP radiograph and MRI of the pelvis show a lytic lesion involving the entire Right femoral head and neck. Biopsy specimen shows a highly cellular bony specimen with irregular thickened bony trabeculae. The bone marrow is replaced by vascular fibrous tissue with very irregular appearing cells. This specimen shows some evidence of the classic mosaic pattern of Pagetoid bone. Sarcomatous transformation in Paget’s disease occurs in 0.9% to 5.5% of cases, most commonly in Males (2 : 1), with an average age of 67 (42-78). Most commonly involving the pelvis, femur, humerus, and skull with most lesions occurring in the metaphyseal-diaphyseal region. These patients typically present with acute pain or a progressive increase in the intensity of chronic pain and swelling or associated soft tissue mass. Pathologic fracture is the initial presentation in 10-37% of the cases AP radiograph and MRI typically show a lytic lesion with irregular margins and cortical penetration often with an associated soft tissue mass. Biopsy, which is required for definitive diagnosis, shows bony trabeculae with the typical mosaic pattern and enlarged hyperchromic and irregular nucleated cells. Multinucleated giant cells are also typically seen.Telangiectatic osteosarcoma – Typically occurs in the 2nd decade of life. Radiographs show purely lytic lesion with no sclerosis. Biopsy of the lesion shows minimal osteoid production with spaces separated by Septa as seen in an aneurysmal bone cyst.Bone infarct-associated osteosarcoma – Typically occurs in the 1st or 2nd decade, and presents with acute onset of pain and swelling. The roentgenograms typically show a well-circumscribed lesion with mineralization at the periphery and irregularly mineralized soft tissue masses. Biopsy shows minimal osteoid production.Dedifferentiated chondrosarcoma – Radiographs show dense mineralization surrounded by poorly defined lytic lesions. Biopsy shows hyaline cartilage surrounded by cellular neoplasm.Metastatic carcinoma – Radiographs demonstrate irregular destructive lesions, mostly osteolytic with the biopsy showing glandular arrangements of epithelial cells in the stroma. Patients may have a know primary cancer and often present with systemic signs and symptoms of metastasis. References: Hadjipavlou A, Lander P, Srolovitz, et al: Malignant transformation in Paget’s disease of Bone. Cancer 1992;70:2802-2808.Unni KK; Osteosarcoma, in Unni KK (ed): Dahlin’s Bone Tumors: General Aspects and Data on 11,087 cases, ed 5. Philadelphia, PA, Lippincott-Raven, 1996, pp 143-183. 

Page 635: Oncology oite-review-2012

Question 188  A 12-year-old girl sustained several puncture wounds to her palm from cat bites. Twenty-four hours later the hand is swollen, erythematous, and painful. What organism is most likely associated with this infection? 1. Eikenella corrodens2. Proteus mirabilis3. Streptococcus4. Staphylococcus aureus5. Pasteurella multocida

Page 636: Oncology oite-review-2012

Answer: 5

The acute onset of cellulitis, lymphangitis, and serosanguineous or purulent drainage from hand wounds 12-24 hours after cat or dog bites should suggest Pasteurella multocida. P. multocida has been isolated in oral cavities of up to 70% of domestic cats and 66% of domestic dogs.The oral drug of choice for animal bites is Augmentin.

Reference: Aron MS, Fernando L: Pasteurella multocida: The major cause of hand infections following domestic animal bites.

Lucas GL, Bartlett DH: Pasteurella multocida infection of the hand. Plat Reconstr Surg 1981; 67:49-53.

Page 637: Oncology oite-review-2012

Question 189 -  Which of the following is considered the most common presenting symptom in a patient with a high-grade primary malignant bone tumor? 1. Weight loss2. Fatigue3. Fever4. Pain5. Malaise

Page 638: Oncology oite-review-2012

Answer: 4

The most common presenting symptom in people with high grade primary malignant bone tumor is pain. Weight loss, fatigue, fever and malaise are symptoms seen with malignancy. However they are usually not the presenting symptom encountered in primary malignant bone tumor. Source: McCarthy EF, Frassica FJ (eds): Pathology of Bone and Joint Disorder with Clinical and Radiological Correlation. Philadelphia, PA, WB Saunders, 1998, p197. 

Page 639: Oncology oite-review-2012

Question 193 -  Which of the following organisms can produce gas gangrene and may necessitate open amputation above the level of infection? 1. Pseudomonas aeruginosa2. Staphylococcus aureus methicillin-resistant3. Staphylococcus aureus coagulase-negative4. Clostridium perfringens5. Group D enterococcus

Page 640: Oncology oite-review-2012

Answer: 4DeHaven KE, Evarts CM: The continuing problem of gas gangrene: A review

and report of illustrative cases. J Trauma 1971;11:983-991.MacLennan has defined gas gangrene as an invasive, anaerobic infection of

muscle, characterized by extensive local edema, massive tissue necrosis, variable degrees of gas production, and profound toxemia.

The bacteriology of his condition has been well studied. There are six species of clostridia capable of producing gas gangrene (table 1). The most important of which is C. perfringens. These organisms are Gram-positive rods that seldom produce spores in the tissues or in culture media. They are obligate anaerobes, and cannot multiply in the relatively high oxidation-reduction potentials found in healthy tissues. Clostridia are widely distributed in nature, being found in soil and in feces of most animals, including man. BacteriologyA. Organisms B. Toxins Clostridium perfringens (C. welchii) Lecthinase (toxin )C. novyi CollagenaseC. septicum HyaluronidasC. histolyticum LeukocidinC. bifermentans DeoxyribonucleaseC. fallax Protease Lipase

Page 641: Oncology oite-review-2012

Laggaard SW, McElfresh EC,Premer RF: Gangrene of the upper extremity in diabetic patients. J Bone Joint Surg 1989;71A:257-264.Twenty-two patients who had diabetes mellitus and needed an amputation for gas gangrene in an upper extremity at an age of fifty-one years were identified and followed…Oe patient was not included in the study because the gangrene had been caused by an aggressive infection with Clostridium perfringens…Gangrene was thought to have developed spontaneously in seven upper extremities, as the patients were unable to recall an injury or infection that preceded the gangrene. Seven patients associated the development of gangrene with an episode of minor trauma: a cracked nail in two patients and a sliver, blister, fall, bump, and blow from a hammer in one patient each. The gangrene was preceded by spontaneous infection in eight patients, although the infecting organisms, which were always multiple, were identified in preoperative or intra-operative cultures in only four of these eight patients. The organisms that were identified as an initial infection were Staphylococcus (3 infections), Streptococcus (2 infections, Proteus (2 infections), Enterobacter (1 infection) and Veillonella (1 infection). In patients who did not have an initial infection, the cultures of secondary infections grew Staphylococcus twice, Proteus once, and Corynebacteria once.The results of amputation of the upper extremity were very unsatisfactory. In only two of the twenty-two patients did the surgical wound heal at the level that was initially selected, without additional surgical intervention…

Page 642: Oncology oite-review-2012

Question 200 -  Which of the following soft-tissue sarcomas most commonly shows prominent mineralization within the lesion on plain radiographs? 1. Rhabdomyosarcoma2. Liposarcoma3. Epithelioid sarcoma4. Synovial sarcoma5. Angiosarcoma

Page 643: Oncology oite-review-2012

Answer: 4

“Amorphous calcifications are frequently present (up to 30% of patients) in synovial sarcomas and thus this entity should always be considered in the differential diagnosis of a soft tissue mass that contains amorphous calcification.” Calcification can be observed in benign soft tissue lesions including nodular (pseudosarcomatous) fasciitis and myositis ossificans.

Page 644: Oncology oite-review-2012

Question 207 -  Figures 55a through 55c show the AP radiograph and T1- and T2-weighted axial

MRI scans of an 8-year-old boy who has a painless mass in the right antecubital space. The mass increases in size with activity and then decreases after activity. Gross blood is obtained during a needle biopsy. Management should include 1. wide resection.2. observation.3. open incisional biopsy.4. CT scan-guided core biopsy.5. radiation therapy.

Page 645: Oncology oite-review-2012
Page 646: Oncology oite-review-2012
Page 647: Oncology oite-review-2012
Page 648: Oncology oite-review-2012

Answer: 2 A hemangioma may be present at birth or may exist initially as a small red spot. Within several months these lesions undergo rapid growth and proliferation and usually reach maximal size by two to three years of age. Lesions within the hand and wrist are usually circumscribed and have no predominant location. By age five, spontaneous involution is underway. This process is heralded by gray areas of lighter pigmentation called "herald spots." Most of these lesions have the appearance of a "strawberry" if dermis is involved, but many deeper lesions will show few skin changes. CT scan will demonstrate hemangiomas to be homogeneous lesions with well‑defined borders. MRI demonstrates depth of structures as well as distinguishing these lesions from sarcomas and capillary malformations. Treatment is conservative. Compression garments for large lesions and reassurance of parents remains the treatment of choice. Earlier surgery is reserved for serious obstructions of the aerodigestive tract, obstruction of vision, gastrointestinal bleeding, and other life‑threatening conditions. Early surgery is rarely indicated in the upper extremity. OKU: Hand Surgery Update, Chapter 36: Pediatric Hand Tumors

Page 649: Oncology oite-review-2012

Question 212  Which of the following antibiotics is contraindicated in patients who require oral anticoagulants? 1.Trimethoprim2.Clindamycin3.Ciprofloxin4.Aminoglycoside5.Erythromycin

Page 650: Oncology oite-review-2012

Answer: 5

Erythromycin interacts with oral anticoagulants to increase prothrombin time. Sulfamethoxazole, Fluconazole, Metronidazole and Rifampin also increase prothrombin time. Trimethoprim combined with methotrexate is associated with megaloblastic pancytopenia. Clindamycin enhances effects of neuromuscular blocking agents. Ciprofloxacin increases theophyline levels while its absorption is decreased by antacids, iron and zinc salts, and Carafate. Aminoglycoside’s nephro- and ototoxicity is enhanced when combined with diuretics. Aminoglycides enhance respiratory suppression of neuromuscular blocking agents. Reference: Kasser JR (ed): Orthopaedic Knowledge Update 5. Rosemont, IL, American Academy of Orthopaedic Surgeons, 1996, ppp149-161

Page 651: Oncology oite-review-2012

Question 235 -  Figures 59a through 59c show the radiograph and biopsy specimens of an 11‑year‑old child who has distal thigh pain. What is the most likely diagnosis? 1‑ Nonossifying fibroma2‑ Aneurysmal bone cyst3‑ Intraosseous ganglion4‑ Periosteal chondroma5‑ Telangiectatic osteosarcoma

Page 652: Oncology oite-review-2012
Page 653: Oncology oite-review-2012
Page 654: Oncology oite-review-2012
Page 655: Oncology oite-review-2012

Answer: 2 An aneurysmal bone cyst is a solitary, expansile lesion of unknown etiology which is generally eccentric in location. These lesions are most commonly seen in individuals under 20 years of age, where swelling, pain, and/or tenderness may be the presenting complaint. The periphery of the lesion is often indistinct, and the tumor itself often has a trabeculated appearance. On histologic examination, the lesion is found to contain cystic spaces of different sizes which are filled with blood but are not lined with a vascular endothelium. Between the blood filled spaces are fibrous septa containing giant cells and foci of immature bone or osteoid. Bullough, PG: Orthopaedic Pathology, ed 3. Chicago, IL, Mosby‑Wolfe, 1997, pp. 402‑404.

Page 656: Oncology oite-review-2012

Question 243  A 12-year-old boy has pain at rest and at night over the proximal tibia. Plain radiographs show a 6mm lucent area in the cortex surrounded by sclerotic bone. What is the most likely diagnosis? 1. High-grade osteosarcoma2. Periosteal osteosarcoma3. Osteoblastoma4. Osteoid osteoma5. osteofibrous dysplasia

Page 657: Oncology oite-review-2012

Answer: 4

Osteoid osteoma is a benign tumor of bone that affects 5-20-year-olds. It has a male to female ratio of 3/1. Patients complain of night pain, relieved with ASA, NSAIDS or prostaglandin inhibitors. X-ray findings are consistent with the test question, that is a well circumscribed radiolucent nidus that is less than 2 cm in diameter. The lesion is typically surrounded by dense reactive bone.

The other choices are incorrect, not consistent with the history.Osteoblastoma would have a similar picture on X-ray except the

lucency tends to be greater than 2 cm. It also lacks night pain.Periosteal osteosarcoma and high-grade osteosarcoma typically

present with a painful, palpable mass, both with distinct X-ray findings.Osteofibrous dysplasia usually affects girls less than 10-years-old.

Pain is usually absent unless a pathological fracture occurred. May see an enlarged tibia with anterolateral bowing. On X-ray one will see a radiolucent, expansible and often bubbly appearance with well-defined margins in the metadiaphyseal anterior cortex of the tibia.

Page 658: Oncology oite-review-2012

Question 251 -  Figures 64a and 64b show the plain frog lateral radiograph and axial MRI scan of a 7-year-old child who has had left groin pain and a limp for the past 3 months. A biopsy reveals an aneurysmal bone cyst. Management should include 1. observation with serial radiographs.2. hip spica casting.3. curettage and bone grafting.4. curettage and cementation.5. en bloc wide excision.

Page 659: Oncology oite-review-2012
Page 660: Oncology oite-review-2012
Page 661: Oncology oite-review-2012

Answer: 3

The patient’s plain x-rays show an aneurysmal bone cyst of the left proximal Femur. In light of the patient’s 3 month history of groin pain and limp for 3 months, surgical curettage and bone grafting is indicated. Ref: Cancer, 1992; 2921-2931. 

Page 662: Oncology oite-review-2012

Question 257 -  Figures 65a and 65b show the hematoxin eosin stains of the biopsy specimen, and figures 65c and 65d show the T1 and T2 weighted axial MRI scans of a 36

year old woman who has a painless mass in the posterior thigh. Management should include: 1. marginal excision2. wide excision3. wide excision and radiation therapy4. radiation therapy alone5. amputation

Page 663: Oncology oite-review-2012
Page 664: Oncology oite-review-2012
Page 665: Oncology oite-review-2012
Page 666: Oncology oite-review-2012
Page 667: Oncology oite-review-2012

Answer: 1

Lesion in figures are most likely representative of neurilemmomas which are most commonly found in extremities. These lesions are usually treated by excision with preservation of nerve and major trunk. 

Page 668: Oncology oite-review-2012

Question 267  Figures 71a and 71b show the plain AP radiograph and coronal T1-weighted MRI scan of a 75 year old patient who has had progressively worsening pain in the left hip for the past 6 months. Figure 71c shows the incisional biopsy specimen. Staging studies show localized disease. Surgical treatment should include: 1. prophylactic fixation2. curettage and cementation3. curettage and bone grafting4. en bloc wide excision5. hemipelvectomy 

Page 669: Oncology oite-review-2012
Page 670: Oncology oite-review-2012
Page 671: Oncology oite-review-2012
Page 672: Oncology oite-review-2012

Answer: 4

The plain films, MRI, and biopsy clearly show a chondrosarcoma.“Low grade peripheral lesions (chondrosarcoma) have almost no risk of metastasis and an extremely low recurrence rate when a wide margin is achieved” Reference: Musculoskeletal Tumor Surgery. 1983. p 963-4. Enneking, WF, ed. Churchill Livingstone, New York, NY 

Page 673: Oncology oite-review-2012

14. An otherwise healthy 10‑year‑old boy has an erythematous, painful epitrochlear lymph node after visiting a relative with cats 3 days ago. Which of the following organisms is the most likely cause of this problem? 1‑ Bartonella henselae2‑ Mycobacterium marinum3‑ Eikenella corrodens4‑ Blastomycosis dermatitidis5‑ Pasteurella

OITE 2000

Page 674: Oncology oite-review-2012

Answer: 1 Bartonella henselae is one of four Bartonella species that have been shown to be pathogenic for humans. B. henselae is the cause of cat-scratch fever, a localized lymphadenopathy in a person who reports being in contact with or being scratched by a cat. The pathologic response is generally a granulomatous, suppurative, extracellular and intracellular one, self-limited, mild and usually unresponsive to antibiotic treatment. The diagnosis is made by biopsy of the involved lymph node and a positive skin test with sterile pus from the biopsy. The orthopaedic reference (Laskin and Potenza) presents two cases of cat-scratchfever that presented as a soft tissue mass in an otherwise healthy child. A tentative diagnosis of an atypical soft tissue mass was entertained, but open biopsy confirmed an infected, granulomatous process. Thus, an infectious process must always be entertained in the differential diagnosis of a soft tissue mass work-up. Mycobacterium marinum causes “swimming pool granuloma,” ulcerating, granulomatous lesions that occur in the skin at the site of abrasions incurred at swimming pools. The natural habitat is both fresh and salt water, with the treatment being tetracycline. Eikenella corrodens is a gram-negative rod found in the human mouth flora. Although not the most common organism (Alpha-hemolytic strept. and S. aureus are the most common) isolated from fight bite injuries (7-29%), it must be covered in the antibiotic regimen (i.e. Augmentin). Blastomycosis dermatitidis is a mold in the soil, endemic in North and Central America causing respiratory tract infections, but is usually asymptomatic and rarely recognized. Disseminated disease may result in ulcerative granulomas of skin, bone or other sites, with Ketoconazole the drug of choice. Pasturella multocida is a gram-negative rod found in the mouth flora of dogs and cats. About 26% of animal bites become infected with the organism (alpha-hemolytic strept most common 46%). Penicillin or augmentin remains the antibiotic of choice.  References:Bass JW, Vincent JM, and Person DA. The expanding spectrum of Bartonella infections: II. Cat-scratch disease. Pediatr Infect. Dis J. 1997: Feb;16(2):pp 163-79. Laskin RS and Potenza AD. Cat Scratch Fever—a confusing diagnosis for the orthopaedic surgeon. JBJS 1971; Vol 53-A, no.6, Sept. pp.1211-1214.

Page 675: Oncology oite-review-2012

33. Improvement in hip range of motion following cemented total hip arthroplasty in patients with ankylosing spondylitis has been found to be limited by 1‑ infection.2‑ neurologic involvement.3‑ heterotopic ossification.4‑ soft‑tissue contractures.5‑ ankylosis of the lumbar spine. 

Page 676: Oncology oite-review-2012

Answer: 3 Arthritis of the hip joint occurs in approximately 30% of patients with ankylosing spondylitis and, when present, it is usually bilateral. Cemented total hip arthroplasty has been shown to provide excellent relief of pain and improved ambulatory capacity. However, the improvement in the total range of motion of the hip was limited in several series due to the high incidence of heterotopic ossification. In one series of 29 hips, severe heterotopic ossification occurred in 23% of the hips. Prophylaxis against heterotopic ossification by means of either low-dose radiation or indomethacin therapy is generally recommended. References:OKU: Hip and Knee Reconstruction, 1996, pp 79-86.

Page 677: Oncology oite-review-2012

46. Which of the following processes is related to osteofibrous dysplasia (Campanacci's disease)? 1‑ Nonossifying fibroma2‑ Osteoid osteoma3‑ Adamantinoma4‑ Fibrosarcoma5‑ Ollier's disease (multiple enchondromatosis)

Page 678: Oncology oite-review-2012

Answer: 3  Osteofibrous dysplasia and adamantinoma are uncommon lesions that usually occur in the tibia. They were considered unrelated until the authors of recent publications suggested that osteofibrous dysplasia is either the benign counterpart of a neoplastic process that produces adamantinoma or that it is the residual of a spontaneously regressing adamantinoma. Alternate possibilities include osteofibrous dysplasia is a precursor to and may become adamantinoma, this, based on a few isolated cases of osteofibrous dysplasia apparently becoming adamantinoma. The importance of understanding their relationship is significant because observation is the recommended treatment for a patient with osteofibrous dysplasia. References:Springfield DS, Rosenberg AE, Mankin HJ, Mindell ER,: Relationship between osteofibrous dysplasia and adamantinoma. Clin Orthop 1994; 309:234-244. Bridge JA, Dembinoski A, DeBoer J, Travis J, Neff JR: Clonal chromosomal abnormalities in osteofibrous dysplasia: Implications for histopathogenesis and its relationship with adamantinoma. Cancer 1994;73:1746-1752. 

Page 679: Oncology oite-review-2012

54. Which of the following soft‑tissue lesions is best described as a Musculoskeletal Tumor Society stage 3 lesion (aggressive)? 1‑ Nodular fasciitis2‑ Lipoma3‑ Malignant fibrous histiocytoma4‑ Fibromatosis (extra‑abdominal desmoid)5‑ Giant cell tumor of the tendon sheath

Page 680: Oncology oite-review-2012

Answer: 4 MSTS stage 3 lesion is a benign aggressive lesion. Therefore one can exclude malignant fibrous histiocytoma. A giant cell tumor is an aggressive lesion but not a giant cell tumor of the tendon sheath. An extra-abdominal desmoid is the most locally invasive of all benign soft tissue tumors. The lesion infiltrates adjacent tissues. Therefore, it is the answer to this question. 

Page 681: Oncology oite-review-2012

79. A 28‑year‑old woman has had intermittent aching pain in the left ankle for the past year that is exacerbated by activity. Figures 17a through 17e show the plain radiograph, the coronal T1‑weighted MRI scan, the axial T2‑weighted MRI scan, and low‑ and high power photomicrographs. What is the most likely diagnosis? 1‑ Giant cell tumor2‑ Osteochondroma3‑ Enchondroma4‑ Chondromyxoid fibroma5‑ Chondroblastoma

Page 682: Oncology oite-review-2012
Page 683: Oncology oite-review-2012
Page 684: Oncology oite-review-2012
Page 685: Oncology oite-review-2012
Page 686: Oncology oite-review-2012

Answer: 4 Chondromyxoid fibroma usually presents as an eccentrically placed lytic lesion with well defined margins in the metaphysis of the lower extremity. The lesion usually has a sclerotic margin of bone. Usually presents in the 2-3 decade. These are all clues to the diagnosis, but as Dr. Mott says the proof is in the pudding, and the histology gives the diagnosis. Histologically, nodules of cartilage are found between nests of fibromyxoid areas. The chondrocytes are often spindle shaped. References: Scarborough MT, Moreau G: Benign cartilage tumors. Orthop Clin North Am 1996;27:583-589. 

Page 687: Oncology oite-review-2012

82. A 10‑year‑old girl has had knee pain for the past 3 months. History reveals that an incidental knee radiograph obtained 2 years ago showed no skeletal abnormalities. Current plain radiographs and a biopsy specimen are shown in Figures 18a through 18c. The patient's current condition is most likely associated with 1‑ familial infantile retinoblastoma.2‑ multiple hereditary osteochondromatosis.3‑ multiple enchondromatosis.4‑ polyostotic fibrous dysplasia.5‑ Gaucher's disease.

Page 688: Oncology oite-review-2012
Page 689: Oncology oite-review-2012
Page 690: Oncology oite-review-2012

Answer: 1 Retinoblastoma gene mutation is associated with development of osteosarcoma. The radiographs are consistent with osteosarcoma. The plain films show an aggressive lesion in the distal femur. As Dr. Mott says all lesions around the knee in the 2nd decade of life is osteosarcoma until proven otherwise. Histology confirms the diagnosis ass it shows tumor cells producing osteoid. A two step answer as 1st recognize the osteosarcoma diagnosis, then make the connection to retinoblastoma. RB gene is a tumor suppressor gene and when both alleles mutate, thus decreases tumor suppression and makes the subject more susceptible to oncologic sequelae. References: Hansen MF: Molecular genetic consideration in osteosarcoma. Clin Orthop 1991;270:237-246. 

Page 691: Oncology oite-review-2012

87. A 62‑year‑old man has had an enlarging painless mass on his thigh for the past 4 months. A biopsy specimen and MRI scans are shown in Figures 19a through 19c. This type of tumor will most often metastasize to which of the following structures? 1‑ Bone2‑ Lungs3‑ Liver4‑ Lymph nodes5‑ Kidney

Page 692: Oncology oite-review-2012
Page 693: Oncology oite-review-2012
Page 694: Oncology oite-review-2012
Page 695: Oncology oite-review-2012

Answer: 2 Soft tissue tumors metastasize to the lungs. You don’t really even need to look at the MRI and photomicrograph. You don’t need to waste any precious brain cells figuring out which type of tumor it is, except to realize it is big (more than 5cm), and since it is rapidly growing it will be malignant. I couldn’t find the soft tissue tumor book they referenced, but another tumor book by Enzinger and Weiss tells us that “by far the most frequent site of metastasis is the lungs”. (page 22) There are two series in the journal Cancer that have 26/36 (72%) and 69/77 (90%) of MFH mets show up in the lungs. (Bertoni, 1985 and Pezzi, 1992) The lymph node answer is the one to trip you up, but most of the tumors we will be faced with don’t do nodal mets as much as pulmonary. (Epithelioid sarcomas, embryonal rhabdomyosarcomas, angiosarcs and melanoma do nodes- you know enough from the pictures that the tumor isn’t one of those. Just remember lungs for all ortho tumor mets and you’ll be safe. References:Chang AE, Sondak VK: Clinical Evaluation and Treatment of Soft Tissue Tumors (ch 2) and Malignant Fibrous Histiocytoma (ch 15), in Enzinger FM and Weiss SW (eds): Soft Tissue Tumors, 3rd ed. St. Louis, Mosby, 1995, pp17-23 and pp351-69. Bertoni, F, Capanna R, Biagini R, Bacchini P, et al, Malignant Fibrous Histiocytoma of Soft Tissue. Cancer 1985; 56:356-67. Pezzi CM, Rawlings MS, Esgro JJ, Pollock RE, Romsdahl MM, Prognostic Factors in 227 Patients with Malignant Fibrous Histiocytoma. Cancer 1992; 69:2098-2103.

Page 696: Oncology oite-review-2012

92. A 52‑year‑old woman with no history of malignancy has had mild aching pain in the left tibia for the past 2 years. Examination reveals a firm mass in the subcutaneous border of the tibia; a bone scan shows this to be an isolated finding. Figures 20a through 20d show the plain radiograph, the sagittal Tl‑weighted MRI scan, the axial T2‑weighted MRI scan, and the biopsy specimen. Management should now include 1‑ serial radiographic observation.2‑ curettage and bone grafting.3‑ wide en bloc excision.4‑ chemotherapy and wide en bloc excision.5‑ prophylactic stabilization and radiation therapy.

Page 697: Oncology oite-review-2012
Page 698: Oncology oite-review-2012
Page 699: Oncology oite-review-2012
Page 700: Oncology oite-review-2012

Answer: 3 Whenever I hear about adamantinoma I think of Adam and the Ants, the punk rock group from the1980’s. Anyway. Must differentiate OFD / fibrous dysplasia / adamantinoma.  Adamantinoma is treated with wide en bloc excision. Mets can occur in up to 30 percent of cases which can lead to death (tends to occur in cases where incomplete resections were performed). Does not respond to chemotherapy or radiation therapy.OFD treatment is close observation. After the age of ten, the lesion should be biopsied and extensive histological study should be done to exclude the presence of epitheloid components, even if the radiological findings are highly suggestive for OFD. Large specimens need to be taken at biopsy to avoid missing tumour cells (no needle biopsy).

Page 701: Oncology oite-review-2012

93. Normal cortical bone has which of the following MR signal characteristics? 1‑ Low on T1‑weighted images and low on T2‑weighted images2‑ Low on T1‑weighted images and high on T2‑weighted images3‑ Moderate on T1‑weighted images and low on T2‑weighted images4‑ High on T1‑weighted images and low on T2‑weighted images5‑ High on Tl‑weighted images and high on T2‑weighted images 

Page 702: Oncology oite-review-2012

Answer: 1 The following is the chart that needs to be memorized! Tissue T1-weighted T2-weightedFat high intermediate to highArticular cartilage intermediate highFluid (joint, CSF) low highCortical bone low lowTendon/ligament low low /fibrocartilageHemosiderin low very low References:Beaty JH (ed): Orthopedic Knowledge Update 5. Rosemont, IL, American Academy of Orthopedic Surgeons, 1999, pp 81-87

Page 703: Oncology oite-review-2012

106. Figure 28 shows the AP radiograph of a 65‑year‑old woman who has mild shoulder pain and anemia. History reveals no significant illnesses or injuries. What is the next step in evaluation? 1‑ MRI scan2‑ Serum protein electrophoresis3‑ Serum alkaline phosphatase studies4‑ Open biopsy5‑ Needle biopsy

Page 704: Oncology oite-review-2012
Page 705: Oncology oite-review-2012

Answer: 2 This patient has radiographs consistent with a diagnosis of multiple myeloma. It is a neoplasm composed of plasma cells showing various degrees of differentiation. The process is usually multi-centric and diffusely involves the bone marrow. Increasing pain is the most common patient complaint, and is most often centered in the thoraco-lumbar spine. Most patients will also complain of weakness and weight loss. Pathologic fracture is common, again most commonly in the vertebral column. Diagnosis is confirmed by serum electrophoresis revealing a single monoclonal protein spike. The presence of Bence-Jones proteins in the urine and moderate to severe anemia are other common findings. Radiographically, these patients demonstrate multiple osteolytic, “punched out” lesions throughout the axial and appendicular skeletons. Bone scan is not helpful as these lesions appear cold secondary to a predominance of osteoclastic activity. An MRI would be superfluous; this is a plain film diagnosis. Biopsy is not indicated. Serum alkaline phosphatase elevation may be present in multiple neoplastic conditions, but not specifically in multiple myeloma. References:Unni KK: Dahlin’s Bone Tumors, ed 5, Philadelphia, PA, Lippincott-Raven, 1996, pp 225-236.

Page 706: Oncology oite-review-2012

111. Which of the following bone tumors is typically multifocal and involves bones in the same extremity? 1‑ Osteoblastoma2‑ Osteosarcoma3‑ Chondrosarcoma4‑ Chondroblastoma5- Hemangioendothelioma 

Page 707: Oncology oite-review-2012

Answer: 5 Hemangioendothelioma (Hemangiosarcoma) is a rare malignant vascular tumor of bone. Up to 1/3 of these patients will have multifocal lesions, which are predominantly lytic. This lesion can occur in any age group and usually presents with pain. Osteoblastoma is a benign bone-producing tumor, typically appearing in the spine, hip and proximal humerus. Radiographically, the bone destruction appears moth-eaten. Osteosarcoma is a malignant spindle cell neoplasm that produces osteoid. There are several variants of osteosarcoma including: intramedullary (classic), parosteal, periosteal, telangiectatic, occurring with Paget’s, and post irradiation. The most common is high-grade intramedullary osteosarcoma, which presents with pain about the knee of a child or young adult. Of these, 90% present with a stage IIB lesion (i.e. high-grade extra-compartmental, no metastases) and 10-20% have pulmonary metastases. Chondrosarcoma is malignant neoplasm of cartilage occurring in adults and older age groups. They usually present with pain or a mass in the knee, spine, shoulder and pelvis girdles. Chondroblastoma is a benign cartilage tumor centered in the epiphysis, triradiate cartilage of the pelvis and apophysis of young people (i.e. open physes). These patients also present with pain. References:Dorfman HD, Czerfniak B: Bone Tumors. St. Louis, MO, Mosby, 1998, pp 369-370. McCarthy EF, Frassica FJ: Pathology of Bone and Joint Disorders. Philadelphia, PA, WB Saunders, 1998, p 267.

Page 708: Oncology oite-review-2012

120. Figure 33 shows the AP radiograph of the femur of an asymptomatic 14‑year‑old patient. What is the most likely diagnosis? 1‑ Ollier's disease2‑ Paget's disease3‑ Fracture malunion4‑ Residual proximal femoral focal deficiency5‑ Fibrous dysplasia

Page 709: Oncology oite-review-2012
Page 710: Oncology oite-review-2012

Answer: 5

Figure 33: AP radiograph of proximal femur with large ground glass lesion extending distal to femoral neck. It has a well defined rim of sclerotic bone. There appears to be a slight varus bowing to the proximal femur possibly developing into a "Shepard's Crook Deformity." This is a radiographic diagnosis, in this case Fibrous Dysplasia. Fibrous Dysplasia is a developmental abnormality of bone which is either mono or polyostotic. When associated with skin (yellow or brown patches) and endocrine abnormalities (precocious puberty) in addition to multiple bone lesions the diagnosis of McCune-Albright syndrome can be made. Patients are usually asymptomatic the lesions being found incidentally on Xray. By far the most common location is the proximal femure. Radiographically Fibrous Dysplasia usually shows well defined zones of rarefaction (highly lytic or ground glass appearing), often surrounded by narrow rim of relatively sclerotic bone. These lesions may expand and thin the cortex especially in thinner bones. These lesions may contain large amounts of cartilidge which may show ring-like or dot-like calcification especially around femoral neck. These lesions can appear aggressive and suggesting diagnois of sarcoma. As for the other answers Ollier's Disease is associated with multiple enchondromas which have a different radiographic appearance, Paget's disease as well is entirely differnet having characteristic coarse trabeculae, remodeled cortices, and a blastic appearance. Identifying that a lesion is present would rule out fracture malunion and residual PFFD as answers.

Reference: Unni KK: Dahlin's Bone Tumors, Ed 5 1996 pp 355-433

Page 711: Oncology oite-review-2012

135. Which of the following is considered a risk factor for osteoporosis? 1‑ Obesity2‑ Mediterranean heredity3‑ Fair skin and hair4‑ A history of manual labor5‑ Late‑onset menopause 

Page 712: Oncology oite-review-2012

Answer: 3. “Genetic predisposition (individuals who are fair-skinned and small, have hypermobile joints, are of Northern European ancestry, or have scoliosis), cigarette smoking, and excessive alcohol intake are other risk factors.” References:Beatty JH (ed): Orthopaedic Knowledge Update 6. Rosemont, IL, American Academy of Orthopaedic Surgeons, 1999, p. 153 

Page 713: Oncology oite-review-2012

143. A 76‑year‑old woman has had generalized muscle weakness and arthralgias for the past 2 years. Radiographs show generalized osteopenia and a pseudofracture (Looser's zone) in the inferomedial aspect of the femoral neck. Laboratory studies show normal serum calcium and hemoglobin levels and a mildly elevated alkaline phosphatase level. What is the most likely diagnosis? 1‑ Osteomalacia2‑ Osteoporosis3‑ Multiple myeloma4‑ Leukemia5‑ Paget's disease of bone

Page 714: Oncology oite-review-2012

Answer: 1 Osteomalacia is a metabolic disorder in which there is inadequate mineralization of newly formed osteoid. It can result from vitamin D deficiency, vitamin D resistance, intestinal malabsorption, acquired or hereditary renal disorders, intoxication with heavy metal such as aluminum or iron, and other assorted etiologies. The childhood form of osteomalacia is termed rickets. The diagnosis of osteomalacia often is difficult because patients usually have nonspecific complaints such as muscle weakness of diffuse aches and pains. Radiographic evidence of osteomalacia often mimics other disorders including osteoporosis. However, the presence of pseudofractures or Looser’s transformation zones is good evidence that some degree of osteomalacia is present. Usual lab finds – elevated alkaline phosphatase, low calcium or low phosphorus levels. Serum assays for vitamin D metabolites help clarify the abnormality.Pagets disease of bone is characterized by bone resorption and formation, showing areas of wide lamellae and irregular cement lines, which produce the characteristic mosaic pattern appearance of pagetic bone. Two markers are used to follow the course of disease- serum alkaline phosphatase and urinary piridinium cross links.Osteoporosis is a silent disease, without specific laboratory findings. The only radiographic finding can be a fracture (most common wedging of vertebrae), or bone loss, which is detected on radiographs only after 30% to 50% of the mineral is lost.Multiple Myeloma will present as lytic lesions in bone, and not as Looser’s lines.Leukemia is more common in children, with skeletal involvement in 50% of cases. Diffuse osteopenia is the most frequent manifestation. Sometimes lucencies and periostitis may mimic osteomyelitis. References:Beaty JH (ed): Orthopedic Knowledge Update 5. Rosemont, IL, American Academy of Orthopedic Surgeons, 1999, pp427-439.

Page 715: Oncology oite-review-2012

150. What type of chondrosarcoma has the lowest 5‑year disease‑free survival rate? 1‑ Chondrosarcoma developing in an osteochondroma2‑ Chondrosarcoma developing in Ollier's disease3‑ Intermediate grade (grade 11)4‑ Mesenchymal5‑ Clear cell

Page 716: Oncology oite-review-2012

Answer: 4 Just Memorize.

Page 717: Oncology oite-review-2012

159. A 21‑year‑old marathon runner who is 5' 2" tall and weighs 95 lb reports the onset of left leg pain when she increased her training program 2 months prior to an event. History reveals that her last menstrual period was 18 months ago. Bone density of the spine tested by DEXA is 1.8 standard deviations below the mean for age. A bone scan is normal. Management should include 1‑ cessation of running and 10 mg of alendronate per day.2‑ nutritional counseling and a stretching program.3‑ an ultrasound of the heel and 500 mg of calcium per day.4‑ calcitonin, a short leg cast, and multivitamins.5‑ evaluation of the amenorrhea, achievement of nutritional balance, and cross training.

Page 718: Oncology oite-review-2012

Answer: 5 Amenorrhea in the female athlete may be related to a low body fat percentage and/or stress. The incidence approaches 50 % in elite runners and is related to stress fx (osteopenia) and eating disorders. Dietary management and birth control pills are helpful for treating the problem. A period of cross training would help to alleviate supraphysiologic stresses and maintain cardiovascular fitness (5 correct). A bone scan fails to show evidence of tibial stress fx so casting is not indicated (4 incorrect). Bisphosphonates are not indicated for this process which should be amendable to dietary management (1 incorrect).

References:OKU: Sports Medicine 2 pp 43-47.

Page 719: Oncology oite-review-2012

171. Normal mineralization of bone is seen in which of the following conditions? 

1‑ Rickets2‑ Seizure disorder treated with phenytoin3‑ Renal osteodystrophy4‑ Fanconi syndrome type II.5‑ Osteoporosis

Page 720: Oncology oite-review-2012

Answer: 5 

Rickets = Osteomalacia in Adults = failure of meneralization due to many causes. Labs are low normal Ca, low Pa, increased PTH, low Vit D. Many physical deformities result (Miller p31). Tx with Vit D (5000 IU daily) Phenytoin administration is one of the many causes of rickets. Renal Osteodystrophy is another cause of rickets which is a failure of meneralization  

Page 721: Oncology oite-review-2012

173. Which of the following bone lesions arises exclusively in the epiphysis or apophysis of the long bones? 1‑ Giant cell tumor2‑ Aneurysmal bone cyst3‑ Unicameral bone cyst4‑ Fibrous dysplasia5‑ Chondroblastoma 

Page 722: Oncology oite-review-2012

Answer: 5  Chondroblastomas (and infection) is the only lesion that arises from the epiphysis. The epiphyseal location is a major clue for making the diagnosis but 52% in the series quoted also involved the metaphysis. However none of the tumors arising in long bones involved only the metaphysis. The radiographic appearance of chondroblastomas of long bones is relatively typical when the tumor involves the epiphysis or an apophysis, but radiographic features are less typical for flat bone lesions. The most frequent feature of chondroblastomas in general was an eccentric oval or round lytic lesion that was well defined without a sclerotic rim and showed no sclerosis of the surrounding bone and that eroded or destroyed the adjacent cortex without periosteal reaction. A sclerotic rim and matrix calcification are helpful in making the diagnosis , but these features generally are not present. Chondroblastoma can be treated effectively with curettage and bone grafting. Giant cell tumors arise in both the epiphyseal and metaphyseal region of long bones. Aneurysmal bone cysts are located eccentrically in the metaphysis of a long bone and are usually well delineated. The unicameral bone cyst is a benign tumor that develops in metaphyseal bone adjacent to the physis. Fibrous dysplasia frequently affects more that one bone on the same side and is usually metaphyseal. References:Dorfman HD: Bone tumors. St.Louis, MO, Mosby, 1998, pp296-297 Turcotter RE: Chondroblastoma. Hum Pathol 1993;23:944-949

Page 723: Oncology oite-review-2012

183. An 11‑year‑old boy has an enlarging, slightly tender mass over the proximal phalanx of the third digit. The plain radiograph and a biopsy specimen are shown in Figures 39a and 39b. What is the most likely diagnosis? 1‑ Giant cell tumor2‑ Chondroblastoma3‑ Periosteal chondroma4‑ Aneurysmal bone cyst5‑ Nonossifying fibroma

Page 724: Oncology oite-review-2012
Page 725: Oncology oite-review-2012
Page 726: Oncology oite-review-2012

Answer: 3 Figure 39a shows a saucer-like erosion with a well-defined rim of reactive bone underlying the lesion which is characteristic of periosteal chondroma. Lesions may measure up to 4cm and calcifications may be present. Figure 39b shows lobules of hypercellular, immature cartilage; double nuclei; faint staining matrix and little, if any calcification, and mild cellular atypia which is consistent with this lesion. Periosteal chondromas are benign cartilaginous tumors forming beneath the periosteum and external to the cortex of bone. This extremely rare lesion, with few cases reported in the literature can cause saucerization of the underlying cortex and is mainly discovered in the third to fourth decades of life (case reports involve children as young as 4) and arises from subperiosteal cartilage formation. Male to female ration of 2:1. The classic location for this tumor is metaphyseal cortex of proximal humerus. Surgical treatment consists of resection of symptomatic larger lesions with a wide or marginal margin, recurrence with intralesional resection is common. Pain only occurs if the lesion is in proximity with a joint. Regression with maturity does not occur. Asymptomatic patients with latent lesions may be observed with serial radiographs. Chondroblastoma appear more aggressive radiologically, begin in the epiphyseal portion of long bones, generally are painful and patients present with associated joint swelling. Histologically this tumor shows immature chondroblasts, giant cells, less cartilaginous matrix and a greater degree of cellular atypia and hypercellular content. ABC’s histologically are composed of spindle cells, some bone formation and large bone cysts that usually are blood-filled and pain is the most common symptom. Giant cell tumors invariably present with pain and are usually located in the proximal tibia and histologically are composed of multinucleated giant cells and not cartilage matrix. NOF would not histologically show this immature cartilage matrix. References:Weiner SD, Iorio CD: Painless deformity of a long finger phalanx of a 4-year old girl. CORR 1999;369:357-65. Scarborough MT, Moreau G: Benign cartilage tumors. Orthop Clin North Am 1996;27:583-89. 

Page 727: Oncology oite-review-2012

184. A 30‑year‑old woman has chronic ankle pain and swelling without any history of trauma. Examination reveals diffuse swelling and soft‑tissue fullness along the anterior aspect of the ankle joint. Plain radiographs are normal. An MRI scan shows an effusion of the ankle and a soft‑tissue mass arising from the ankle joint that is dark on both Tl‑ and T2weighted images. A needle biopsy specimen is shown in Figure 40. Management should consist of 1‑ below‑knee amputation and chemotherapy.2‑ external beam radiation.3‑ extra‑articular resection and tibiotalar arthrodesis.4‑ open synovectomy.5‑ triple antibiotic therapy.

Page 728: Oncology oite-review-2012
Page 729: Oncology oite-review-2012

Answer: 4  Pigmented Villonodular synovitis is probably a proliferative reaction to some type of inflammatory agent. This reaction, characterized by giant cells, villous and nodular masses that fuse together in the synovial membrane to form a single mass. Arthroscopy and biopsy are valuable in making the diagnosis. Figure 40 shows nodules of glandular tissue w/ lipid filles histiocytes and giant cells. PVNS is relatively rare lesion, occurs in adults and the knee is the usual joint affected. The lesion produces a bulky mass in the synovial membrane and may even erode bone(this patient c/o mass at anterior ankle and has h/o swelling/inflammation/pain). The lesion in synovial joints responds well to surgical excision of the involved area of synovial membrane. For diffuse and widespread intra-articular disease, extensive synovectomy (either open or artroscopic is required. Antibiotics will not help, fusion is not necessary, radiation is not helpful and BKA and chemotherapy is too aggressive for this benign lesion. References:Mizel MS, Miller RA, Scioli MW (eds): Orthopaedic Knowledge Update: Foot and Ankle 2. Rosemont IL, American Academy of Orthopaedic Surgeons, 1998, pp11-26.

Page 730: Oncology oite-review-2012

195. An otherwise healthy 37‑year‑old man has had bilateral posterior heel pain for the past year. Examination reveals fullness, warmth, and tenderness over the posterior aspect of the heels. Radiographs are normal. Laboratory studies show a normal CBC and an erythrocyte sedimentation rate of 50 mm/h (normal up to 20 mm/h), and an HLA‑B27 is positive. What is the most likely diagnosis? 1‑ Rheumatoid arthritis2‑ Ankylosing spondylitis3‑ Lupus erythematosus4‑ Reiter syndrome5‑ Lyme disease

Page 731: Oncology oite-review-2012

Answer: 4 Given the clinical presentation (37yo man, bilateral heel pain, elevated ESR, and HLA B-27 +) Reiter syndrome is the logical choice. Reiter’s can consist of the triad of conjunctivitis, urethritis, and arthritis. Unknown cause, 96% + for HLA B-27, lower extremities usually involves knees, ankles, feet. Heel pain can be presenting symptom.Ankylosing Spondylitis - Mainly effects axial skeleton, adolescent males, 90% + for HLA B-27, + hip and BACK pain, XR show bilateral symmetric sacroiliac erosions, followed by joint space narrowing, then ankylosis and late “bamboo spine”. Limitation of chest wall expansion is specific for AS.Rheumatoid arthritis - Systemic disease affecting synovial tissues usually symmetrically, 3:1 female to male ratio, 90% have FOREFOOT involvement (hallux valgus, dorsal subluxation of plantar fat pad (walking on marbles).Lyme disease - spirochete (Borrelia burgdorferi) transmitted by deer tic (Ixodes dammini). Target shaped rash, fever, and systemic disease. Late musculoskeletal overuse syndrome. Endemic regions northeast U.S., Minnesota, Michigan, Oregon and California. Get Lyme titers. High index of suspicion needed. Tx with Zithromax.Lupus erythrematosis - multisystem autoimmune disease, 9:1 female to male ratio, usually 2-3rd decade of life, skin rashes, joints- nonerosive synovitis with little deformity, renal disorders-glomerular nephritis, hematologic disorders-anemias, neurologic- seizures. Titers of anti-DS DNA and anti-Sm antibodies are virtually diagnostic of lupus. References:Thomas FM, Mann RA: Arthritides, in Mann RA (ed); Surgery of the Foot and Ankle 1994 pp 618-619. 

Page 732: Oncology oite-review-2012

206. Which of the following diseases is characterized by a defect in type I collagen metabolism? . 1‑ Diastrophic dwarfism2‑ Osteogenesis imperfecta3‑ Mucopolysaccharidosis4‑ Pseudoachondroplasia5‑ Multiple epiphyseal dysplasia 

Page 733: Oncology oite-review-2012

Answer: 2  1-Diastrophic dwarfism: caused by abnormal sulfate transporter leading to undersulfated proteoglycans…proteoglycan swelling is part of the mechanism that produces growth2- Osteogenesis imperfecta: the defect is in the gene for the 1 and 2 chains of type I collagen3- Mucopolysaccharidosis: lysosomal enzyme defect that alters the ability to mobilize glycosaminoglycans (ex. Hunter’s and Hurler’s disease)4- Pseudoachondroplasia: mutation in fibroblast growth factor receptor 35- Multiple epiphyseal dysplasia: autosomal dominant defect in the pericentromeric region of chromosome 19 and 1 Reference:Simon SR (ed): Orthopaedic Basic Science. Rosemont, IL, American Academy of Orthopaedic Surgeons, 1994, pp 187-217.

Page 734: Oncology oite-review-2012

247. Figure 60 shows the bone mineral density (BMD) report for a 57‑year‑old Caucasian woman. According to the National Osteoporosis Foundation guidelines, management should consist of 1‑ observation and serial BMD testing.2‑ vitamin D and calcium dietary supplements only.3‑ pharmacologic treatment if additional risk factors for fracture are positive.4‑ pharmacologic treatment in the absence of additional risk factors.5‑ pharmacologic treatment only if the patient has a history of a osteoporotic fracture

Page 735: Oncology oite-review-2012
Page 736: Oncology oite-review-2012

Answer: 4 Physicians Guide to Prevention and Treatment of Osteoporosis: www.nof.org/physguide.Hip BMD is the best predictor of hip fractures and predicts fracture at other sites as well as other measurements. BMC is expressed as a relationship between two norms:

Z-score: expected BMD for the patient’s age and sexT-score: BMD for a normal, young adult of same sex.

The difference between the patients score an Z-score is expressed in the form of standard deviations above or below the mean. (one SD = 10-12% difference in bone density). The T-score is the parameter correlated with risk of osteoporotic fracture. Interpretation of T-score is as follows:

Above (-1)SD NormalBetween (-1)SD and (-2.5)SD OsteopenicBelow (-2.5)SD Osteoporotic

Treatment: 1) Everyone gets Calcium(1200 mg), Vitamin D(400-800 IU), regular weight bearing and muscle strengthening exercises, and advise to avoid tobacco and to keep alcohol intake to a minimum. 2) Pharmacologic therapy (Hormone replacement, Alendronate, Calcitonin, Raloxifene) is initiated when BMD T-score is <-2 and there are no risk factors or when T-score is <-1.5 and there are risk factors present. References:National Osteoporosis Foundation Development Committee: Osteoporosis: Physician’s Guide to Prevention and Treatment of Osteoporosis. Belle Mead, NJ, Excerpta Medica, 1998, pp 18-22.

Page 737: Oncology oite-review-2012

254. What is the rate of bone loss per year at menopause? 

1‑ 0.3% to 0.5% for 3 to 5 years2‑ 0.3 % to 0.5 % until age 65 years3‑ 2% to 3% for 2 to 3 years4‑ 2% to 3% for 6 to 10 years5‑ 4% to 5% for 2 to 3 years

Page 738: Oncology oite-review-2012

Answer: 4 In OKU 6 and in two supporting papers from the New England Journal of Medicine the rate of bone loss per year at menopause is 0.3% to 0.5% for 3 to 5 years.